You are on page 1of 78

MONTHLE

MARCH 2019

©LegalEdge Tutorials Page 1 of 76


Replication or other unauthorized use of this material is prohibited by the copyright laws of India
Contents Page
No.
From the Directors’ Desk 3
Victory Story
“I did SWOT analysis of my mocks, especially for AILET”: Utkarsh rathi 4
Current Affairs Timeline
National News 5
International News 6
Banking and Economy 7
Appointment & Resignations 7
Awards and Recognitions 8
Obituaries 10
Sports News 10
Defence and Space 11
Science and Technology 12
LE Explains
US-North Korea Peace talks 13
The Scheduled Tribes and other Traditional forest dwellers (Recognition of Forest Rights Act, 2006) 14
Geneva Convention and Indo-Pak Relations 15
Potpourri
20 Legal One Liners 16
Maxims and Foreign Phrases 16
LE Prep Assist
Mentor View General Knowledge 18
Prep Test – Legal Aptitude 19
Prep Test – Mathematics 35
Current Affairs Practice Sheet CAPS
Current Affairs Practice Sheet (CAPS) 39
Answer Key with Explanations
Answer Key with Explanations Prep Test 52
Answer Key with Explanations (CAPS) 57

Published by: LegalEdge Tutorials. 127, Second Floor, Zone-II, M.P. Nagar, Bhopal – 462011
Contact No: 9111555433
www.legaledge.in

©LegalEdge Tutorials Page 2 of 76


Replication or other unauthorized use of this material is prohibited by the copyright laws of India
From the Directors ' Desk

Dear Student,

We hope that your preparations for CLAT and AILET 2019 are in full swing. You must be pondering over the last-minute
preparations to push your mock scores even further. Here are some key points that will help you.

1. Believe that you will improve the most in the last 40 days; just like you improved for your XII-Boards in the last few
days.

2. Write a lot of offline Sectional Tests. Practice each and every assignment on an OMR Sheet.

3. Every morning sit down to write 1 thing from every section that needs immediate attention. Complete it by the end
of the day.

4. Between all the regular tasks, don't forget the following:

I.) Legal Maxims, Legal Aptitude & Current Legal Knowledge.

II.) Current Affairs of the previous 6 months (Well, there are close to 80-100 stories per month that are important.
Everything else is a waste of time. Ensure that you are through the complete details of these 80 stories).

III.) Solve Atleast 100Q from the 13 iconic Chapters of Mathematics (couple this with a lot of Offline Sectional Tests
-- GMB should be a very helpful collection of relevant Sectional tests).

IV.) High Frequency Words, Idioms and Phrases, 300 Common Errors of Grammar, Confusing Words are some of
the key words that you should search on Google to find relevant sources for studying English Language.

V.) Don't stop reading the editorials of the Hindu and Indian Express. These should even be read on the CLAT day.
You are not only reading the Editorials for the Current Affairs but also for getting used to a variety of topics that
can be asked in Reading Comprehensions (and even paragraph based Critical Reasoning).

VI.) Internalize the Verbal Reasoning book by R.S. Aggarwal. You should remember it word by word.

VII.) Solve every LegalEdge Mock Test, again. Note down the questions/concepts that you have to revise in the last
3 days.

5. Watch an occasional movie. Have an occasional Pizza. Give a small treat to your friends. Couple of hours a week
with the Play Station is just fine. In a nutshell, stay happy, energetic and motivated while preparing.

Harsh Gagrani Karan Mehta


(Director, LegalEdge) (Director, LegalEdge)

©LegalEdge Tutorials Page 3 of 76


Replication or other unauthorized use of this material is prohibited by the copyright laws of India
“I did SWOT analysis of my mocks, especially for AILET”: Utkarsh rathi
not with general knowledge (current affairs),
about which i had to be updated till the date of
CLAT. Meanwhile, I used to revise a subject at a
time during last 6 weeks. For continuous
upheaval I did SWOT analysis of my mocks,
especially for AILET.

4. Were you scared of any subjects in the


beginning? What did you do to overcome that?

I wouldn't say i was scared of any subject but


what I strived for was accuracy because both
CLAT and AILET not only test your aptitude
and intelligentsia but your accuracy and speed
Utkarsh Rathi
too, until and unless you don't have latter by
(Nalsar Hyderabad)
your side, you need to exert more.
1. How instrumental has LegalEdge been to you
5. How did you analyze your mocks?
for your preparation for CLAT?

One thing all the clat aspirants will realize is that


I did my Clat prep along with my 12th grade
the mock is not done within those prescribed
which is why I needed apt guidance to
120 minutes as its analysis and evaluation hold
manage time for both the things and Legaledge
indispensable importance. Legaledge's mock
did just the same for me. Legal edge study
tracker and digital statistical graphs helped me
material being best in the business gave me an
analyse my mocks thoroughly. Mocks are the
edge from the very beginning. Constant support
only way through which one can enhance
of the faculty members and regular mocks
his/her performance, so do analyze your and
analysis proved quite instrumental to me.
legal edge's digital support will make it quite
easy.
2. How did you prepare for AILET? Was your
approach different or same for both CLAT and
6. Who was your inspiration throughout your
AILET?
preparation?
I started off with the same approach for both of
Though I m not from a legal background but
them but there being differences in the exam
law as a profession always excited me and
pattern and mode of exam I made few
probably because of this reason I always had
adjustments during the later part of my prep.
that fire and zeal with into break top National
Also, since AILET tests you more on
Law University of India. My other batchmate's
constitution and its an offline exam therefore,
hard work and faculty members constant words
paying heed to that I had to make few finer
of advice too influenced me. Atlast I can say that
changes specifically for Ailet.
my dream to make it big in the legal society kept
me on the right track throughout my clat
3. What was your revision strategy while
preparation.
preparing for CLAT?

For clat and other relevant law entrance exams


you can get done with any of the subjects but

©LegalEdge Tutorials Page 4 of 76


Replication or other unauthorized use of this material is prohibited by the copyright laws of India
Current affairs Timeline

NATIONAL NEWS Delhi IGI Airport becomes the 12th busiest in the
India Hosts ‘4th Global Digital Health world as per ACI report
Partnership Summit’
J P Nadda, Union Minister of Health and Family
Welfare inaugurated the ‘4th Global Digital Health
Partnership Summit’ in New Delhi. The global
intergovernmental meeting on digital health is
being hosted by the Ministry of Health and Family
Welfare in collaboration with the World Health
Organization (WHO) and the Global Digital
Health Partnership (GDHP).
The nation capital’s Indira Gandhi International
India To Attend OIC Meet for the 1st Time
Airport (IGIA) has now become the world’s 12th
External Affairs Minister Sushma Swaraj has been
busiest airport, moving up four places from the
invited to deliver an address at the Organisation of
16th spot in 2017. According to the preliminary
Islamic Cooperation (OIC) in Abu Dhabi. This is
world airport traffic rankings for 2018 released by
the first time that a leader of the non-Muslim
the Airports Council International (ACI) , it over
country has been invited to the OIC
took mega-hubs like Frankfurt, Dallas Forth Worth,
conference. Mrs Swaraj will be the Guest of Honour
Guangzhou and Istanbul Ataturk airports.
and address the inaugural plenary of the 46th
Martyr’s Day: 23rd March
Session of the Council of Foreign Ministers of OIC
The nation is observing Shaheed Diwas or
on the 1st and 2nd of March.
Martyr’s Day on 23rd March. Each year, March 23,
Kerala’s Marayoor Jaggery Gets GI Tag
homage is paid to the great revolutionary
Idukki’s The Marayoor Jaggery, the traditional and
fighters Bhagat Singh, Rajguru and Sukhdev who
handmade product from Idukki district of Kerala
sacrificed their lives for the country. It was on
received the Geographical Indication (GI) tag from
March 23, 1931, three young freedom fighters were
the Central Government. The Marayoor jaggery
hanged to death in the Lahore Central Jail (Now in
finally managed to get the GI Tag after two years of
Pakistan).
continuous efforts by the Agricultural Department
Erode Turmeric receives GI tag
of the state.
Erode Turmeric has finally received GI tag from the
Geographical Indication Registry. Erode is the city
in the state of Tamil Nadu. The product to acquire
GI tag has to indicate that a product of a particular
origin has a certain quality or reputation or some
other characteristics, which is essentially
attributable to its geographical origin.
NGT slaps Rs 500 crore fine on Volkswagen India Ranks 16th In Terms Of Highest Number
The National Green Tribunal has slapped a fine of Of Impacted Species In Hotspots
Rs 500 crore on German auto major Volkswagen for A recent study published in PLOS Biology found
damaging the environment through the use of that human impacts on species occur across 84% of
“cheat device” in its diesel cars in India. A bench the earth’s surface and India ranks 16th in such
headed by NGT chairperson Justice Adarsh Kumar impacts, with 35 species impacted on an
Goel directed the carmaker to deposit the amount average. Malaysia ranks 1st among the countries
within two months. with the highest number of impacted species (125).
India ranks 76th on WEF Global Energy India Ranks 11th In Gold Holding
Transition Index India, which is the world’s largest consumer of
India has moved up two places to rank 76th on a gold, has the 11th largest gold reserve, with the
global energy transition index. This annual list, current holding pegged at 607 tonnes, as per the
compiled by Geneva-based World Economic latest report by the World Gold Council (WGC).
Forum (WEF), says India is amongst the countries The top spot is occupied by the US with gold
with high pollution levels and has a relatively high reserves of 8,133.5 tonnes, followed
CO2 intensity in its energy system. However, India by Germany with 3,369.7 tonnes.
is the only amongst the five economies to improve Mediation Panel Created: Ram Mandir-Babri
its rank since last year. Masjid Case

©LegalEdge Tutorials Page 5 of 76


Replication or other unauthorized use of this material is prohibited by the copyright laws of India
The Supreme Court ordered a court-monitored World Meteorological Day takes place every year
mediation in the Ram Janmabhoomi-Babri on 23 March and commemorates the coming into
Masjid land dispute case to arrive at a “permanent force on 23 March 1950 of the Convention
solution”. The five-judge constitution bench establishing the World Meteorological
headed by CJI Ranjan Gogoi appointed a panel of Organization. The theme for this year is “The Sun,
3 mediators in the title suit with Retired Justice the Earth and the Weather”.
Kalifullah chairing the court-appointed and World’s largest e-waste recycling hub opens in
monitored mediation process. The other two Dubai
members are spiritual leader Sri Sri Ravi The world’s largest e-waste recycling facility has
Shankar and senior advocate Sriram Panchu. opened in Dubai. Located at the Dubai Industrial
Madhya Pradesh Government Increases Park, the 280,000 square feet plant will
Reservation Quota of OBCs to 27% process Waste Electrical and Electronic
The Madhya Pradesh government issued an Equipment (WEEE), IT asset disposition (ITAD),
ordinance increasing reservation quota for refrigerant gas and specialised waste.
the Other Backward Classes (OBC) to 27% from Kazakhstan to rename country’s capital Astana as
the current 14%. The announcement was made by Nursultan
the state’s Law and Legal Affairs Minister PC Kazakhstan’s parliament voted to rename the
Sharma. country’s capital Astana as Nursultan in honour of
Assam’s Hailakandi Became The Top longtime ruler Nursultan Nazarbayev, a day after
Aspirational District he resigned as president. Astana is now officially
According to the Delta ranking report released renamed Nursultan. Kazakhstan’s new interim
by NITI Aayog, Assam’s Hailakandi has become president Kassym-Jomart Tokayev proposed
the top aspirational district among the 112 renaming the capital.
aspirational districts of the country. Hailakandi in
Assam has made a giant leap from 52nd position to
no. 1 position on the basis of its performance during
November-December 2018 and January 2019

INTERNATIONAL NEWS
Zimbabwe Starts Trading New Currency
International Day for the Elimination of Racial
Discrimination
The International Day for the Elimination of
Racial Discrimination is observed annually on 21
March. On that day, in 1960, police opened fire and
killed 69 people at a peaceful demonstration in
Sharpeville, South Africa, against the apartheid
pass laws. The theme of this year is, “Mitigating
Zimbabwe started to trade its new currency, the and countering of rising nationalist populism and
RTGS dollar, two days after the central bank extreme supremacist ideologies”.
UN Releases Global Environment Outlook 2019
announced measures to try and resolve a chronic
Report
monetary crisis. The new currency replaces
The United Nation Environment Program
electronic bank savings called digital dollars and
(UNEP) has released the sixth edition of the Global
the bond notes and was named after the real-time
Environment Outlook (2019) titled ‘Healthy
gross settlement system that banks use to transfer
Planet, Healthy People’. The report calls on the
money between each other.
decision makers to take immediate action to
Donald Trump Signs SPD-4
address pressing environmental issues to achieve
US President Donald Trump signed Space
the Sustainable Development Goals as well as
Policy Directive-4 (SPD-4),ordering the Pentagon
other internationally agreed environment goals,
to establish the Space Force as the 6th branch of the
such as the Paris Agreement.
United States military, to go along with the Army,
Vienna Topped The Mercer’s Quality Of Living
Navy, Air Force, Marines, and Coast Guard, in
Survey 2019
order to secure and extend American dominance of
The human resource consulting firm, Mercer’s
the space domain.
annual Quality of Living Survey 2019 has placed
World Meteorological Day

©LegalEdge Tutorials Page 6 of 76


Replication or other unauthorized use of this material is prohibited by the copyright laws of India
Austria’s capital city Vienna at the number 1 spot,
making it the 10th year in a row. The only cities
outside of Europe to make the top 10 list are
Vancouver, Canada, and Auckland, New Zealand. APPOINTMENTS AND
Hyderabad and Pune, at a rank of 143, continue to RESIGNATIONS
be the most liveable cities in India for expatriates.
Next Chief of Naval Staff
International Women’s Day
International Women’s Day is celebrated
on March 8 every year.
2019 campaign theme: #BalanceforBetter.
Malaysia joins International Criminal Court
Malaysia has become a member of the International
Criminal Court. The ICC is the world’s only
permanent war crimes court and aims to prosecute
The Government of India has appointed Vice
the worst abuses when national courts are unable
Admiral Karambir Singh,presently Flag Officer
or unwilling. Malaysia becomes the 124th member
Commander-in-Chief (FOC-in-C) Eastern Naval
of the court since its establishment in 2002.
Command as next Chief of the Naval Staff with
World Wildlife Day: March 3
effect from May 31, 2019. The present Chief of
World Wildlife Day is observed on 3rd
Naval Staff Admiral Sunil Lanba retires on the
March every year to spread awareness about
same day.
wildlife. The theme for World Wildlife Day 2019
Dr. A.K. Mohanty Takes Over As Director BARC
is ‘Life below water: for people and planet’, which
Dr. A.K. Mohanty, Distinguished Scientist and
corresponds to goal 14 of the Sustainable
Director, physics group of the Bhabha Atomic
Development Goals (SDGs) which is ‘Life below
Research Centre (BARC) and Director, Saha
water’.
Institute of Nuclear Physics, Kolkata took over as
Director, BARC from Mr. K.N. Vyas, Chairman,
BANKING AND ECONOMY Atomic Energy Commission and Secretary to
Women’s Livelihood Bond Launched Soumya Swaminathan Named Chief Scientist
World Bank, United Nations Entity for Gender
Equality and the Empowerment of Women (UN
Women), and Small Industries
Development Bank of India (SIDBI) launched
new social impact bonds exclusively for women,
called Women’s Livelihood Bonds (WLBs).
India’s GDP Forecasts by Fitch Ratings
In its Global Economic Outlook report, Fitch Soumya Swaminathan, a Deputy Director-General
Ratings has cut India’s economic growth forecast of the World Health Organisation has been named
for the next financial year 2019-20 starting from Chief Scientist of the WHO. She was one of the
April 1 to 6.8 per cent from its previous estimate of three DDGs assisting WHO Director-General
7 per cent, on the account of weaker than expected Tedros Adhanom Ghebreyesus.
momentum in the economy. However it sees Chairman Of the National Commission For
Indian GDP growth to hold up reasonably well, at Backward Classes
6.8 per cent, followed by 7.1 per cent in 2021-22. Bhagwan Lal Sahni has been appointed as
OECD cuts global economic growth forecasts the chairman of the newly constituted National
again Commission for Backward Classes (NCBC).
The OECD cut forecasts again for the global Kaushalendra Singh Patel, Sudha Yadav and
economy in 2019 and 2020, following on from Achary Talloju have been appointed as members of
previous downgrades in November, as it warned the commission.
that trade disputes and uncertainty over Brexit Naresh Goyal steps down as Jet Airways
would hit world commerce and businesses. The Chairman
Organisation for Economic Co-Operation &
Development forecast in its interim outlook report
that the world economy would grow 3.3 percent in
2019 and 3.4 percent in 2020.

©LegalEdge Tutorials Page 7 of 76


Replication or other unauthorized use of this material is prohibited by the copyright laws of India
disputes related to cricket administration within
the cricket body, the Board of Control for Cricket
in India (BCCI). Narasimha will hear the BCCI’s
position, and then make recommendations to
the Committee of Administrators (CoA).

AWARDS, HONORS AND


Jet Airways Chairman Naresh Goyal stepped
RECOGNITIONS
down, paving way for lenders to bail out the Oscars 2019
The 91st Academy Awards (Oscars) have been
financially troubled airline he set up 25 years ago.
The airline has over one billion dollars in debt and announced at the Dolby Theatre in Hollywood,
has to repay money to banks, lessors of planes and the USA. Rami Malek was named Best Actor in a
Leading Role for ‘Bohemian
suppliers besides clearing pending salaries to its
pilots. Rhapsody’, while Olivia Colman won Best Actress
Justice P.C. Ghose Named India’s First Lokpal Oscar for ‘The Favourite’ at the 91st Academy
Former Supreme Court judge Justice Pinaki Awards. India-based ‘Period. End of
Chandra Ghose was recommended to be the first Sentence.’ has won the Oscar for Best
Lokpal or anti-corruption ombudsman of Documentary Short Subject.
India. Justice Ghose, 67, is a member of the CISF Sets Guinness Record
National Human Rights Commission (NHRC)
since June 2017.
Pramod Sawant Sworn In As Goa Chief Minister

The Central Industrial Security Force


(CISF) created the Guinness World
Pramod Sawant (45), former Speaker of Goa Record for ‘longest single line bicycle parade
legislative Assembly and MLA from Sankhelim (moving)’ at the Yamuna Expressway, Noida. CISF
in North Goa was sworn-in as 11th Chief Minister personnel rode 1,327 bicycles non-stop in a single
of Goa, succeeding Manohar Parrikar, who died line, maintaining a uniform distance between the
after a prolonged battle with advanced pancreatic bicycles. The record was earlier held by Hubballi
cancer. Bicycle Club of India, with 1,235 bicycles in a single
Mohammad Shtayyeh Named As Palestinian PM chain.
“World’s 100 Most Influential People in Climate
Policy for 2019”
The “100 Most Influential People in Climate
Policy” list comprises seven Indian
names including Union Ministers Piyush Goyal
and Dr. Harsh Vardhan, who took important
measures to combat climate change. The list has
been topped by Alexandria Ocasio-Cortez
(Congresswoman, US Congress). The other 5
Mohammad Shtayyeh has been named names from India: Mukta Tilak– Mayor of
as Palestinian Prime Minister by President of Pune, Jyoti Kirit Parikh- Executive director of
Palestinian Authority Mahmoud Abbas. A Integrated Research and Action for
member of the West Bank’s dominant Fatah party, Development, Sunita Narain- Director of the
he is a longtime ally of Abbas. Centre for Science and the Environment, Vandana
Mediator To Resolve Disputes Within BCCI Shiva- Founder of the Research Foundation for
The Supreme Court appointed senior advocate PS Science, Technology, and Natural Resource
Narasimha as a mediator for resolving various

©LegalEdge Tutorials Page 8 of 76


Replication or other unauthorized use of this material is prohibited by the copyright laws of India
Policy Upendra Tripathy- Director general of the Prize, worth $1.4m, for his work blending science
International Solar Alliance. and spirituality. Gleiser is the first Latin American
Kenyan science teacher Peter Tabichi wins 2019 to win the award, which honours “a living person
Global Teacher Prize who has made an exceptional contribution to
A science teacher from rural Kenya, who gives affirming life’s spiritual dimension”.
away most of his salary to support poorer pupils, Mukesh Ambani tops list of world’s richest sports
has won a $1m prize (£760,000) for the world’s best team owners
teacher. Peter Tabichi, a member of the Franciscan Reliance Industries chairman and Asia’s richest
religious order, won the 2019 Global Teacher man Mukesh Ambani recently topped Forbes’
Prize. The World’s Richest Sports Team Owners 2019
Tata Steel: One Of World’s Most Ethical list. With a total net worth of about $50 billion,
Companies Ambani topped the list. He bought the IPL
The award has been given by Ethisphere team Mumbai Indians for over $100 million in 2008
Institute,USA. The company was also recognised through a RIL subsidiary.
7 times previously and is one of the only two BusinessLine Changemaker Of The Year
awardees in the ‘Metals, Minerals and Union Finance Minister Arun Jaitley received the
Mining’ industry. In 2019, 128 honourees across 21 award from Manmohan Singh for the successful
countries and 50 industries have been recognised. implementation of the Goods and Services Tax
DRDO Chairman Wins Missile Systems Award (GST). The award was also presented to the
Satheesh Reddy, the Secretary of India’s petitioners against Section 377 of the Indian Penal
Department of Defence R&D and Chairman of Code (IPC), which criminalised homosexuality.
the Defence Research and Development Dipa Karmakar Selected As Barbie Role Model
Organisation (DRDO) has been chosen by
the American Institute of Aeronautics and
Astronautics (AIAA) as the co-winner of the 2019
Missile Systems Award. The other co-winner
is Rondell J. Wilson who is a former Principal
Engineering Fellow of Raytheon Missile Systems.
International ‘Golden City Gate Tourism Awards
2019’ In Berlin
The Ministry of Tourism, Government of Indian gymnast Dipa Karmakar was selected as
India has won the First Prize in the category of TV Barbie role model & was presented with a one-of-
Cinema Spot at the prestigious international a-kind doll by Barbie as the company celebrates
Golden City Gate Tourism Awards 2019. The its 60th anniversary. She was part of the ‘Shero’
awards were received by Yogendra Tripathi, program launched in 2015 by the company.
Commonwealth Youth Award For The Asian
Secretary, Tourism at ITB, Berlin.
Region
BBC journalist Priyanka Dubey gets Chameli
Devi Jain Award A social entrepreneur from Tamil
Priyanka Dubey, a bilingual correspondent with Nadu, Padmanaban Gopalan, founder of No Food
the BBC at its Delhi bureau, has been named for Waste, was named the winner of
the Chameli Devi Jain Award for an Outstanding the Commonwealth Youth Award for the Asian
Woman Journalist for the year 2018.Dubey was region. His initiative runs a geo-mapping platform,
chosen for her multi-faceted, investigative and which allows users to drop food at located “hunger
interrogative reportage. spots” and has so far recovered over 650,000
Brazilian astronomer Marcelo Gleiser wins 2019 meals across 14 cities and the platform has been
Templeton Prize used by over 12,000volunteers.
World’s Oldest Person Alive

Brazilian physicist and astronomer Marcelo


Gleiser has been awarded the 2019 Templeton A 116-year-old Japanese woman who loves
playing the board game Othello is being honored
©LegalEdge Tutorials Page 9 of 76
Replication or other unauthorized use of this material is prohibited by the copyright laws of India
as the world’s oldest living person by Guinness SPORTS
World Records. Tanaka was born January 2, 1903, Manpreet Singh Honoured With AHF’s Player Of
the seventh among eight children. The previous The Year
oldest living person was another Japanese woman,
Chiyo Miyako, who died in July at age 117.

OBITUARIES
Senior journalist, environmental activist Darryl
D’Monte passes away
The Asian Hockey Federation has honored Indian
Senior journalist and environmental activist
skipper Manpreet Singh with the 2018 Player of
Darryl D’Monte, who worked with leading
the Year award. Women team’s
English dailies, passed away at a hospital in
striker Lalremsiami has bagged the Rising Player
Mumbai after a brief illness. He was 74. In a career
of The Year prize. Manpreet led the Indian team to
spanning several decades, he served as the resident
an unbeaten streak at the Asian Champions Trophy
editor of Mumbai editions of The Times of India
in Muscat where the team was declared joint
and The Indian Express.
winners with Pakistan.
Actor Luke Perry Passes Away
Roger Federer Claims 100th ATP Title Of His
Career
Roger Federer has claimed the 100th ATP title of
his career after beating 20-year-old Greek Stefanos
Tsitsipas in the final of the Dubai Tennis
Championships. The 20-time grand slam
champion is the second man after American Jimmy
Connors to claim 100 titles. Martina
American actor Luke Perry has passed away Navratilova holds the all-time record having
in California at the age of 52, less than a week after won 167 women’s singles crowns during her
suffering a massive stroke. The actor rose to fame career.
for his roles in ‘Beverly Hills 90210′ and 500 Sixes In International Cricket
“Riverdale“. West Indies opener Chris Gayle become the first
Andre Previn, 4 Time Oscar Winner Passes Away batsman ever to smash 500 or more sixes in
international cricket. In the 4th ODI against
England in Grenada, Gayle scored 162 runs. His
innings contained a staggering 14 sixes and
powered the Windies to their highest-ever ODI
score of 389.
Sanath Jayasuriya banned after ICC anti-
corruption probe
Legendary composer and pianist Andre Previn has
The former Sri Lanka captain Sanath Jayasuriya has
passed away at the age of 89 in Manhattan, New
been banned for two years from all cricket-related
York, USA. He was a 10-time Grammy Award
activity by the ICC’s anti-corruption unit. He was
winner and had won 4 Oscars throughout his
punished for refusing to co-operate with
lifetime.He is known for immortal classics like
investigations concerning corruption in the
1964’s ‘My Fair Lady‘ and 1996’s ‘The Fortunate
country. Jayasuriya is also a former member of Sri
Cookie.’
Lanka’s parliament and a former deputy minister.
Nobel Laureate Zhores Alferov Passes Away
India Clinch SAFF Women’s Championship For
Zhores Alferov, a Russian physicist and Nobel
5th Time
Prize laureate, has passed away. He was 88. In
India has lifted South Asian Football Federation
2000, Alferov received the Nobel Prize in Physics
SAFF Women’s Championship 5th time in a row.
together with US scientists Jack Kilby and Herbert
Keeping its winning streak India defeated host
Kroemer, for developing semiconductor
Nepal in the final by 3-1 in Biratnagar, Nepal.
heterostructures used in high-speed-and
India wins 368 medals at Special Olympics World
optoelectronics. Summer Games
India returned with 368 medals, including 85
gold, at the Special Olympics World Summer

©LegalEdge Tutorials Page 10 of 76


Replication or other unauthorized use of this material is prohibited by the copyright laws of India
Games which was held at Abu Dhabi, UAE from
March 14 to 21. The Indian team of 284 athletes also
clinched 154 silver and 129 bronze medals in its
highly successful campaign at the
Asian Youth Championship 2019 Concludes:
India Finishes 2nd, China Tops
In Asian Youth Athletics Championships in Hong
Kong, boys medley relay team won a gold medal
for India on the final day to help Indian youth team
finish second in the overall medals tally with Mercedes team Formula One (F1) driver Valtteri
a total of 26 medals including 8 gold, 9 silver and Bottas won his first champion in the Australian
9 bronze medals. Grand Prix held in Melbourne. It was the fourth
Bengaluru FC Crowned ISL Champions For 1st time in as many years that Hamilton has taken pole
Time position in Australia only to finish second.
Bengaluru FC clinched their maiden Indian Super India To Host U-17 Women’s Football World Cup
League title as Rahul Bheke found a In 2020
championship-winning header in the dying India will host the Under-17 Women’s Football
minutes of the final against FC Goa, in Mumbai. World Cup in 2020. This was announced by the
Youngest Indian woman to win Ladies European President of International Football Federation
Tour (FIFA) Gianni Infantino after the council meeting
in Miami, USA.
Upamanyu Dutta Clinches Bronze In Asian
Regatta
Young Mumbai sailor Upamanyu Dutta bagged
the bronze medal in the Laser Standard race for
men in the Laser Asian Sailing Championship
which concluded in Singapore. Nethra Kumanan
of Chennai achieved a creditable 4th position in the
Laser Radial event for women.
Finland Boxing Tournament: India Wins 1 Gold,
Diksha wrote her own piece of history by 4 Silver
becoming the youngest Indian woman to win the Kavinder Singh Bisht in the 56kg-category struck
Ladies European Tour (LET) in Cape Town, South gold while Shiva Thapa in the 60kg category and
Africa. In 2016 Diksha was given a place to play at three others claimed silver medals to end a fine
India’s biggest women’s pro event, the Hero Indian campaign at the 38th GeeBee Boxing
Women’s Indian Open. Tournament in Helsinki, Finland.
K T Irfan becomes first Indian athlete to qualify
for Tokyo Olympics
National record holder K.T. Irfan became the first
DEFENCE AND SPACE
Indian from athletics to qualify for the next year’s Cobra Gold Military Exercises Held In Thailand
Olympics while finishing fourth in the 20 km event Thailand and the United States hosted the
of the Asian Race Walking Championships in annual Cobra Gold military exercise, the biggest
Nomi, Japan. The Olympics qualification period activity of its type in the Asia-Pacific region with 29
for all other athletics events will start from May 1 nations taking part as participants or observers.
this year and will run till June 29, 2020. Apart from the 2 countries, 7 countries who
Andreescu beat Kerber to lift Indian Wells title participated in the exercise were India, Singapore,
Young Canadian Bianca Andreescu was Japan, China, Indonesia, Malaysia and South
proclaimed champion at the Indian Wells Masters, Korea.
after she overcame the German Angelique Kerber. AFINDEX-19 Begins
It is the first time ever that a player who received a The inaugural Africa-India Field Training
wildcard entry to the tournament leaves with the Exercise-2019 for India and African nations
title. called AFINDEX-19 started with a grand opening
Valtteri Bottas wins Australia GP for Mercedes ceremony at Aundh Military Station, Pune.
Contingents of the 17 African Nations came
together for the opening ceremony along with a

©LegalEdge Tutorials Page 11 of 76


Replication or other unauthorized use of this material is prohibited by the copyright laws of India
contingent of Maratha Light Infantry representing U.S. And South Korea End Military Exercises
India. ‘Foal Eagle 2019’ And ‘Key Resolve’
PINAKA Guided Weapons System “Key Resolve” will be replaced by “Dongmaeng”
which is also known as alliance in English, Foal
Eagle drills will be replaced by a smaller battalion-
sized exercise. This decision has been made after
there was a failure to reach an agreement on
denuclearisation during the summit in Hanoi,
Vietnam between President Donald Trump and
North Korean leader Kim Jong Un.

Defence Research and Development SCIENCE AND TECHNOLOGY


Organization (DRDO) successfully test fired IIT (BHU) scientist discovered tadpole-like jets
the PINAKA guided WEAPON rocket on Sun
system from Pokhran range in Rajasthan. The
weapon system is equipped with state-of-the-art
guidance kit comprising of an advanced navigation
and control system.
India Now 2nd Largest Arms Importer In The
World

Scientists led by IIT (BHU) researcher Abhishek


Srivastava have discovered tadpole-shaped jets
coming out of regions with intense magnetic fields
on the Sun. Formally called pseudo-shocks, the
“tadpoles” are made entirely of plasma, which
accounts for an estimated 99% of the observable
India is the second largest importer of weapons in universe. It could help understand why corona
the world, as per the annual report ‘Trends in (Sun’s upper atmosphere) is over 200 times hotter
International Arms Transfers-2018’released by the than its surface.
think tank Stockholm International Peace Evidence of water, particle plumes found on
Research Institute (SIPRI). Saudi Arabia is now the asteroid BennuNASA’s OSIRIS-Rex mission
world’s largest weapons importer. discovers evidence of water, particle plumes on
NASA astronauts to carry out first all-female asteroid Bennu among numerous other findings.
spacewalk Bennu is only slightly wider than the height of the
Empire State Building and it is expected to contain
unaltered material from the very beginning of our
solar system.
West Nile Virus
A seven-year-old in Kerala has been detected with
the West Nile Virus. The central government has
sent the team to the state and is monitoring the case
Just in time for International Women’s Day, NASA
closely. West Nile Virus is a viral infection which
announced that it will be conducting the first-ever,
typically spread by mosquitoes and results in
all-female spacewalk. Astronauts Christina Koch
neurological disease as well as death in people. The
and Anne McClain will exit the International Space
Virus is the member of the flavivirus genus and
Station on March 29, 35 years after the first woman
belongs to the Japanese encephalitis antigenic
performed a spacewalk. They’ll be guided from the
complex of the family Flaviviridae.
ground by flight director Mary Lawrence, and
flight controllers Jackie Kagey and Kristen Facciol.

©LegalEdge Tutorials Page 12 of 76


Replication or other unauthorized use of this material is prohibited by the copyright laws of India
US-North Korea Peace Talks
US North Korea Peace Talks were started long back in and in 2007, when the parties agreed on a series of steps
order to resolve the long-term conflict between North to implement that 2005 agreement.
Korea and US and South Korea. The talks were also aimed Those talks, however, broke down in 2009 following
at denuclearizing the Korean peninsula. Recently, a series disagreements over verification and an internationally
of summits were held between the leaders of these condemned North Korea rocket launch. Pyongyang has
nations, North Korea's Kim Jong-un, South Korea's Moon since stated that it would never return to the talks and is
Jae-in, and Donald Trump of the United States. In parallel no longer bound by their agreements. The other five
to this, a number of cultural exchanges also began. parties state that they remain committed to the talks, and
Since many years, the United States along with the have called for Pyongyang to recommit to its 2005
international community have tried to negotiate to put an denuclearization pledge.
end to nuclear and missile development by North Korea In January 2018, another diplomatic effort began when
and its export of ballistic missile technology. Those efforts North Korean leader Kim Jong Un declared the country's
have been replete with periods of crisis, stalemate, and nuclear arsenal "complete" and offered to discuss with
tentative progress towards denuclearization, and North Seoul North Korea's participation in the South Korean
Korea has long been a key challenge for the global nuclear Olympics. North Korea's delegation to the
nonproliferation regime. Olympics included Kim Jong Un's sister, who met with
The United States has tried to pursue a variety of policy South Korean President Moon Jae-in. That meeting led to
responses to the proliferation challenges posed by North a sustained inter-Korean dialogue, including a meeting
Korea, including military cooperation with U.S. allies in between Kim Jong Un and Moon Jae-in April 27 that
the region, wide-ranging sanctions, and non-proliferation produced a declaration referencing the shared goal of
mechanisms such as export controls. The United States denuclearization of the Korean peninsula.
also engaged in two major diplomatic initiatives to have During a high-level meeting with South Korean officials
North Korea abandon its nuclear weapons efforts in in Pyongyang in March, Kim Jong Un conveyed his
return for aid. interest in meeting with U.S. President Donald Trump.
In 1994, faced with North Korea’s announced intent to Trump accepted the offer and met with Kim Jong-un on
withdraw from the nuclear Nonproliferation Treaty June 12, 2018, in Singapore, in the first summit meeting
(NPT), which requires non-nuclear weapon states to between the leaders of the United States and North Korea.
forswear the development and acquisition of nuclear They signed a joint statement, agreeing to security
weapons, the United States and North Korea signed the guarantees for North Korea, new peaceful relations,
Agreed Framework. Under this agreement, Pyongyang reaffirmation of the denuclearization of the Korean
committed to freezing its illicit plutonium weapons Peninsula, recovery of soldiers' remains, and follow-up
program in exchange for aid. negotiations between high-level officials. Immediately
Following the collapse of this agreement in 2002, North following the summit, Trump announced that the US
Korea claimed that it had withdrawn from the NPT in would discontinue "provocative" joint military exercises
January 2003 and once again began operating its nuclear with South Korea, and he wishes to bring the U.S. forces
facilities. back home at some point, but he said that was not part of
The second major diplomatic effort were the Six-Party the Singapore agreement.
Talks initiated in August of 2003 which involved China, U.S. President Trump and North Korean leader Kim Jong
Japan, North Korea, Russia, South Korea, and the United Un meet in Hanoi, Vietnam for their second summit. The
States. In between periods of stalemate and crisis, those talks end without a signed agreement. It was stated after
talks arrived at critical breakthroughs in 2005, when the meeting that the two sides had made progress but that
North Korea pledged to abandon “all nuclear weapons North Korea had called for sanctions to be lifted "in their
and existing nuclear programs” and return to the NPT, entirety" in exchange for partial denuclearization which
the United States rejected
.

©LegalEdge Tutorials Page 13 of 76


Replication or other unauthorized use of this material is prohibited by the copyright laws of India
THE SCHEDULED TRIBES AND OTHER TRADITIONAL FOREST DWELLERS
(RECOGNITION OF FOREST RIGHTS) ACT, 2006
The recent Supreme Court decision to evict the Adivasis Recognition of Rights The Gram Sabha , initiates the
whose claim has been rejected her brought the The process to determine the nature and extent of individual
Scheduled tribes and other traditional Forest Dwellers( or 2 community rights that could be recognised, the prime
Recognition of forest Rights) Act in the limelight. example of this was when 12 Gram Sabhas in Niyamgiri,
Commonly referred as Forest Rights Act (FRA) has a long Odisha following the Supreme Court order to not mine
surviving history of roadblocks and achievements. Lord Bauxite in Niyamgiri Hills. The sub division or taluka
Dalhousie was the first to emphasis for the need of a screens and approves the resolution followed up at
definitive policy for forests, though the main reason was district level. Three government officials consisting of
economic interests like railways and ship building. Forest, Revenue and tribal welfare departments and three
Continuing with the policy of Britishers even post elected members of local body make up the screening
Independence initially the idea was focussed to conserve committee. Appeals can also be filed to this committee.
forests but it was majorly a state subject. In 1976, forests Rights Recognised under the Act There are three kinds of
were shifted to concurrent list from state list by 42nd rights recognised under the act:- 1. Land Rights:
amendment.Even though conservation efforts were made Ownership of the land upto a maximum of 4 hectares
with the National Forest Policy,1980 and other acts. The which is being farmed by the forest dwellers, no new land
enactment of FRA brought in major policy shift even can be granted. The ownership rights can only be passed
though the conservation of forests in the national interest through inheritance that is, it cannot be sold or
was still the primary policy objective, the emphasis transferred. 2. Right to use: The forest land can be used to
shifted to the bonafide requirements of the marginalised collect minor forest produce like tendu patta, herbs etc
individuals and communities dependent on forests. FRA excluding timber. It can be also be used as grazing land
is an act that grants legal recognition to the rights of or water bodies. A village assemble of all adult members
traditional forest dwelling communities to land and other of the village 2 3. Right to Protect and Conserve: Apart
resources, partly correcting the historical injustice with from the forest department, for the first right of
the schedules tribes and other traditional forest dwellers, conservation of local communities is recognised. Current
who are integral to the very survival of the forest Issue: The case of eviction filed in the supreme court has
ecosystem. Eligibility to be considered? Section 2 of the been filed environmentalists who believe the local
act, specifies the conditions which need to be fulfilled to communities are invading on the forest lands, on the
be recognised as “members of the community” who contrary the local population when involved can actually
reside in the the forest and depend on the forest area for amp up the conservation efforts. This act was
the 1 livelihood. More so, either the claimant should be a implemented to the correct the historical wrongs to the
member of the Scheduled Tribes recognised in the state community, by evicting the locals will go against the very
or the generation have resided in the forest or forest lands essence of the act
for three generation (75 years) prior to 13-12-2005.
.

©LegalEdge Tutorials Page 14 of 76


Replication or other unauthorized use of this material is prohibited by the copyright laws of India
Geneva Convention and Indo-Pak Relations
Geneva Conventions are a set go international treaties or scientific experiments which are justified for the
which consist of four conventions and three protocols prisoner and carried out in his interest. - Must be
agreed to between 1864 and 1949 to ensure that the parties protected against acts of intimidation and insult and
in war conduct themselves in a humane way with:- 1. public curiosity. - Released quickly after conflicts. - Not
“Non combatants” such as civilians and medical compelled to give an information except for the name,
personnel 2. “combatants” no longer engaged in fighting, age, rank and service number. Refusal to answer
example prisoner of war and sick personnel. The Geneva questions should not invite punishment. - Use of POWs
Conventions and the protocols form the basis of are hostages or human shield in strictly Prohibited.
International Humanitarian Law. India and Pakistan both Current Scenario In aerial engagement with Pakistan post
are signatories of Geneva Conventions. First Convention: Balikot incident, India lost one of its MiG-21 aircraft. The
Formed in 1864, it deals with improving the condition of fighter jet was shot down in POK and its pilot, Wing
the wounded and sick soldiers in the field during the war. Commander Abhinandan Varthaman was captured and
It contains 64 articles. Second Convention: It deals with taken into custody by the Pakistan army. India working
sick, wounded and ship wrecked members of armed through diplomatic channels demanded the safe and
forces at sea. Third Convention: It deals with the immediate return of the Indian pilot, in the mean time
treatment of prisoners of war(POW) during the conflict some videos were available on social media which
times and post that. It contains 143 articles. Fourth showed the officer being manhandled by a crowd, India
Convention: It deals with the treatment of civilians and strongly objected to the videos. Under the Geneva
their protection during the war time. It contains 159 Convention Article 118 POWs shall be released and
articles. Applicability of the convention The conventions repatriated without delay the the hostilities between the
apply at time of war and armed conflict governments two nations end and any unjustifiable delay in the
who have ratified the convention or made a local law to repatriation of PoW will be against the protocol. After
implement it. The International Committee of Red cross two days of continued pressure and diplomatic channels
usually makes sure that the conventions are followed. the IAF was returned. A similar situation happened
Rights of POWs - Treated humanely with respect for their during the Kargil War, when Flight lieutenant
persons and their honour. - Able to inform their next of KAmbamapati Nachiketa was captured and was in
kin and the international Committee of Red Cross of their Pakistan’s custody for more than a week. He was
Capture. - Any unlawful act or omission by the detaining returned to India post diplomatic efforts by the then
power causing death or seriously endangering the health government and the International Committee of red
of a prisoner of war in its custody prohibited. - Pows cross.
cannot be subjected to physical mutilation or to medical
.

©LegalEdge Tutorials Page 15 of 76


Replication or other unauthorized use of this material is prohibited by the copyright laws of India
20 LEGAL ONE LINERS
1. The Doctrine of Eclipse is in regard to Article 13 of the Constitution of India.
2. The principle of ‘collective responsibility’ has been incorporated in Article 75 of the Constitution of India.
3. Ordinarily, place of trial is where the offence has been committed.
4. Inquiry is conducted by a magistrate with a view to find out prima facie case.
5. Damages awarded for tortuous liabilities are unliquidated.
6. For an offence of cheating, intention to cheat should be present from the very beginning.
7. For defence of intoxication to escape criminal liability, the intoxication administered should be against the
defendant’s will or knowledge.
8. An agreement enforceable by law at the instance of one or more parties and not of others under section 2(i) of the
Indian Contract Act is called a voidable contract.
9. Defamation by spoken words or gestures is known as Slander.
10. The liability of master for the acts of his servant in Law of Torts is called vicarious liability.
11. Pawan Chamling has become India’s longest serving Chief Minister. He hails from Sikkim.
12. National Panchayati Raj Day is celebrated on 24th April every year.
13. Rajesh Bindal Committee has submitted its report on inter-country removal and retention of children.
14. Damnum Sine Injuria means the exercise of ordinary rights for a lawful purpose and in a lawful manner is no
wrong even if it causes damages.
15. Petition for restitution of conjugal rights shall be instituted in the District Court/Family Court.
16. Consent of minor is wholly immaterial in case of the offence of kidnapping.
17. Writ of Mandamus cannot be issued under Article 32 where fundamental right is infringed by a private body.
18. Transfer of Property Act, 1882 came into force on 1st July, 1882.
19. Mandal Commission for Backward Classes was set up in 1978
20. The concept of Fundamental Duties in Indian Constitution is derived from the Constitution of U.S.S.R.

Maxims and Foreign Phrases


1. Respondeat superior- Let the master be responsible
# The principal is vicariously liable for the acts of their agents, only if it is performed within the course of their
employment or scope of duty or only for those act for which he is authorized. This includes Master-Servant,
Employer- Employee Relationship. Independent Contractor is not liable in such cases.
2. Vexatious litigation- Law suit without any merits
# Vexatious litigations are such ligations where the legal Action which is taken is regardless of merits of the case and
has no grounds. The sole purpose is to vex or harass the adversary. It is a frivolous law suit or a false claim. This is
an abuse of judicial process, may attract sanctions against the offenders.
3. Unjust enrichment- Profit unjustly obtained by a wrongdoer
# Those enrichments in the circumstances that the law sees as unjust and at the expense of another is called unjust
enrichment. The aim of the law in such cases is restitution or compensation of restitution not possible so that someone
does not have to face unjust loss. Examples may be Wrongful gain and wrongful loss mentioned in Indian Penal
Code, Section 23.
4. Plea bargaining- Pleading guilty of crimes in order to seek concession in punishment
# In those offenses for which punishment is up to a period of 7 years pre-trial negotiations between the accused and
the prosecution is possible. It may lead to are duction of sentence of the accused or guilty. These are not applicable to
the crimes committed against women or children below 14 years.
5. Delegated Legislation- Laws made on behalf of the authority or parliament
# It is an exercise of legislative powers on the behalf of Parliament or any legislative authority of such nature. It is
done by the gency that is subordinate to the legislature, acquiring powers from the act of the legislature. It reduces
the burden of already overburdened Legislature.
6. Hearsay evidence- Testimony was given in the court by a person other than the one who perceived it
#Section 60 of the Indian Evidence Act lays down that oral evidence must in all cases be direct. “The word ‘direct’ to
this section is used as opposed to mediate or derivatives or what is technically called hearsay.” If the evidence is
based on received information from other person and that person does not say about it, such evidence would be
inadmissible being hearsay evidence i.e. would not be accepted in a court of law as evidence.
7. Corroborating Evidence- Supporting evidence

©LegalEdge Tutorials Page 16 of 76


Replication or other unauthorized use of this material is prohibited by the copyright laws of India
# It is evidence which supports and confirms a proposition which was already proved by initial evidence. It is
additional evidence from an independent source which confirms/substantiates the complainant’s testimony. It need
not be direct evidence.
8. Parole- The release of a convicted criminal before the completion of his sentence.
# Parole means temporary release out of a sentence in prison. Parolees may be returned to prison if they conditions
of their parole are violated. It may be given on medical or compassionate grounds i.e. death, marriage etc. In the US,
the concept of Parole is different than in India, and even after completion of a sentence in some offense is not given.
9. Arson- Criminal act of setting fire to someone’s property
# Provisions related to arson are given under section 435 and 436 of the Indian penal code. It refers to the voluntary
setting up of fire to moveable and immovable property. A Person who is guilty of arson shall be imprisoned up to 7
years and fine.
10. Blasphemy- Insulting or disrespecting one’s religion or god
# It refers to speaking offensive against god or religious feelings. India does not have recognized law on blasphemy,
however, Section 295-A of the Indian Penal Code can be construed to be it.

©LegalEdge Tutorials Page 17 of 76


Replication or other unauthorized use of this material is prohibited by the copyright laws of India
LE Prep Assist

Mentor View- General Knowledge

Hello Aspirants,
Most of the year has passed and we are now at the most crucial point in our preparation the exam
pressure can make you nervous and hence it is necessary that we follow a strict study regime and
keep things manageable. Some of the things to remember from the GK perspective are:-
(i) Do not look for anymore sources:- It is hard to resist the temptation to study from additional
sources during the last part of the preparation. But it is very important that you do not start
anything new now. Not only will you be not able to remember much, wasting all this precious time
will make you even more nervous furthering and worsening the problem. Concentrate, instead, on
revising what you have already invested time in.

(ii) Revise in the proper order :- While revising GK topics, start with the “CLASS NOTES” they are
very important and will have a “dijavu” effect on your memory. Next move on to the RSMS and
only then try question on that topic

(iii) Believe in yourself:- You have studied enough for the exam, even a lot perhaps with MOCKS,
MONTHLES, CLASS TESTS ON current affairs, GK today questions and various other sources, The
question number is between 10,000- 12000. It is very hard to make a test paper outside of these
questions so understand and believe that if after so much preparation, you aren’t able to do the
questions in the final paper, MOST ASPIRANTS WILL NOT BE ABLE TO AS WELL

All The Best!


Make us Proud

ABHISHEK DUBEY
HOD - GK Dept
LegalEdge Bhopal

©LegalEdge Tutorials Page 18 of 76


Replication or other unauthorized use of this material is prohibited by the copyright laws of India
LE Prep Assist

Prep Test– Legal Aptitude


Time: 45 min. No. of Questions: 50
Principles:
I. Any person who is of the age of majority according to the law to which he is subject, and who is of sound mind, may employ an
agent and will be the ‘principal’
II. As between the principal and third person any person may become an agent, but no person who is not of the age of majority and
of sound mind can become an agent, so as to be responsible to his principal according to the provisions in that behalf under the
Indian Contract Act
III. An agent has authority, in an emergency, to do all such acts for the purpose of protecting his principal from loss as would be done
by a person of ordinary prudence, in his own case under similar circumstances
1. Facts: Daulat is 20 years old. He has been enrolled in a mental institution and cannot comprehend situations.
Shramikhad been appointed as his agent to deal with the transactions of their family construction business.

(a) A principal agent relationship exists between Daulat and Shramik


(b) A principal agent relationship does not exist between Daulat and Shramik
(c) A principal sub-agent relationship exists between Daulat and Shramik
(d) None of the above

Principles:
I. Any person who is of the age of majority according to the law to which he is subject, and who is of sound mind, may employ an
agent and will be the ‘principal’
II. As between the principal and third person any person may become an agent, but no person who is not of the age of majority and
of sound mind can become an agent, so as to be responsible to his principal according to the provisions in that behalf under the
Indian Contract Act
III. An agent has authority, in an emergency, to do all such acts for the purpose of protecting his principal from loss as would be done
by a person of ordinary prudence, in his own case under similar circumstances
2. Facts: Zimmer consigned a set of luxury watches from Mumbai to Diu to Andrew for sale in Lucknow. When Andrew
got the watches in Diu, he found that the batteries of many of the watches needed to be replaced so that they could
be sold as proper pieces. Andrew got the batteries replaced.
Which of the following statement (s) is/are correct?

I. Zimmer and Andrew share a principal agent relationship


II. Andrew’s actions are justifiable under principle (iii)
III. Andrew’s actions are not justifiable under principle (ii)
IV. Andrew’s actions are justifiable under principle (i)

(a) I, III, IV (b) I, II (c) I, II, III (d) I, IV

Principles: When either the husband or the wife has, without reasonable excuse, withdrawn from the society of the other, the aggrieved
party may apply, by petition to the district court, for restitution of conjugal rights and the court, on being satisfied of the truth of the
statements made in such petition and that there is no legal ground why the application should not be granted, may decree restitution of
conjugal rights accordingly.
Where a question arises whether there has been reasonable excuse for withdrawal from the society, the burden of proving reasonable
excuse shall be on the person who has withdrawn from the society.
3. Facts: Seema and Seetaram had been married for a decade. But, on their eleventh wedding anniversary, Seema left
Seetaram when he beat her up, suspecting her of having an affair and threatened to kill her. Seetaram filed for
restitution of conjugal rights.

(a) Seema withdrew from the society of her husband without reasonable excuse
(b) The court will grant a decree for restitution of conjugal rights to Seema
(c) The court will grant a decree for restitution of conjugal rights in favour of Seetaram
(d) The fact that Seetaram threatened to kill Seema and bashed her will help determine the status of decree of
restitution of conjugal rights

©LegalEdge Tutorials Page 19 of 76


Replication or other unauthorized use of this material is prohibited by the copyright laws of India
LE Prep Assist
Principles: When either the husband or the wife has, without reasonable excuse, withdrawn from the society of the other, the aggrieved
party may apply, by petition to the district court, for restitution of conjugal rights and the court, on being satisfied of the truth of the
statements made in such petition and that there is no legal ground why the application should not be granted, may decree restitution of
conjugal rights accordingly.
Where a question arises whether there has been reasonable excuse for withdrawal from the society, the burden of proving reasonable
excuse shall be on the person who has withdrawn from the society.
Facts: Seema and Seetaram had been married for a decade. But, on their eleventh wedding anniversary, Seema left Seetaram
when he beat her up, suspecting her of having an affair and threatened to kill her. Seetaram filed for restitution of conjugal
rights.

4. On whom will the burden of proof of reasonable excuse for withdrawal from society lie?
(a) Seetaram (b) Seema (c) The court (d) All of the above

Principles: It may be necessary that documents, things and other materials relevant for investigation, inquiry or trial should be made
available for conducting proceedings before the Court. While a person in possession of the aforementioned things should produce them
when required, if he/she fails to produce it, the competent court can pass necessary orders and issue directions to the police for search
and seizure of such documents and things.
5. Facts: Bhiku and Miku are two brothers who were having a dispute regarding the will of their deceased father. A
case was filed by Bhiku in court to establish the credibility of the will in question and he blamed Miku for conspiring
against their father. Bhiku said that his father had already made him the owner of their two houses at Brixton Road
last year and the will in question contradicted that.
I. The will in question will have to be produced before the Court
II. A document evidencing the said deal between Bhiku and his father about the two houses will have to be produced
before the Court
III. Documents pertaining to their father’s property may be produced before the Court
IV. It is upto the parties to admit the documents they desire in the course of the proceedings

(a) I ,II, IV (b) I, II, III (c) I, IV (d) IV

Principles: It may be necessary that documents, things and other materials relevant for investigation, inquiry or trial should be made
available for conducting proceedings before the Court. While a person in possession of the aforementioned things should produce them
when required, if he/she fails to produce it, the competent court can pass necessary orders and issue directions to the police for search
and seizure of such documents and things.
6. Facts: Assuming in the above facts, that Miku had the only copy of the will in question and refused to bring it to
Court. Bhiku claimed he had evidence to prove that his father had left the two houses at Brixton Road to him while
his father’s assistant, Brijlal claimed that he had left the houses to him. Bhiku and Brijlal did not produce any
documents to support their claims for two Court hearings.

Which of the following statements is incorrect?


(a) The Court can order for search and seizure of the will from Miku
(b) The Court can order for search and seizure of documents on the property from Bhiku
(c) The Court cannot order for search and seizure of documents from Brijlal as he was not related to Bhiku, Miku
and their father
(d) The Court can give appropriate directions to the police to procure the documents necessary for proceedings in
the case

Principle: Coercion is the committing, or threatening to commit, any act forbidden by the Indian Penal Code or the unlawful detaining,
or threatening to detain, any property, to the prejudice of any person whatsoever, with the intention of causing any person to enter into
an agreement.
It is immaterial whether the Indian Penal Code is or is not in force in the place where the coercion is employed.
7. Facts: Dhawal met Mohan in London and asked him to sell his luxury watch to him for Rs.10,000 when they get back
to India. When Mohan refused to honour his part of the deal, Dhawal threatened him and criminally intimidated him
as under the Indian Penal Code. When they returned to India, Mohan refused to sell the watch to him.

(a) Dhawal’s actions amount to coercion

©LegalEdge Tutorials Page 20 of 76


Replication or other unauthorized use of this material is prohibited by the copyright laws of India
LE Prep Assist
(b) Dhawal’s actions do not amount to coercion
(c) Dhawal will be guilty of coercion for he offered a very low price for the watch
(d) None of the above

Principle: Frivolous litigation is the practice of starting or carrying on law suits that, due to their lack of legal merit, have little to no
chance of being won. The term does not include cases that may be lost due to other matters not related to legal merit. The fact that a
claim is lost does not imply that it was frivolous.
8. Facts: Srishti was working in a textile company. She was denied a long-overdue promotion and she attributed it to
the fact that she was getting married and the boss felt she would not be able to handle the workload. She filed a
lawsuit in Court. However, the claim was lost due to lack of proper evidence.

(a) The lawsuit filed by Srishti was frivolous because there was no evidence to support her claim
(b) The lawsuit filed by Srishti was not frivolous
(c) The lawsuit filed by Srishti was frivolous because she lost the case
(d) It cannot be determined whether the lawsuit was frivolous

Principles: Malicious prosecution is the malicious institution of unsuccessful criminal or bankruptcy or liquidation proceedings against
another without reasonable or probable cause. The main ingredients for an action for malicious prosecution are:
I. That he was prosecuted by the defendant.
II. That the proceeding complained was terminated in favour of the present plaintiff
III. That the prosecution was instituted against without any just or reasonable cause.
IV. That the prosecution was instituted with a malicious intention, that is, not with the mere intention of getting the law into effect,
but with an intention, which was wrongful in fact.
V. That he suffered damage to his reputation or to the safety of person, or to security of his property.
9. Facts: Tenali used to work as a butler at Dharmodas’s house. Dharmodas’s relative Kaalia came to visit him for a few
weeks. He never liked Tenali much. One day he lost his wallet in the market. Seeing an opportunity to trap Tenali,
he went and complained to Dharmodas, saying that Tenali stole his wallet. Tenali was arrested on suspicion but was
finally acuitted in the case. Dharmodas refused to employ Tenali again. Word about the case had spread and no one
was willing to employ Tenali due to the stigma of his having been arrested.

(a) There was malicious prosecution in this case


(b) There was no malicious prosecution in this case
(c) Tenali did not suffer reasonable harm as a result of the proceedings
(d) It cannot be determined

Principles: Malicious prosecution is the malicious institution of unsuccessful criminal or bankruptcy or liquidation proceedings against
another without reasonable or probable cause. The main ingredients for an action for malicious prosecution are:
I. That he was prosecuted by the defendant.
II. That the proceeding complained was terminated in favour of the present plaintiff
III. That the prosecution was instituted against without any just or reasonable cause.
IV. That the prosecution was instituted with a malicious intention, that is, not with the mere intention of getting the law into effect,
but with an intention, which was wrongful in fact.
V. That he suffered damage to his reputation or to the safety of person, or to security of his property.
10. Facts: Tilakratne was Dilshan’s accountant. Recently, Dilshan discovered that there were major discrepancies in the
accounts and that Tilakratne had been careless in making and checking them. He filed a case against Tilakratne but
the Court acquitted Tilakratne for lack of evidence. Tilakratne joined a company as a chartered accountant and
suffered no damage whatsoever.

(a) It is a case of malicious prosecution


(b) It is not a case of malicious prosecution
(c) It may have been a case of malicious prosecution had Tilakratne not got a job elsewhere
(d) It cannot be determined

©LegalEdge Tutorials Page 21 of 76


Replication or other unauthorized use of this material is prohibited by the copyright laws of India
LE Prep Assist
Principle: The exposure of goods by a shopkeeper does not amount to an offer to sell. When a customer picks a good, it is an offer by the
customer to buy, and sale is not effected until the buyer’s offer price is accepted by the shopkeeper.
11. Facts: Tina walked into a gift shop and started trying out bracelets which were displayed in the jewellery shop. When
she finally chose a greeting card to buy at the counter, the shopkeeper told her she had to buy the bracelets she had
been trying else she should not have tried them if she did not intend to buy them.

(a) Tina will have to buy the bracelets as sale had been effected
(b) Tina will have to buy the greeting card and the bracelets as sale had been effected
(c) Tina could buy just the greeting card if she wants, as sale had not been effected yet
(d) Tina will have to necessarily buy something from the gift shop

Principle: A person is said to cause an effect “voluntarily” when he causes it by means whereby he intended to cause it, or when he was
employing those means, he knew or had reason to believe to be likely to cause it.
Facts: Balu entered his arch enemy Kalu’s house to confront him about a business deal gone bad. He brought a gun along
and knew it could inflict serious damage. Later, the two got into a scuffle and Balu fired the gun in Kalu’s direction to scare
him. The bullet hit Kalu in the head instead, and he died.

12. Can Balu’s act be said to have been done “voluntarily”?


(a) Yes (b) No
(c) Yes, Balu and Kalu had an old enmity (d) Cannot be determined

Principles:
I. Subject to provisions in the Sale of Goods Act, when the buyer of the goods becomes insolvent, the unpaid seller who has parted
with the possession of the goods has the right of stopping them in transit, that is to say, he may resume possession of the goods as
long as they are in the course of transit, and may retain them until payment or tender of the price
II. If the buyer or his agent in that behalf obtains delivery of the goods before their arrival at the appointed destination, the transit is
at an end
III. The seller of goods is deemed to be an “unpaid seller” when the whole of the price has not been paid or tendered
Facts: Zohrab contracts to buy one thousand gallons of oil from Tehreek. After paying some token amount as advance,
Zohrab tells Tehreek that he will pay the remaining amount once the oil barrels reach Egypt where he will receive them.
Tehreek loads the consignment on the ship and when the ship is in the middle of the journey towards Egypt, Zohrab calls
to tell him that he should deliver the goods to Namibia instead. Tehreek soon finds out that Zohrab had become insolvent
and all his businesses had been shut down by the government. He instructs his ship with the consignment to return and
tells Zohrab that he will not send the consignment until he is paid in full.

13. If Zohrab: Buyer:: Tehreek:___________ (Choose the alternative that best fits)
(a) Seller (b) Unpaid seller (c) Consignee (d) None of the above

Principles:
I. Subject to provisions in the Sale of Goods Act, when the buyer of the goods becomes insolvent, the unpaid seller who has parted
with the possession of the goods has the right of stopping them in transit, that is to say, he may resume possession of the goods as
long as they are in the course of transit, and may retain them until payment or tender of the price
II. If the buyer or his agent in that behalf obtains delivery of the goods before their arrival at the appointed destination, the transit is
at an end
III. The seller of goods is deemed to be an “unpaid seller” when the whole of the price has not been paid or tendered
Facts: Zohrab contracts to buy one thousand gallons of oil from Tehreek. After paying some token amount as advance,
Zohrab tells Tehreek that he will pay the remaining amount once the oil barrels reach Egypt where he will receive them.
Tehreek loads the consignment on the ship and when the ship is in the middle of the journey towards Egypt, Zohrab calls
to tell him that he should deliver the goods to Namibia instead. Tehreek soon finds out that Zohrab had become insolvent
and all his businesses had been shut down by the government. He instructs his ship with the consignment to return and
tells Zohrab that he will not send the consignment until he is paid in full.

14. Can the goods be said to be in transit when Zohrab asked for a change of destination to Namibia?
(a) Yes (b) No
(c) It depends on Tehreek’s actions (d) Cannot be determined

©LegalEdge Tutorials Page 22 of 76


Replication or other unauthorized use of this material is prohibited by the copyright laws of India
LE Prep Assist
Principles:
I. Subject to provisions in the Sale of Goods Act, when the buyer of the goods becomes insolvent, the unpaid seller who has parted
with the possession of the goods has the right of stopping them in transit, that is to say, he may resume possession of the goods as
long as they are in the course of transit, and may retain them until payment or tender of the price
II. If the buyer or his agent in that behalf obtains delivery of the goods before their arrival at the appointed destination, the transit is
at an end
III. The seller of goods is deemed to be an “unpaid seller” when the whole of the price has not been paid or tendered
Facts: Zohrab contracts to buy one thousand gallons of oil from Tehreek. After paying some token amount as advance,
Zohrab tells Tehreek that he will pay the remaining amount once the oil barrels reach Egypt where he will receive them.
Tehreek loads the consignment on the ship and when the ship is in the middle of the journey towards Egypt, Zohrab calls
to tell him that he should deliver the goods to Namibia instead. Tehreek soon finds out that Zohrab had become insolvent
and all his businesses had been shut down by the government. He instructs his ship with the consignment to return and
tells Zohrab that he will not send the consignment until he is paid in full.

15. Is Tehreek’s reaction towards Zohrab’s change of circumstances justified?


(a) It depends on Zohrab
(b) It depends on trade practice followed in Egypt and Namibia
(c) Yes, as it is mandatory that Tehreek do so
(d) None of the above

Principles:
I. The Protection of Women from Domestic Violence Act, 2005 covers women who are or have been in a relationship with the abuser
where both parties have lived in a ‘shared household’ and are related by consanguinity, marriage or through a relationship in the
nature of marriage or adoption. Even those women who are sisters, widows, mothers, single women, or living with abuser are
entitled to legal protection under the Act
II. The Act does not enable any female relative of the husband or male partner to file a complaint against the wife or the female
partner
III. The divorced wife would come within the category of aggrieved person and is entitled to live with shared household
IV. “shared household” means a household where the person aggrieved lives or at any stage has lived in a domestic relationship either
singly or along with the respondent and includes such a household whether owned or tenanted either jointly by the aggrieved
person and the respondent, or owned or tenanted by either of them in respect of which either the aggrieved person or the respondent
or both jointly or singly have any right, title, interest or equity and includes such a household which may belong to the joint
family of which the respondent is a member, irrespective of whether the respondent or the aggrieved person has any right, title or
interest in the shared household
Facts: Kirti married Soumitr and started residing with him and his mother, Jayshri at her husband’s ancestral home. Soon
after marriage Soumitr started drinking regularly and beat Kirti up every night and abused her. Kirti left the house and
filed for divorce. Once she left Soumitr, his mother started blaming Kirti for destroying her son’s life and marriage, and
causing Jayshri immense mental trauma at such age. She wanted to sue Kirti for domestic violence. Meanwhile, Kirti got
the divorce from Soumitr. Soumitr started torturing his elderly mother for money.

16. Would Kirti be covered under the ambit of the PoWDV Act, 2005?
(a) Yes (b) No
(c) Cannot be determined (d) It is not relevant to the case

Principles:
I. The Protection of Women from Domestic Violence Act, 2005 covers women who are or have been in a relationship with the abuser
where both parties have lived in a ‘shared household’ and are related by consanguinity, marriage or through a relationship in the
nature of marriage or adoption. Even those women who are sisters, widows, mothers, single women, or living with abuser are
entitled to legal protection under the Act
II. The Act does not enable any female relative of the husband or male partner to file a complaint against the wife or the female
partner
III. The divorced wife would come within the category of aggrieved person and is entitled to live with shared household
IV. “shared household” means a household where the person aggrieved lives or at any stage has lived in a domestic relationship either
singly or along with the respondent and includes such a household whether owned or tenanted either jointly by the aggrieved
person and the respondent, or owned or tenanted by either of them in respect of which either the aggrieved person or the respondent
or both jointly or singly have any right, title, interest or equity and includes such a household which may belong to the joint
©LegalEdge Tutorials Page 23 of 76
Replication or other unauthorized use of this material is prohibited by the copyright laws of India
LE Prep Assist
family of which the respondent is a member, irrespective of whether the respondent or the aggrieved person has any right, title or
interest in the shared household
Facts: Kirti married Soumitr and started residing with him and his mother, Jayshri at her husband’s ancestral home. Soon
after marriage Soumitr started drinking regularly and beat Kirti up every night and abused her. Kirti left the house and
filed for divorce. Once she left Soumitr, his mother started blaming Kirti for destroying her son’s life and marriage, and
causing Jayshri immense mental trauma at such age. She wanted to sue Kirti for domestic violence. Meanwhile, Kirti got
the divorce from Soumitr. Soumitr started torturing his elderly mother for money.

17. Who of the following can Jayshri sue under the Act?
(a) Kirti (b) Soumitr (c) Both (a) and (b) (d) None of the above

Principles:
I. The Protection of Women from Domestic Violence Act, 2005 covers women who are or have been in a relationship with the abuser
where both parties have lived in a ‘shared household’ and are related by consanguinity, marriage or through a relationship in the
nature of marriage or adoption. Even those women who are sisters, widows, mothers, single women, or living with abuser are
entitled to legal protection under the Act
II. The Act does not enable any female relative of the husband or male partner to file a complaint against the wife or the female
partner
III. The divorced wife would come within the category of aggrieved person and is entitled to live with shared household
IV. “shared household” means a household where the person aggrieved lives or at any stage has lived in a domestic relationship either
singly or along with the respondent and includes such a household whether owned or tenanted either jointly by the aggrieved
person and the respondent, or owned or tenanted by either of them in respect of which either the aggrieved person or the respondent
or both jointly or singly have any right, title, interest or equity and includes such a household which may belong to the joint
family of which the respondent is a member, irrespective of whether the respondent or the aggrieved person has any right, title or
interest in the shared household
Facts: Kirti married Soumitr and started residing with him and his mother, Jayshri at her husband’s ancestral home. Soon
after marriage Soumitr started drinking regularly and beat Kirti up every night and abused her. Kirti left the house and
filed for divorce. Once she left Soumitr, his mother started blaming Kirti for destroying her son’s life and marriage, and
causing Jayshri immense mental trauma at such age. She wanted to sue Kirti for domestic violence. Meanwhile, Kirti got
the divorce from Soumitr. Soumitr started torturing his elderly mother for money.

18. Considering Kirti and Soumitr lived together in his ancestral home, would it be considered a ‘shared household’?
(a) Yes
(b) No
(c) It depends on whether the house is in Soumitr’s name
(d) None of the above

Principles:
I. The Protection of Women from Domestic Violence Act, 2005 covers women who are or have been in a relationship with the abuser
where both parties have lived in a ‘shared household’ and are related by consanguinity, marriage or through a relationship in the
nature of marriage or adoption. Even those women who are sisters, widows, mothers, single women, or living with abuser are
entitled to legal protection under the Act
II. The Act does not enable any female relative of the husband or male partner to file a complaint against the wife or the female
partner
III. The divorced wife would come within the category of aggrieved person and is entitled to live with shared household
IV. “shared household” means a household where the person aggrieved lives or at any stage has lived in a domestic relationship either
singly or along with the respondent and includes such a household whether owned or tenanted either jointly by the aggrieved
person and the respondent, or owned or tenanted by either of them in respect of which either the aggrieved person or the respondent
or both jointly or singly have any right, title, interest or equity and includes such a household which may belong to the joint
family of which the respondent is a member, irrespective of whether the respondent or the aggrieved person has any right, title or
interest in the shared household
Facts: Kirti married Soumitr and started residing with him and his mother, Jayshri at her husband’s ancestral home. Soon
after marriage Soumitr started drinking regularly and beat Kirti up every night and abused her. Kirti left the house and
filed for divorce. Once she left Soumitr, his mother started blaming Kirti for destroying her son’s life and marriage, and

©LegalEdge Tutorials Page 24 of 76


Replication or other unauthorized use of this material is prohibited by the copyright laws of India
LE Prep Assist
causing Jayshri immense mental trauma at such age. She wanted to sue Kirti for domestic violence. Meanwhile, Kirti got
the divorce from Soumitr. Soumitr started torturing his elderly mother for money.

19. Which of the following statements is correct?


(a) Kirti must severed all ties from Soumitr
(b) Kirti must avoid meeting Soumitr’s family members
(c) Kirti may continue to live at Soumitr’s ancestral home
(d) None of the above

Principle: Where both parties to an agreement are under a mistake of fact essential to the agreement, the agreement is void.
An erroneous opinion as to the value of the thing which forms the subject-matter of the agreement, is not to be deemed a mistake as to a
matter of fact.
20. Facts: Reena entered into an agreement with Seema to sell her gold necklace for Rs.50,000. On reaching home, Reena
found out that the necklace had been stolen. She told Seema about the same. The agreement is:

(a) Void (b) Voidable (c) Enforceable (d) None of the above

Principle: Where both parties to an agreement are under a mistake of fact essential to the agreement, the agreement is void.
An erroneous opinion as to the value of the thing which forms the subject-matter of the agreement, is not to be deemed a mistake as to a
matter of fact.
21. Facts: Rahul agrees to sell his motorcycle to Roshan for Rs. 1.5 lakhs. Meanwhile, Roshan finds out the market price
of the same motorcycle to be not more than Rs.90,000. He says the agreement is void as he was not aware of the worth
of the motorcycle earlier. Roshan is:

(a) Right (b) Wrong (c) Confused (d) None of the above

Principles:
I. False imprisonment is the complete deprivation of his liberty for any time, however short, without lawful cause. There need not
be any actual imprisonment in the ordinary sense
II. Restraint must be complete
III. There must be no reasonable condition imposed by occupiers of premises
IV. There must be no reasonable and honest belief which would justify the confinement
Facts: Madhu was working for a local NGO. She was asked to stay back one day after 4 pm , even though her working
hours were from 7.30 am to 3pm. The head coordinator, Ruchi told her that she was suspected of having bungled funds
and thus, could not leave the premises till further inquiry had been completed. But she would be provided with facilities
in the office till then. Further, she could leave if an emergency arose.

22. Would the above situation qualify as false imprisonment?


(a) Yes
(b) No
(c) It depends on whether Madhu wants to press charges
(d) It cannot be determined

Principles:
I. False imprisonment is the complete deprivation of his liberty for any time, however short, without lawful cause. There need not
be any actual imprisonment in the ordinary sense
II. Restraint must be complete
III. There must be no reasonable condition imposed by occupiers of premises
IV. There must be no reasonable and honest belief which would justify the confinement
Facts: Shyamu used to work for the Talwars. One day he went to borrow some sugar from the neighbours, the Bhatias, who
bolted him in a room. After fifteen minutes, Mr.Talwar came to check on Shyamu at the Bhatias. The Bhatias revealed that
they had locked Shyamu to get back at the Talwars for some old grudge.

23. Which of the following statements is correct?


I. There was no false imprisonment of Shyamu as he was released after a short period of time

©LegalEdge Tutorials Page 25 of 76


Replication or other unauthorized use of this material is prohibited by the copyright laws of India
LE Prep Assist
II. There was false imprisonment of Shyamu as he was totally restrained, without reasonable conditions
III. There was no reasonable and honest belief to justify the confinement
IV. None of the above

(a) I (b) II (c) II, III (d) IV

Principle: Any loss incurred by the plaintiff due to the negligence of independent contractors, where the contractors have been employed
by the plaintiff with proper and reasonable care and caution, unless just a monetary loss, must be compensated for. The contractors
hired must possess proper skill to perform the service and it is the responsibility of the employer to judge their qualification.
24. Facts: Tragedy struck the Merton household when Matrin’s widowed mother passed away in a car accident. Their
house in the Hamptons is left to Martin and so Martin and his wife Ann are forced to uproot their lives from New York
city and move to the Hamptons. The ancestral property holds sentimental value to Martin as he cherishes his childhood
memories in that place. But soon it is brought to their knowledge that his mother had mortgaged the house to pay his
dead father’s medical bills. To release the house, the Mertons are forced to sell their New York apartment, which they
had constructed by hiring the best builders in the city. They expect the price for the apartment to be around $10,000
and are therefore, terribly shocked when the real estate agent’s analysis promises a price of just $7000. The reason for
the reduction is the negligence of the builders in making the floor boards weak and susceptible to damage. In despair,
they claim damages of $3000 from the builders, which they need to save Martin’s childhood home.

(a) The claim for damages is legitimate since the loss is cause by the negligence of the builders and some amount of
damages must be awarded but not $3000.
(b) Damages cannot be awarded because there is no redeemable loss.
(c) Compensation worth $3000, which is the loss that the Mertons have to incur due to the negligence of the builders,
should be provided.
(d) The Mertons should sue for fraud.

Principle:
I. A person is responsible for any damage, which is a subsequent result of an action of the defendant, not too remote and reasonably
foreseeable, having a direct link or chain of causation.
II. Chain of causation is broken when a third party intervenes between the action of the defendant and the damage to the plaintiff or
the damage can be traced to some other negligent act.
25. Facts: Jack Reacher was driving in his rusty Mustang on an isolated road in the forgotten town of Texas, Bodeen. He
had picked up the car from an old, unused parking lot. No one travels these roads at this time of the night. The car
started making unusual noises in the middle of the road just before stopping Reacher figured there was some problem
in the engine. He considered the option of abandoning the car but finding a ride at this time of the night would be a
tedious task. He remembered the mechanic shop he had passed by a few miles ago and decided to get help from
there, leaving the car on the road.
Ronald was a lorry driver passing by on that fateful night. He was slightly high from the hash he had just abused
before taking off but had full confidence in his driving skills. After all he had been doing this job for quite some time.
What he did not realize in his blissful high was that he was driving way above speed limit quite recklessly and
therefore, it was no surprise when he collided with the car left by Reacher. Ronald’s lorry overturned which led to
further collision of two other cars. Whose liability is it?

(a) Reacher was negligent in leaving the car on the road, resulting in the collisions and therefore it is his liability.
(b) Even though Ronald contributed to the accidents, the liability will be of Reacher since he started the chain of
causation.
(c) Reacher could not have foreseen the negligent act of Ronald. Ronald’s negligent driving was the main cause of
the accidents and the damages can be traced back to his negligence, therefore he is liable for the accidents.
(d) Ronald and Reacher both will be liable.

Principle:
I. Strict liability: A person, who for his own purpose, brings or stores or keeps, onto his land an inherently dangerous substance,
which on escaping has the potential to do harm to a third person, must keep it at his peril and is answerable for every direct
consequence of the escape of such a substance.
II. The above principle is nullified when the substance in question is being used in discharge of public duty.

©LegalEdge Tutorials Page 26 of 76


Replication or other unauthorized use of this material is prohibited by the copyright laws of India
LE Prep Assist
26. Facts: Daenerys is employed at Khal Drogo and Co. which is famous for production of explosive shells. The factory
is spread over an area of a thousand acres. The company enters into a contract with the government of Tazakia, to
produce explosive shells, which are to be used in the war against the state’s enemies and for the protection of the
citizens. Daenerys is promoted to the post of the manager of the testing division soon after joining the company.

Tazakia is in urgent need of novel inventions as its enemies are gaining strength and the chances of a war have been
exponentially increased. The Government delegated this responsibility to the company. The scientists invented ‘The
Dragon’, an explosive shell and the promised area of explosion was a good 50 acres. ‘The Dragon’ was then sent for
testing to the testing department and Daenerys was personally handed the responsibility of overlooking the
operation. While testing though, it was discovered that ‘The Dragon’ is unstable at room temperature and the test
shows unexpected and unforeseeable results. Daenerys is gravely injured during the testing. Daenerys files a suit in
court holding the company strictly liable for her injuries.

(a) The company will be strictly liable in the light of principle 1.


(b) The operation was undertaken for the protection of the public and therefore in the light of principle 2, the liability
of the company is nullified.
(c) Since the accident was unexpected and unforeseeable, the company should not be held liable for the negligence.
(d) The company will not be strictly liable in the light of principle 1 as all the conditions are not met.

Principle: The person is bound by the duty of possessing the skill that he professes to possess. When a person representing to be
possessing a particular skill, performs an action professing that skill, he is binding himself to the duty of performing that action with
proper and due care that an ordinary man possessing the same skill would consider reasonable to undertake while performing the action.
27. Facts: Dr. Panparag is a renowned oncologist, who specializes in practice of a new treatment for breast cancer, which
promises to eradicate the cancerous cells altogether. Dr. Panparag firmly stands by the promise of this mode of
treatment and has a 100% success rate. Ms. Rajnigandha is a patient who suffers from first stage breast cancer. She visits
Dr. Panparag who advises the treatment. After five months of getting the treatment, there is regrowth of cancerous
cells. Decide:

(a) Dr. Panparag professed the success of the treatment, which failed, and is
Therefore liable.
(b) Dr. Panparag cannot be held responsible for the reappearance of the cells. He professed the treatment, which
eradicated the cells once.
(c) Dr. Panparag did not perform the treatment withdue care and is therefore liable.
(d) Dr. Panparag is not liable for the failure of the treatment.

Principle: ‘Volenti non fit injuria’ is a defence in tort that means where a person engages in an event accepting and aware of the risks
inherent in that event, then they cannot later complain of, or seek compensation for an injury suffered during the event.
28. Facts: The claimant was a photographer at a horse show. He was situated within the ring of the horse show and not
behind the barriers where the spectators were housed. He was on a bench with Miss Smallwood who was a director
of the company which employed the Claimant. He had been taking little interest in the proceedings and was not
experienced in regard to horses. During the competition, one of the horses owned by the defendant, came galloping
at great speed towards the bench where they were sitting. The Claimant took fright at the approach of the galloping
horse and attempted unsuccessfully to pull Miss Smallwood off the bench. He stepped and fell back into the course
of the horse which passed three or few feet behind the bench, and was knocked down. The Claimant brought an
action in negligence arguing the rider had lost control of the horse and was going too fast. The defendant raised the
defence of volenti non fit injuria. Will the defendant succeed?

(a) The defendant will not succeed because the rider of the horse owned by him was negligent while riding.
(b) The defendant will not succeed because the claimant was trying to save Miss Smallwood.
(c) The defendant will succeed because by sitting in the wrong area he had voluntarily accepted the risk of getting
injured.
(d) The defendant will succeed because the rider had not lose control of the horse.

©LegalEdge Tutorials Page 27 of 76


Replication or other unauthorized use of this material is prohibited by the copyright laws of India
LE Prep Assist
Principles: Mens rea (guilty mind) and actus reus (guilty act) result in a crime only when they occur together. Else, no crime is
committed.
29. Fact: Reema wanted to injure her neighbour Sheela as she held a grudge against Sheela. She went to Sheela’s house,
armed with a metal rod, but no one was at home. Later, Reema’s car accidentally bumped into Sheela’s car from
behind, and Sheela suffered a head injury as she wasn’t wearing her seatbelt. Has Reema committed a crime?

(a) Yes, because Reema had a guilty mind to injure Sheela.


(b) No, because the injury was Sheela’s fault since she didn’t wear a helmet.
(c) Yes, because perpetrators of road accidents must be severely punished.
(d) No, because the guilty mind of Reema was not the reason behind Sheela’s injury.

Principles: Mens rea (guilty mind) and actus reus (guilty act) result in a crime only when they occur together. Else, no crime is
committed.
30. Fact: Jagat had a plan to kidnap Siya from her school on Monday. She did not, however, attend school that day, and
thus he postponed his plan by 2 days, successfully kidnapping her on Wednesday. Has Jagat committed a crime?

(a) No, because the mens rea was to kidnap her on Monday while the actual kidnapping took place 2 days later.
(b) Yes, because even though the kidnapping was postponed, the mens rea was not extinguished and was present
on Wednesday as well.
(c) Yes, because even though act and guilty mind don’t coincide, the severity of his act makes Jagat guilty.
(d) No, because Jagat was unsuccessful in performing the crime on Monday.

Principle:
I. Duress is a defence. A person is exempted from liability if he acted under any kind of compulsion provided he did not put himself
in that situation.
II. However, this defence does not extend to murder and offences against the State.
III. The threat must continue throughout the offence.
31. Fact: G’s mother asked her to get some groceries and asked her to be careful on the way. She was apprehended by a
robber who asked her to steal something from the grocery store or he would kill her. She tried her best but got caught,
and was charged with theft. Can she claim duress as a defence?

(a) No, because she herself was the cause of her situation.
(b) No, because she could have taken a safer way and invited trouble.
(c) Yes, because she was under a threat of losing her life.
(d) Yes, as she was acting in her best interest which is more important than private property.

Principle: Conversion to another religion for the purpose of marriage is not lawful, and the second marriage is void from the
beginning.
32. Facts: Ram Singh was born Hindu, but was a great admirer of the Islamic religion since a very young age. Soon after
his marriage to a Hindu bride, he decided to formally convert into Islamic religion. On his twentieth wedding
anniversary, he declared that he wished to marry a Muslim woman. A month later, he got married again, while his
first wife was still alive. Is his second marriage legal?

(a) Ram Singh’s first wife was still a Hindu, and therefore, his second marriage cannot be lawful.
(b) The second marriage is void from the beginning as Ram Singh converted to another religion for the purpose of
marriage.
(c) The second marriage is legal as it is clear that Ram Singh’s decision to convert to Islamic religion was not for the
purpose of marriage, but out of a strong affinity towards the Islamic religious culture.
(d) It is not unlawful, because many people, including film actors have converted to Islamic religion to have more
than one marriage at an existing time.

Principle: Nothing is an offence which is committed by a person, who at the time of doing the act, is incapable of knowing the nature
of the act, due to being intoxicated against his will or without his knowledge.
33. Facts: On New Year’s Eve, Reba decided to go to a bar to spend some time partying with her friends. She decided to
not consume alcohol. However, she ate a few rum based chocolates at the party, without being aware that they

©LegalEdge Tutorials Page 28 of 76


Replication or other unauthorized use of this material is prohibited by the copyright laws of India
LE Prep Assist
contained rum. While she was driving back, she accidentally hit a person on the road, causing him minor injuries.
Will she be exempted from the offence?

(a) No, as she was obviously not intoxicated due to rum based chocolates, and committed the act wilfully.
(b) No, as rum based chocolates cannot be said to contain enough alcohol that would cause intoxication.
(c) No, as she was incapable of knowing the nature of the act committed by her.
(d) None of the above.

Principle: Culpable homicide occurs when a person causes death by doing an act with the intention of causing death.
Intention is the person's awareness of the fact that his or her conduct would lead to death of another person.
34. Facts: Akshay was hunting for deer in one of the dense forests of Nagaland. After unsuccessfully chasing deer for
two days, he was tired and sleep deprived. On the third day, he saw a slight movement among the tall bushes and
was certain that it was a tiger. Scared that he was in the territory of a tiger who would attack him any moment, he
aimed his gun at the bush. Once he was sure that the tiger was dead, Akshay went towards the bush and realized
that it was actually a human being. He alerted the forest officials, who later confirmed that it was a poacher who had
been killed. Is Akshay responsible for having committed culpable homicide?

(a) No, as he did not have the time to be careful enough to check if it was indeed a tiger and he had to act in self-
defence.
(b) Yes, as he killed a person with the intention of doing an act causing death.
(c) Yes, but his punishment would be reduced as it is culpable homicide not amounting to murder.
(d) No, as he did not have the required intent to cause death of a person.

Principle: Nothing is an offence which is not intended to cause death, and is done by a person in good faith for the benefit of another.
The person on whom the particular act is done must have consented to that act being committed, with all the associated risks, whether
expressly or impliedly.
35. Facts: Tom is a world famous athlete. It was realized that he was suffering from a life threatening disease, for which
immediate surgery on his left leg was required. However, the surgery required immense expertise, and only very
few surgeons all over the world had knowledge about this particular surgery. One such surgeon was Dr. Alex, who
performed the surgery on Tom. Before the surgery, Tom was sensitized with all the associated risks, including death.
After the surgery, Tom lost his left leg, which had to be amputated to prevent a certain complication from reaching
his heart and other vital organs. Is Dr. Alex liable for medical negligence?

(a) No, as Dr. Alex took the decision to amputate Tom’s legs in order to save his life.
(b) No, as Dr. Alex had no intention to cause Tom’s death and had proceeded to perform the surgery in good faith.
(c) No, Tom had consented to the act with all the associated risks, and one of them can reasonably be deduced to
be possible amputation.
(d) All of the above.

Principles:
I. Nothing is an offence which is done in good faith for the benefit of a person under twelve years of age.
II. (proviso): This exception does not extend to the intentional causing of death.
36. Facts: Sara was an eleven year old child, who suffered from an unfortunate accident a couple of months ago and was
completely paralysed as a result of it. She had been in coma since the accident, and her doctors were certain that she
would not come out of it. Her mother, a doctor herself, decided to free her from her suffering and decided to remove
her life support system, so that she could die in peace. Suppose she acts upon her decision, should she be held liable
in light of the given principles?

(a) No, as the act was performed for the benefit of a terminally ill patient.
(b) No, euthanasia is legal if performed with the consent of a doctor, who in this case, is Sara’s mother.
(c) Yes, as she intentionally caused the death of Sara, by removing her life support system.
(d) No, as Sara’s mother did not intend to cause Sara’s death, but acted in good faith to end her suffering.

©LegalEdge Tutorials Page 29 of 76


Replication or other unauthorized use of this material is prohibited by the copyright laws of India
LE Prep Assist
Principle: A finder of a chattel does not acquire an absolute property right, but only has the title superior to everyone except the
rightful owner.
37. Facts: A found an envelope containing certain money on the floor of B’s restaurant. He immediately left the envelope
with B and asked him to return it to the rightful owner. After a few years, A enquired about the envelope with B and
found that they had remained unclaimed for all these years. When A asked B to hand him the envelope, B refused.
Suppose you are the judge of the trial court in which both A and B have presented their case. Decide the suitable
judgment strictly on the basis of the given principle.

(a) A has the rightful claim over the envelope since he is the finder of the envelope.
(b) B has the rightful claim since he is the owner of the property on which it was found.
(c) Neither of them have any claim, since the envelope does not belong to either of them.
(d) A has the claim since he had taken appropriate actions and had entrusted B with finding the rightful owner, but
B failed in his duties.

Principle: In a given country, the court looks to the custom and usage within an industry to determine the rule of law regarding the
ownership of the industry.
38. Facts: In a particular industry, it was the custom and usage that whoever manages to defeat the owner of the
industry in a specially held wrestling match, would be its rightful successor. X defeated the previous owner, Y, by
following all the rules of the wrestling match, and claimed his ownership. However, Y’s lawyer claimed in the court
of law that this custom was against any rationality and should be removed. Applying the given principle, answer if
Y should remain as the rightful owner of the industry?

(a) No, because Y was defeated in the wrestling match, and according to the established principle, the ownership
of the industry must be handed over to X.
(b) Yes, since the rule of law cannot possibly entertain a law that has an unfair basis for choosing persons in
important positions of industries.
(c) Yes, since it is a fraudulent custom and capacity to run an industry cannot be determined on the basis of
performance in a wrestling match.
(d) None of the above.

Principle: A party is responsible for damages caused as a direct result of their acts.
39. Fact: The Jamnabaad Municipal Corporation (JMC) building caught fire as a result of negligence of some of the
corporation’s employees. The fire also spread to other buildings including the Saj Hotel located about 2 kilometer
away from the JMC building. Saj owners sue JMC. Will they succeed?

(a) Yes, as a state body, it owed a duty to other private buildings in the area.
(b) Yes, it could be reasonably foreseen that the fire could have spread to other buildings.
(c) No, JMC owed no duty to Saj and hence, there was no breach of duty.
(d) No, the damage caused to Saj was remote and not proximate.

Principle:
I. Battery involves intent to cause the unwanted or offensive contact.
II. If the actor knew with reasonable degree of certainty that his action will result in the unwanted or offensive contact, then that
gives rise to liability.
40. Facts: 12 year old Sasha was playing in the backyard of Mrs. Paul’s house during her lunch party. Mrs. Timothy who
is about 75 years old and is suffering from arthiritis was also in the backyard and was about to sit on a chair when
Sasha pulled chair thinking it will be funny. Mrs. Timothy fell on the ground and hurt her hips pretty bad. She sued
Sasha. Will she succeed?

(a) No, Sasha is a minor. There cannot be a suit against a minor.


(b) Yes, Sasha wanted Mrs. Timothy to hit the ground.
(c) Yes, Sasha had a duty to inform Mrs. Timothy while taking the chair.
(d) No, Sasha did not have the requisite intent.

©LegalEdge Tutorials Page 30 of 76


Replication or other unauthorized use of this material is prohibited by the copyright laws of India
LE Prep Assist
Principle:
I. Battery involves intent to cause the unwanted or offensive contact.
II. If the actor knew with reasonable degree of certainty that his action will result in the unwanted or offensive contact, then that
gives rise to liability.
41. Fact: AbhaBai was washing clothes on a second floor balcony. The waste water from the washing machine was very
hot. On the road downstairs, a bunch of people was standing. AbhaBai knew they were standing there. She had to
drain the water out, so she threw the bucket full of waste water from the balcony. It fell on the people standing
downstairs causing burn injuries to some of them. They sue AbhaBai. Will they succeed?

(a) Yes, she knew the water was hot and it will cause some form of injuries to the people standing downstairs.
(b) Yes, she intended to burn them.
(c) No, she did not intend to burn them. She was merely throwing wastewater out of the balcony.
(d) No, she did not owe any duty to the people standing downstairs.

Principles:
I. At the commencement of the constitution, every person who has his domicile in the territory of India and a) was born in the territory
of India or b) either of whose parents were born in the territory of India or c) has been ordinarily a resident in the territory of India
5 years immediately preceding commencement, shall be a citizen of India.
II. Every Citizen of India shall have the Right to Freedom of Speech and Expression.
42. Facts: Raghu’s father Vishwanath (Born in Thiruvananthapuram, India on 10 th Nov 1958) is an Indian diplomat and
consequently has been posted in several countries throughout the world. Raghu was born on 2 nd May 1995 in London
and completed his schooling there. He decides to become a lawyer and appears for CLAT examination as an NRI. He
manages to get into NALSAR and moves to Hyderabad, India. After graduation from college he becomes disenchanted
with the ruling party and writes an incensed article criticising the Prime Minister which is published the Deccan Times.
In the middle of the night, the police enter Raghu’s home and take him away. He is charged with libel and put in lock-
up. Raghu approaches the court claiming he has the freedom of speech and expression, the Police argues that he was
born in London which implies that he is not a citizen of India and hence does not have the freedom of speech and
expression which is guaranteed to ‘citizens’ only.

Decide:
(a) Raghu is a citizen of India as he is now ordinarily resident in India.
(b) Raghu is a citizen of India as his Father was born in India.
(c) Both (a) and (b).
(d) Raghu is not a citizen of India though he is ordinarily resident in India as he was born in London, UK.

Principles: No person shall be convicted of any offence except for violation of a law in force at the time of the commission of the Act
charged as an offence.
43. Facts: Kharak Singh is an IAS officer who has assets worth 3.5 Crore INR. He has accumulated these assets since 1998
by illegal means i.e. through receiving bribes and tax evasion. The Prevention of Corruption Bill, 2015 envisages that
all government officials with disproportionate assets to pay shall be investigated. The Bill is passed by both houses of
the Parliament and comes into force on 1st May 2016. Subsequently Kharak Singh is investigated for Corruption. He
claims that the law will not apply to him as his assets were accumulated from 1998- April 2016 and the Act was passed
only on 1st May 2016 i.e. there is ex post facto application of law, the law would apply to him only for assets
accumulated on 1st May 2016 and onwards.

(a) Kharak Singh’s argument is valid.


(b) Kharak Singh’s argument is invalid as there is no time limit prescribed in the law, the law applies to all assets
accumulated whenever.
(c) Kharak Singh’s argument is valid as protection from ex post facto application of laws is a Constitutional right.
(d) Both (a) and (b).

Principle: Nothing is an offence which is done by a Judge when acting judicially in the exercise of any power which is, or which in
good faith he believes to be, given to him by law.
44. Facts: Justice Sikander Singh went to attend his nephew’s wedding where he criticised the Chief Minister and his party
for being religiously intolerant. He called him names and abused him. Bhagwan Das, a party member, was also present

©LegalEdge Tutorials Page 31 of 76


Replication or other unauthorized use of this material is prohibited by the copyright laws of India
LE Prep Assist
there. Incensed by the judge’s comments he files a case against him for slander. Justice Singh claims that he had said
the statements in judicial capacity.

(a) Justice Singh cannot be charged with slander as he is acting in judicial capacity.
(b) Justice Singh can be charged with slander as he used abusive language.
(c) Justice Singh cannot be charged with slander as he is acting in good faith as a member of the judiciary.
(d) Justice Singh can be charged with slander as he is not acting in judicial capacity. He was just attending a social
event.

Principle: Every person has the right against self-incrimination. Though a person can be forced to provide fingerprints or DNA or
other similar physical evidence.
45. Facts: Bijju Singh was suspected of committing murder. Inspector Sandhya was a very persistent interrogator and was
losing her patience with the non-cooperative Bijju. She had Bijju tied up and forcibly obtained fingerprints, blood
samples and DNA samples. Bijju’s prints were found on the murder weapon.

(a) Bijju’s right against self-incrimination was violated.


(b) Bijju’s right was not violated as fingerprints and DNA can be obtained by force.
(c) Bijju’s right vas violated only to the extent of the blood sample.
(d) Biiju’s right was not violated as he had actually committed murder.

Principles:
I. The Constitution of India has divided the power to make laws between the Central and State Legislatures by mentioning the
subjects for legislation under the Union (under Central Government), State (under State Government) and Concurrent List
(under both Central and State Governments) under the Seventh Schedule.
II. When the Central or State Legislatures exceed the subjects in their respective lists, the law so made is considered ultra vires the
Constitution and has no effect.
III. A subsequent retrospective amendment of the Constitution widening the scope of legislation of the said legislature cannot
validate such ultra vires legislation.
IV. A fresh law enacted by the said Legislature post such an amendment of the Constitution is considered valid.
V. A Legislature in exercise of its valid legislative power may extend to its territory a law made by another Legislature.
Facts: In light of the growing economic development in the state of Gujarat, the state legislature passed a law on 26 th
December 2012, setting up Gujarat State Railways Corporation to ply goods trains between commercial areas within
Gujarat. The law faced criticism since railways formed an entry under the Union List in the Seventh Schedule of the
Constitution. The Gujarat legislature stood by the legislation owing to the 118 th Constitutional Amendment of 28th
December 2012, shifting only commercial railways to the Concurrent List.

46. Post the Constitutional Amendment, laws in relation to passenger railways can be made by:
(a) State Legislatures
(b) Union Legislature
(c) Both State and Union Legislatures
(d) None of the above

Principles:
I. The Constitution of India has divided the power to make laws between the Central and State Legislatures by mentioning the
subjects for legislation under the Union (under Central Government), State (under State Government) and Concurrent List
(under both Central and State Governments) under the Seventh Schedule.
II. When the Central or State Legislatures exceed the subjects in their respective lists, the law so made is considered ultra vires the
Constitution and has no effect.
III. A subsequent retrospective amendment of the Constitution widening the scope of legislation of the said legislature cannot
validate such ultra vires legislation.
IV. A fresh law enacted by the said Legislature post such an amendment of the Constitution is considered valid.
V. A Legislature in exercise of its valid legislative power may extend to its territory a law made by another Legislature.
Facts: In light of the growing economic development in the state of Gujarat, the state legislature passed a law on 26 th
December 2012, setting up Gujarat State Railways Corporation to ply goods trains between commercial areas within
Gujarat. The law faced criticism since railways formed an entry under the Union List in the Seventh Schedule of the

©LegalEdge Tutorials Page 32 of 76


Replication or other unauthorized use of this material is prohibited by the copyright laws of India
LE Prep Assist
Constitution. The Gujarat legislature stood by the legislation owing to the 118 th Constitutional Amendment of 28th
December 2012, shifting only commercial railways to the Concurrent List.

47. In light of the validity or invalidity of the legislation passed, the railway lines installed by the Gujarat State Railways
Corporation:
(a) Can be removed by Union Government agencies.
(b) Shall be considered the valid property of Gujarat Government
(c) Must be protected and developed by the State Government.
(d) Cannot be removed by Union Government agencies.

Principles:
I. The Constitution of India has divided the power to make laws between the Central and State Legislatures by mentioning the
subjects for legislation under the Union (under Central Government), State (under State Government) and Concurrent List
(under both Central and State Governments) under the Seventh Schedule.
II. When the Central or State Legislatures exceed the subjects in their respective lists, the law so made is considered ultra vires the
Constitution and has no effect.
III. A subsequent retrospective amendment of the Constitution widening the scope of legislation of the said legislature cannot
validate such ultra vires legislation.
IV. A fresh law enacted by the said Legislature post such an amendment of the Constitution is considered valid.
V. A Legislature in exercise of its valid legislative power may extend to its territory a law made by another Legislature.
Facts: In light of the growing economic development in the state of Gujarat, the state legislature passed a law on 26 th
December 2012, setting up Gujarat State Railways Corporation to ply goods trains between commercial areas within
Gujarat. The law faced criticism since railways formed an entry under the Union List in the Seventh Schedule of the
Constitution. The Gujarat legislature stood by the legislation owing to the 118 th Constitutional Amendment of 28th
December 2012, shifting only commercial railways to the Concurrent List.

48. Assuming that the entire subject of railways was moved to the Concurrent list under the 118 th Constitutional
Amendment:
(a) The Legislation enacted by the Gujarat Legislature shall be valid.
(b) The Gujarat State Railway Corporation would be permitted to continue its operations.
(c) The Legislation would be considered ultra vires.
(d) The Union would become incompetent to make laws related to railways.

Principles:
I. The Constitution of India has divided the power to make laws between the Central and State Legislatures by mentioning the
subjects for legislation under the Union (under Central Government), State (under State Government) and Concurrent List
(under both Central and State Governments) under the Seventh Schedule.
II. When the Central or State Legislatures exceed the subjects in their respective lists, the law so made is considered ultra vires the
Constitution and has no effect.
III. A subsequent retrospective amendment of the Constitution widening the scope of legislation of the said legislature cannot
validate such ultra vires legislation.
IV. A fresh law enacted by the said Legislature post such an amendment of the Constitution is considered valid.
V. A Legislature in exercise of its valid legislative power may extend to its territory a law made by another Legislature.
Facts: In light of the growing economic development in the state of Gujarat, the state legislature passed a law on 26 th
December 2012, setting up Gujarat State Railways Corporation to ply goods trains between commercial areas within
Gujarat. The law faced criticism since railways formed an entry under the Union List in the Seventh Schedule of the
Constitution. The Gujarat legislature stood by the legislation owing to the 118 th Constitutional Amendment of 28th
December 2012, shifting only commercial railways to the Concurrent List.

49. If the Gujarat Legislature decides to amend the said enactment to bring it in consonance with the 118 th Constitutional
Amendment on 4th January 2013:
(a) The Legislation would no longer be ultra vires.
(b) The Union Government will lose the power to challenge the legislation.
(c) The Gujarat State Railway Corporation will become a validly created entity.
(d) The Legislation will continue to have no effect.

©LegalEdge Tutorials Page 33 of 76


Replication or other unauthorized use of this material is prohibited by the copyright laws of India
LE Prep Assist
Principles:
I. The Constitution of India has divided the power to make laws between the Central and State Legislatures by mentioning the
subjects for legislation under the Union (under Central Government), State (under State Government) and Concurrent List
(under both Central and State Governments) under the Seventh Schedule.
II. When the Central or State Legislatures exceed the subjects in their respective lists, the law so made is considered ultra vires the
Constitution and has no effect.
III. A subsequent retrospective amendment of the Constitution widening the scope of legislation of the said legislature cannot
validate such ultra vires legislation.
IV. A fresh law enacted by the said Legislature post such an amendment of the Constitution is considered valid.
V. A Legislature in exercise of its valid legislative power may extend to its territory a law made by another Legislature.
Facts: In light of the growing economic development in the state of Gujarat, the state legislature passed a law on 26 th
December 2012, setting up Gujarat State Railways Corporation to ply goods trains between commercial areas within
Gujarat. The law faced criticism since railways formed an entry under the Union List in the Seventh Schedule of the
Constitution. The Gujarat legislature stood by the legislation owing to the 118 th Constitutional Amendment of 28th
December 2012, shifting only commercial railways to the Concurrent List.

50. An enactment similar to the one passed by the Gujarat government was passed by the Bihar Government on 16 th
January 2013. This legislation would be:
(a) Invalid (b) Valid (c) Subject to court approval (d) Ultra vires

©LegalEdge Tutorials Page 34 of 76


Replication or other unauthorized use of this material is prohibited by the copyright laws of India
LE Prep Assist

Prep Test– Mathematics


Time: 15 min. No. of Questions: 20
1. Following graph shows which of the following linear equation

(a) 𝑥 + 4𝑦 = 8
(b) 𝑥 + 4𝑦 = −8
(c) – 𝑥 + 4𝑦 = 8
(d) 𝑥 − 4𝑦 = 8

2. The average speed for an entire journey is 60 kmph, without considering the stoppages. When the stoppage is
considered the average speed become 48 kmph. How many minutes per hour on an average was the stoppage?
(a) 10 minutes.
(b) 12 minutes.
(c) 16 minutes.
(d) 18 minutes

3. The diagonal of a square is 4√2 cm. The diagonal of another square whose area is double that of the first square is:
(a) 8 cm
(b) 8√2 cm
(c) 4√2cm
(d) 16 cm

4. Find the largest 4 digit number which, when divided by 19 leaves a remainder of 6.
(a) 9984
(b) 9978
(c) 9999
(d) 9981

5. How many natural numbers up to 200 are divisible 2 and 3 both?


(a) 35
(b) 33
(c) 29
(d) 27

6. A man spends 35% of his income on food, 25% on children’s education and 80% of the remaining on house rent. What
percent of his income he is left with?
(a) 8%
(b) 10%
(c) 12%
(d) 14%

7. A cricketer has a certain average for 9 innings. In the tenth he scores 50, thereby increasing his average by 4 runs.
What is his new average?
(a) 14

©LegalEdge Tutorials Page 35 of 76


Replication or other unauthorized use of this material is prohibited by the copyright laws of India
LE Prep Assist
(b) 40
(c) 20
(d) 30

8. The rate of flow of water of Pipe A is 30 litres per minute, and pipe B is 50 litres per minute. If Pipe A is used for
empyting a tank, B for filling and both are opened simultaneously, what is the volume of the tank if it gets full in 40
minutes?
1. 200
2. 1,000
3. 750
4. 800

9. From a vessel containing only milk, 10 litres are drawn and replaced with water. 10 litres of the mixture is now
taken out and replaced with water again. The ratio of milk to water now is 25 : 24. How many litres of milk was
there initially?
(a) 20 litres
(b) 25 litres
(c) 32 litres
(d) 35litres

10. In a partnership A invested 1/6th of the capital for 1/6th of the time, B invest 1/3rd of the capital for 1/3rd of the time
and C, the rest of the capital for the whole time. Out of a profit of Rs. 4600, B’s share is:
(a) 650
(b) 800
(c) 960
(d) 1000

11. A trader marked the price of his commodity so as to include a profit of 25%. He allowed discount of 16% on the
market price. His actual profit was?
(a) 5%
(b) 9%
(c) 16%
(d) 25%

12. Out of 5 men and 3 women, a committee of 3 members is to be formed so that it has 1 woman and 2 men. In how
many different ways can it be done?
(a) 20
(b) 10
(c) 23
(d) 30

13. Of 3600 apples, A gets half, B gets one third and C gets one-twelth. Find the number of apples each gets and the
remaining number of apples after A gives 1⁄5 of the original number back.
(a) A = 1080, B = 1200, C = 300, 1020
(b) A = 1300, B = 1600, C = 500, 1200
(c) A = 1800, B = 800, C = 900, 1020
(d) None of these.

14. Murari has 9 pairs of dark blue socks and 9 pairs of black socks. he keeps them all in the same bag. if he picks out
three socks at random, what is the probability that he will get a matching pair?
9c3 ×9c1
(a)
18c3
2×9c2 ×9c
1
(b)
18c3

©LegalEdge Tutorials Page 36 of 76


Replication or other unauthorized use of this material is prohibited by the copyright laws of India
LE Prep Assist
(c) 1
4
(d)
7

15. Find the length of PA in the figure given below, if PO = 5 cm, the diameter of the circle is 6 cm, given PA and PB are
tangents.

(a) √34 cm
(b) 4 cm
(c) 3 cm
(d) √11 cm

16. Find the centroid of triangle whose angular points are (3, 5), (−7, −4) and (10,2), respectively:
(a) (1,2)
(b) (2,1)
(c) (2,3)
(d) (2,4)

17. The difference between compound interest and simple interest on a sum for 2 year at 10% per annum, when the
interest is compounded annually is Rs.16. If the interest were compounded half yearly, the difference in two
interests would be?
(a) 24.81 (b) 26.90 (c) 31.61 (d) 32.40

18. The proportion of milk and water in 3 samples is 3 : 1, 3 : 2 and 5 : 1. A mixture comprising of equal quantities of all
3 samples is made. The proportion of milk and water in the mixture is?
(a) 11 : 4
(b) 5 : 2
(c) 335 : 121
(d) 131 : 49

19. A merchant wants to make profit by selling food grains.


Which of the following would maximize his profit?
I. Sell product at 30% profit
II. Increase the price by 15% over the cost price and reduce weight by 15%
III. Use 700 g of weight instead of 1 kg.
IV. Mix 30% impurities in grains and sell it at cost price
(a) III
(b) II and I
(c) II
(d) All give the same profit

20. If n = 1 + 𝑥, where 𝑥 is the product of 4 consecutive positive integers, then which of the following is/are true?
I. n is odd.
II. n is prime.
III. n is a perfect square.
(a) I and III only (b) I and II only (c) I only (d) None of the above

©LegalEdge Tutorials Page 37 of 76


Replication or other unauthorized use of this material is prohibited by the copyright laws of India
LE Prep Assist

1. Which Indian think tank organize a FinTech 8. What was the theme of International Day of
Conclave in New Delhi recently? Remembrance of the Victims of Slavery and the
(a) Centre for Land Warfare Studies Transatlantic Slave Trade 2019?
(b) Centre for Science and Environment (a) Theme -“Remember Slavery: The Power of the
(c) NITI Aayog Arts for Justice”
(d) Research Foundation for Governance in India (b) Theme -“Remember Slavery: Triumphs and
Struggles for Freedom and Equality”
2. Which country topped in the second edition of (c) Theme -“Breaking the Silence, Lest We Forget”
WEF global Energy Transition index 2019? (d) Theme -“Women and Slavery”
(a) Finland
(b) Norway 9. On the basis of which law, centre has banned
(c) Switzerland Jammu Kashmir Liberation Front (JKLF)?
(d) Swedan (a) Indian Law Reports Act, 1875
(b) Unlawful Activities Prevention Act (UAPA),
3. Name the organization, that presents National 1967
Academy Awards to promote visual arts such as (c) Unlawful Association Prevention Act(UAPA),
painting, sculpture and graphics etc. 1943
(a) National Center For Performing Arts (NCPA) (d) Prevention of Unlawful Association
(b) Indian National Trust for Art and Cultural Act(PUAA), 1974
Heritage (INTACH)
(c) Lalit Kala Academy 10. Who won Life Time Achievement Award in the
(d) Indian Numismatic Historical and Cultural 64th Film Fare Awards 2019?
Research Foundation (INHCRF) (a) Sridevi
(b) Amitabh Bachchan
4. Who was re-elected as the president of the (c) Aamir Khan
Basketball Federation of India (BFI)? (d) Shah Rukh Khan
(a) Ugur Erdener
(b) Riccardo Fraccari 11. Who won the Men’s Singles title in 2019 Lingshui
(c) K.Govindaraj China Masters?
(d) Poul Erik Hoyer Larsen (a) Yang Po-hsuan
(b) Weng Hongyang
5. Which online payment system has tied up with (c) Liu Haichao
US-based e-commerce platform eBay to manage (d) Lee Jhe-huei
payment option?
(a) Amazon Pay 12. Name the Indian batsman, who became the first
(b) PayPal person to surpass 5000 runs in Indian Premier
(c) Google Pay League(IPL).
(d) Paytm (a) Virat Kohli
(b) MS Dhoni
6. Which country won the 2019 Badminton Asia (c) Suresh Raina
mixed team trophy? (d) Umesh Yadav
(a) Thailand
(b) Japan 13. What was the theme of the World Tuberculosis
(c) Nepal Day 2019?
(d) China (a) Theme – ‘Wanted: Leaders for a TB-free world’
(b) Theme – ‘Its time’
7. South Asian singer Shahnaz Rahmatullah passed (c) Theme – ‘Unite to End TB’
away at the age of 67 years.She belongs to which (d) Theme – ‘World free from TB’
country?
(a) India 14. When was International Day for the Right to the
(b) Bangladesh Truth Concerning Gross Human Rights Violations
(c) Bahrain and for the Dignity of Victims observed?
(d) Malaysia (a) 22nd March
(b) 23rd March
©LegalEdge Tutorials Page 38 of 76
Replication or other unauthorized use of this material is prohibited by the copyright laws of India
LE Prep Assist
(c) 21st March 21. What was the theme of the World Meteorological
(d) 24th March Organization (WMO) 2019?
(a) Theme – “Weather-Ready, Climate-Smart”
15. Which country has received financial grant from (b) Theme – “The Sun, the Earth and the Weather”
India, for constructing educational campus to (c) Theme – “Understanding Clouds”
boost the learning environment of the students? (d) Theme – “Climate and Water”
(a) Bangladesh
(b) Srilanka 22. Name the Indian personality who was not in the
(c) Nepal list of world’s 100 Most Influential People in
(d) Myanmar Climate Policy for 2019.
i.Piyush Goyal
16. Name the India’s first forest certification ii.Harsh Vardhan
scheme,which gets global recognition by a Geneva iii.Mukta Tilak
based non-profit firm. iv.Narendra Modi
(a) Certification Standard for Habitat restoration v.Sunita Narain
and improvement (a) All Options excluding i
(b) Certification Standard for Sustainable Forest (b) All Options excluding ii
Management (c) All Options excluding iii
(c) Certification Standard for Protection and (d) All Options excluding iv
Conservation measures
(d) Certification Standard for Mitigation of 23. Who was appointed as the chief financial officer of
Human Wildlife Conflict the Ford Motor Co?
(a) Edsel Ford II
17. Rear Admiral Sanjay Jasjit Singh was appointed as (b) Tim Stone
the Fleet commander in which of the following (c) James Hackett
Naval command? (d) Marcy Klevorn
(a) Andaman & Nicobar Command
(b) Southern Naval Command 24. NSEIT acquired Global cyber security company
(c) Eastern Naval Command Aujas recently. NSEIT is a Subsidiary
(d) Western Naval Command of_______________.
(a) Bombay Stock Exchange
18. Gafur Rahimov ,who recently resigned was the (b) National Stock Exchange
president of which association? (c) London Stock Exchange
(a) International Boxing Association (amateur) (d) New York Stock Exchange
(b) International Gymnastics Federation
(c) International Handball Federation 25. Name the UPI-Based Payments App launched by
(d) International Ice Hockey Federation the Xiaomi in India.
(a) Trans Pay
19. Name the Artificial Intelligence(AI),that is (b) UPI Pay
launched by IIT Madras in partnership with ESPN (c) Xi Pay
cric info to analyse cricket statistics? (d) Mi Pay
(a) Cricstats
(b) Superstats 26. Which of the following mobile application was
(c) ESPNstats launched by Central Board of Secondary
(d) StatCric Education (CBSE) to upload podcast on Class 10,
12 exam evaluation process?
20. Lasith Malinga announced his retirement from (a) Umang
international cricket after ICC Men’s T20 World (b) MooFarm
Cup 2020.He belongs to which country? (c) ReUnite
(a) Srilanka (d) Shiksha Vani
(b) Pakistan
(c) Bangladesh
(d) India 27. Name the intelligence blockchain,that is to be
launched by Intain Fintech for facilitating secure,
efficient and trustworthy asset security
©LegalEdge Tutorials Page 39 of 76
Replication or other unauthorized use of this material is prohibited by the copyright laws of India
LE Prep Assist
transactions for Non-Banking Financial 34. What was the theme of the World Water Day
Companies (NBFCs) in India. 2019?
(a) eNBFC (a) Theme – “Nature for Water”
(b) eSecure (b) Theme – “Why Waste Water?”
(c) eMulya (c) Theme – “Leaving no one behind”
(d) eTransact (d) Theme –“ Water and Jobs”

28. Which country won the South Asian Football 35. Where was the 33rd edition of India-Indonesia
Federation (SAFF) Women’s Championship 2019 Coordinated Patrol (Ind-Indo Corpat)inaugurated
for the fifth year consecutively? in order to enhance interoperability and
(a) Bangladesh strengthen existing bonds between India &
(b) Maldives Indonesia?
(c) Nepal (a) Jakarta,Indonesia
(d) India (b) New Delhi,India
(c) Port Blair,India
29. Who has been imposed lifetime ban by the World (d) Serang,Indonesia
football governing body FIFA?
(a) John Terry 36. What was the theme of the first ever High-Level
(b) Emre Belozoglu Dialogue on Indo-Pacific Cooperation (HLD-
(c) Luis Chiriboga IPC),that held in Jakarta, Indonesia?
(d) Luis Suarez (a) Theme – “Towards a Peaceful, Prosperous and
Inclusive Region ”
30. What is the recent official name of European T20 (b) Theme – “Digital Disruption and Innovation:
league? Future of Procurement ”
(a) Euro T20 Slam (c) Theme – “Reviving the Millennial Partnership
(b) Euro T20 ”
(c) Euro T20 league (d) Theme – “Building partnerships through
(d) European T20 Slam global value Chain ”

31. Name the renowned Indian artist and Padma Shri 37. Which bank has signed loan agreement worth
awardee passed away recently at the age of 85 USD 50 million with Avaada Energy Private
years? Limited (AEPL) to expand the Renewable Energy
(a) Haku Shah Capacity in India?
(b) Andrew Lesnie (a) World Bank
(c) Peter Dickinson (b) Asian Development Bank (ADB)
(d) Ankit Keshri (c) European Investment Bank (EIB)
(d) New Development Bank (NDB)
32. Who is the author of the book “The Great
Disappointment: How Narendra Modi 38. Name the first latin American,who won the 2019
Squandered A Unique Opportunity to Transform Templeton Prize worth $1.4m for combining
the Indian Economy “ that Analysis PM Modi’s science and spirituality in his work.
Impact on Economy? (a) King Abdullah II
(a) Anita Desai (b) Marcelo Gleiser
(b) Kiran Desai (c) Dalai Lama
(c) Salman Rushdie (d) Mike Brown
(d) Salman Anees Soz
39. Which football club won the l–League Champion
33. Which Indian English word is added as the 650th by All India Football Federation (AIFF) in
entry in Oxford Dictionary? Chennai?
(a) Mantra (a) Minerva Punjab
(b) Chuddies (b) Bengaluru FC
(c) Loot (c) Chennai City
(d) Multan (d) Aizawl

©LegalEdge Tutorials Page 40 of 76


Replication or other unauthorized use of this material is prohibited by the copyright laws of India
LE Prep Assist
40. Name the former India hockey captain, who was and respect for diversity in the context of
appointed in Olympic Council of Asia (OCA) combatting racial
Standing Committee recently. discrimination ”
(a) Harendra Singh (b) Theme – “Mitigating and countering rising
(b) Manpreet Singh nationalist populism and extreme supremacist
(c) Sandeep Singh ideologies ”
(d) Sardar Singh (c) Theme – “Us and Them – From Prejudice to
Racism”
41. Name the Japanese unmanned spacecraft,that (d) Theme – “Mobilizing Global Capital for
probes Asteroid Ryugu and discoverd small Innovation ”
amount of minerals containing water on the
surface of the asteroid? 47. What was the focus of the World Down Syndrome
(a) Hiten Day 2019?
(b) Hayabusa2 (a) Leave no one behind
(c) Akatsuki (b) Reviving the Millennial Partnership
(d) SELENE (c) Shaping a New World
(d) One World One Sun One Grid
42. What is the name of the new service unveiled by
Google that allows people to play high-end games 48. When was World Poetry Day observed?
without purchasing expensive consoles? (a) 19th March
(a) Stadia (b) 20th March
(b) GDC (c) 21st March
(c) GDC 2019 (d) 18th March
(d) Streaming service
49. What was the theme of the International Day of
43. Which city has signed to host T10 league,the Forests 2019?
shortest form of cricket for five years? (a) Theme-“ Building partnerships through global
(a) Beijing value Chain of Forests”
(b) Melbourne (b) Theme-“ Forests and Education – Learn to
(c) Barcelona Love Forests”
(d) Abu Dhabi (c) Theme-“ Mobilizing Global Capital for
Innovation in India”
44. India participated in which of the following sport (d) Theme-“ Good Governance – Focus on
for the first time in 15th Special Olympics World Aspirational Districts and Forests”
Summer Games 2019 held in Abu Dhabi,United
Arab Emirates? 50. Name the top three cities,that share the top spot in
i.Judo the world’s most expensive city for first time in
ii.Futsal the 30 year long history according to Economist
iii.Powerlifting Intelligence Unit(EIU) survey.
(a) Option i i. Singapore
(b) Option i & ii ii. Paris
(c) Option ii & iii iii. Hong Kong
(d) Option i,ii & iii iv. Geneva
(a) Option i alone
45. Name the 1971 India-Pakistan war hero and Maha (b) Option i & ii alone
Vir Chakra awardee,who passed away recently. (c) Option i,ii & iii
(a) Vikram Batra (d) All the above mentioned Options
(b) Mohammad Usman
(c) Mohan Narayan Rao Samant 51. Where did 4th Round of Free Trade Agreement
(d) Jaswant Singh Rawat (FTA) to enhance the two-way commerce between
India and Peru held?
(a) Trujillo,Peru
46. What was the theme of the International Day for (b) Bengaluru,India
the Elimination of Racial Discrimination 2019? (c) New Delhi,India
(a) Theme – “Promoting tolerance, inclusion, unity
©LegalEdge Tutorials Page 41 of 76
Replication or other unauthorized use of this material is prohibited by the copyright laws of India
LE Prep Assist
(d) Lima,Peru (c) Option ii & iii
(d) Option iii & iv
52. Name the bank,that has signed a loan agreement
worth USD 83.11 million to Congo for financing 58. When was the World Sparrow day observed?
three solar power projects. (a) 17th March
(a) State Bank of India(SBI) (b) 18th March
(b) HDFC Bank (c) 19th March
(c) Export-Import Bank of India (Exim Bank) (d) 20th March
(d) RBL Bank
59. What was the theme of International Day of
53. According to World Intellectual Property Happiness 2019?
Organization (WIPO),which country filed the (a) Theme – “Happier Together ”
greatest number of Patent Cooperation Treaty (b) Theme – “Share Happiness”
(PCT),patent applications in 2018? (c) Theme – “ Happiness to be Shared ”
(a) Germany (d) Theme – “Live Happily ”
(b) Japan
(c) China 60. India Post has released a special stamp cover on
(d) US which environmental initiative that is aimed to
reduce shortage of water in Leh?
54. Name the first Indian low-cost carrier,which (a) Painted Stork
received International Air Transport Association (b) Smooth Indian Otter
(IATA) membership recently. (c) Ice Stupas
(a) SpiceJet (d) Solar Energy
(b) GoAir
(c) Air India 61. Where did 3rd edition of Indo-Japan workshop on
(d) IndiGo Disaster Risk Reduction held recently?
(a) Yokohama,Japan
55. Name the Indian psychologist professor,who won (b) Karnataka,India
the Hans-Kilian Award 2019 for providing deeper (c) Tokyo,Japan
insight regarding human psyche. (d) New Delhi,India
(a) Vipul Rastogi
(b) Kamna Chhibber 62. Which state for the first time has undertaken
(c) Ashis Nandy census of otters in protected areas?
(d) Subin Vazhayil (a) Uttar Pradesh
(b) Maharashtra
56. K M Mammen was appointed as the chairman of (c) New Delhi
which Association? (d) West Bengal
(a) National Association Of Software And Service
Companies 63. Name the bank of India,which had signed line of
(b) Automotive Tyre Manufacturers’ Association credit (LOC) worth $800 million to finance the
(ATMA) infrastructure projects in Maldives during
(c) Indian Banks Association (IBA) External Affairs Minister Sushma Swaraj visit?
(d) Indian Drug Manufacturers’ Association (a) ICICI Bank
(b) State Bank of India
57. Who were the 2 retired Indian Revenue (c) HDFC Bank
Service(IRS) officers appointed as the special (d) Export Import Bank of India (EXIM)
expenditure observers for the upcoming Lok
Sabha Election by Election Commission of India? 64. Name the organization,that became the first
i. Vinod Zutshi clearing corporation of India to receive recognition
ii. Madhu Mahajan as a Third-Country Central Counterparty (TC-
iii. Shailendra Handa CCP) by Bank of England for offering various
iv. A.S.Gill. clearing services to UK-based entities.
(a) Option i & ii (a) NSE IFSC Clearing Corporation Limited
(b) Option i & iii (b) Metropolitan Clearing Corporation of India
Ltd (MCCIL)
©LegalEdge Tutorials Page 42 of 76
Replication or other unauthorized use of this material is prohibited by the copyright laws of India
LE Prep Assist
(c) National Securities Clearing Corporation Ltd. 71. What was the theme of the Global Recycling Day
(d) India International Clearing Corporation 2019?
(IFSC) Limited (a) Theme – “Recycling into the Future”
(b) Theme – “Recycling the Global Economy ”
65. Name the book for which,Raghu Karnad was (c) Theme – “Recycling the Environment ”
awarded with Windham-Campbell prize in the (d) Theme – “Recycling the Future world”
UK?
(a) Good Talk: A Memoir in Conversations by 72. Name the largest exercise in terms of geographical
Mira Jacob spread covering the IOR and also with regard to
(b) Daisy Jones & The Six by Taylor Jenkins Reid the number of participating units that held in
(c) Farthest Field: An Indian story of Second Andaman & Nicobar Islands.
World War (a) Vajra Prahar
(d) Survival Math: Notes on an All-American (b) SEA VIGIL
Family by Mitchell S. Jackson (c) Yudh Abhyas XIV
(d) Theatre Level Operational Readiness Exercise
66. Who was awarded with ‘Uttam Yudh Seva Medal’ (TROPEX 19)
for successfully organizing the counter-terrorist
operations in Jammu & Kashmir? 73. Name the Campaign launched by Prime Minister
(a) Pradip Kumar Panda Narendra Modi for fighting corruption.
(b) Anil Kumar Bhatt (a) Paudhagiri
(c) Vijay Kumar (b) Incredible India
(d) Irfan Ramzan Sheikh (c) Swachhata Hi Seva
(d) Main Bhi Chowkidar
67. Name the person,who has assumed charge as the
Flag Officer Sea Training (FOST) at Kochi after 74. Who was appointed as the head of Project
succeeding Rear Admiral Sanjay J Singh? Implementation and Sanctioning Committee
(a) Krishna Swaminathan under FAME-II programme?
(b) Karambir Singh (a) Amitabh Kant
(c) Ajit Kumar (b) Babul Supriyo
(d) Ashok Kumar (c) Dr.A.R.Sihag
(d) Anant Geete
68. Who was the first Indian appointed as Cultural &
Tourism Ambassador of Uzbekistan in India? 75. Which Insurance Company partnered with
(a) Benegal Rama Rau MobiKwik to provide cyber-insurance cover of
(b) Subrahmanyam Jaishankar Rs.50000?
(c) Arun Kumar Singh (a) New India Assurance
(d) Ritu Beri (b) SBI General Insurance
(c) Acko General Insurance
69. Which app was developed by Indian Institute of (d) ICICI Lombard
Public Health Gandhinagar, Gujarat, (IIPHG) in
collaboration with National Mental Health 76. Which Organization has tied up with NASSCOM
Program to reduce the exam pressure? Foundation,to tackle the challenge of
(a) Reduce Pressure misinformation?
(b) Conquer Exam, Be a Warrior (a) Facebook
(c) Tension Free (b) WhatsApp
(d) Attempt Exam (c) Instagram
(d) Twitter
70. Name the US Economist, who advised its two
presidents, has passed away recently at the age of 77. Name the 13 year old Indian pianist,from
58 years? Chennai,who won a million dollars for his
(a) Kenneth Arrow performance in the American reality show “The
(b) Paul Samuelson World’s Best”.
(c) Milton Friedman (a) Lydian Nadhaswaram
(d) Alan B.Krueger (b) Stephen Devassy
(c) Ashley William Joseph
©LegalEdge Tutorials Page 43 of 76
Replication or other unauthorized use of this material is prohibited by the copyright laws of India
LE Prep Assist
(d) Satyajit Prabhu 2019.Bianca Andreescu belongs to which country?
(a) Germany
78. Who topped in the Forbes list titled “World’s (b) Canada
Richest Sports Team Owners 2019”? (c) Switzerland
(a) Roman Abramovich (d) Argentina
(b) Steve Ballmer
(c) Dietrich Mateschitz 86. Name the Indian Player who was defeated by Shi
(d) Mukesh Ambani Yuqi of China in Men’s Singles title in 2019 Swiss
Open badminton championship held at St.
79. Name the Indian filmmaker,who has been chosen Jakobshalle in Basel, Switzerland.
for 22nd Gollapudi Srinivas National Award. (a) Srikanth Kidambi
(a) Rima Das (b) Pullela Gopichand
(b) Vetrimaaran (c) Sai Praneeth
(c) C Prem Kumar (d) Sameer Verma
(d) Shankar
87. Avantika Santhosh Narale won gold in the Asian
80. Which bank is restricted by the Reserve Bank of Youth Athletes Championship held in Hong
India(RBI),to hold 9.9% stake in Bandhan Bank? Kong.She belongs to which sport?
(a) Yes Bank (a) Sprint
(b) ICICI Bank (b) Relay
(c) HDFC Bank (c) Shot put
(d) Axis Bank (d) Javelin

81. Which Indian Coast Guard Ship,became the first- 88. Name the Indian Athelete and Asian marathon
ever ship to visit Sabang,Indonesia to ensure champion,who has qualified for the World
maritime security and safety? Athletics Championships to be held in Doha.
(a) Samarth (a) Aparna Roy
(b) Vishwast (b) Gopi Thonakal
(c) Samudra (c) Nisar Ahmed
(d) Vijit (d) Praveen Chitravel

82. Name the Mercedes driver,who won the 89. Name the former Defence Minister and four-time
Australian F1 Grand Prix 2019. Goa chief minister who passed away recently,for
(a) Esteban Ocon whom the Central government announced
(b) Michael Schumacher national mourning on 18th March 2019.
(c) Lewis Hamilton (a) Laxmikant Parsekar
(d) Valtteri Bottas (b) Manohar Parrikar
(c) Digambar Kamat
83. Who won the first Indian Super League (ISL) title (d) Pratapsingh Rane
at the Mumbai Football Arena?
(a) Delhi Dynamos Football Club 90. Chinmoy Roy passed away recently at the age of
(b) Chennaiyin Football Club 79 in Salt lake,Kolkata,West Bengal.He belongs to
(c) Bengaluru Football Club which field?
(d) Jamshedpur Football Club (a) Director
(b) Politician
84. Who won the first Men’s Singles Title in Indian (c) Actor
Wells Masters 2019 also known as BNP Paribas (d) Producer
Open that held in California, United States?
(a) Nikola Mektic 91. When was the Ordnance Factory day observed?
(b) Roger Federer (a) 16th March
(c) Dominic Thiem (b) 17th March
(d) Horacio Zeballo (c) 18th March
(d) 19th March
85. Bianca Andreescu defeated Angelique Kerber to
win Women’s Singles title in Indian Wells Masters
©LegalEdge Tutorials Page 44 of 76
Replication or other unauthorized use of this material is prohibited by the copyright laws of India
LE Prep Assist
92. Which of the following institute will adopt 98. Darryl D’Monte passed away at age of 74 years in
Amazon Web Services (AWS) Educate programme Mumbai.He belongs to which of the following
to help students gain cloud computing skills field?
including hands-on experience in artificial (a) Journalist and Environmental activist
intelligence (AI)? (b) Actor and Director
(a) Indian Institute of Technology Kanpur (c) Director and Producer
(b) Indian Institute of Technology Goa (d) Actor and Politician
(c) Indian Institute of Technology (IIT Kharagpur)
(d) Indian Institute of Technology(IIT Hyderabad) 99. India’s leading think-tank,Energy and Resources
Institute (TERI) will set up an industrial
93. How many days is given as default for resolving incubation facility of TERI-Deakin
the loan account which is more than Rs.2000 crore Nanobiotechnology Research Centre in which
under the Insolvency and Bankruptcy Code (IBC)? state?
(a)180 days (a) Karnataka
(b) 90 days (b) New Delhi
(c) 60 days (c) Odisha
(d) 30 days (d) Tamil Nadu

94. Where did two-day national convention of War 100. Where was the 3rd edition of Namaste Thailand
Decorated India held? festival held with an aim to strengthen bilateral
(a) Adampur,Varanasi ties and increase cultural exchange?
(b) Agwanpur,Patna (a) Mumbai
(c) Abhaypur,Guwahati (b) New Delhi
(d) Chandimandir,Chandigarh (c) Guwahati
(d) Kolkata
95. Who was appointed as whole-time directors for a
three-year term in Kotak Mahindra Bank? 101. Who is the head of 5-member committee, that is
i. KVS Manian constituted by RBI to strengthen digital payments?
ii. Ajay Kumar (a) Nandan Nilekani
iii. Gaurang Shah (b) Shaktikanta Das
(a) Option i and ii (c) Urjit Patel
(b) Option ii and iii (d) Viral Acharya
(c) Option i and iii
(d) All the above Options 102. Who was conferred with Global Teacher Prize
2019?
96. Read the following and find the correct statement. (a) Nurten Akkus
i. Astronomers have discovered 83 quasars (b) Marjorie Brown
powered by supermassive black holes. (c) Glenn Lee
ii.The finding is published in the Astrophysical (d) Peter Tabichi
Journal.
iii.It is 15 billion light-years away from the Earth. 103. Naresh Goyal,who recently resigned from the
(a) Option i is correct board was founder of which airlines?
(b) Option i & ii are correct (a) Jet Airways
(c) Option ii & iii are correct (b) IndiGo
(d) All the Options are correct (c) Air India

97. Who became the 2nd Indian golfer to win on 104. Name the manufacturer of Chinook helicopters
Ladies European Tour in South African Women’s that was recently inducted into Indian Air
Open? Force(IAF)?
(a) Anisha Padukone (a) Lockheed Martin, Washington DC
(b) Laura Davies (b) Boeing, United States
(c) Georgia Hall (c) Raytheon, United States
(d) Diksha Dagar (d) Hindustan Aeronautics Limited, India

©LegalEdge Tutorials Page 45 of 76


Replication or other unauthorized use of this material is prohibited by the copyright laws of India
LE Prep Assist
105. Which city became the world’s first city to install language with Washington University?
wireless, induction-based charging stations? (a) Intel (b) IBM
(a) London, England (b) Oslo, Norway (c) Google (d) Microsoft
(c) Bangkok, Thailand
(d) Kuala Lumpur, Malaysia 114. Name the Operation launched by India to provide
Humanitarian Assistance and Disaster Relief
106. When was the International Day of Solidarity with (HADR) support to Mozambique,which was hit by
Detained and Missing Staff Members observed? cyclone IDAI.
(a) 23rd March (b) 24th March (a) Operation Sahayata 19
(c) 25th March (b) Operation Trident 19
(d) 22nd March (c) Operation Cactus 19
(d) Operation Parakram 19
107. Which country has became the first G7 country to
join China’s ‘Belt and Road’ initiative? 115. When was Martyr’s Day (88th) or Shaheed Diwas
(a) Italy (b) Japan observed?
(c) Canada (d) France (a) 21st March (b) 22nd March
(c) 23rd March (d) 20th March
108. Which film won the 64th Film Fare Awards 2019
under the Best Film Category? 116. Reserve Bank of India (RBI) has turned down
(a) Patmaavat (b) Sanju which of the following bank’s proposal to change
(c) Andhadhun (d) Raazi its name recently?
(a) IDBI Bank (b) Dena Bank
109. Name the Italy’s new Earth-observation satellite (c) Bank of Baroda (d) Vijaya Bank
launched from the Guiana Space Centre by 98 foot
(30 meters) tall Vega rocket. 117. Name the organization,which had appointed
(a) THEMIS Aamir Khan as its Brand Ambassador?
(b) Shinsei (a) PayUMoney (b) MobiKwi
(c) Odin (c) PhonePe (d) FreeCharge
(d) PRISMA
118. Scientist from which institute had designed a way
110. Who is the author of the book ‘Every Vote to levitate and propel objects using only light that
Counts’, released by former Vice President of will help to develop a spacecraft?
India, Hamid Ansari in New Delhi? (a) California Institute of Technology (Caltech)
(a) Navin Chawla (b) Arundhati Roy (b) Massachusetts Institute of Technology (MIT)
(c) Vikram Seth (d) Salman Rushdie (c) Harvard University
(d) Imperial College London
111. According to the world airport traffic rankings
2018 by Airports Council International 119. According to its recent discovery of NASA, the
(ACI),which Indian airport became the 12th surface of which asteroid comprises the water and
busiest airport in the world? plumes?
(a) Chhatrapati Shivaji Maharaj International (a) Ceres (b) Bennu
Airport (c) Tholen (d) Itokawa
(b) Indira Gandhi International Airport (IGIA)
(c) Kempegowda International Airport 120. Which Grand Slam tournament had recently
(d) Rajiv Gandhi International Airport increase prize money by eight percent from 2018?
(a) Australian Open
112. Who was appointed as the 24th chief of naval staff (b) Wimbledon Open
of India? (c) US Open
(a) Robin K Dhowan (d) French Open
(b) Devendra Kumar Joshi
(c) Karambir Singh 121. What is the rank of India,according to the United
(d) Nirmal Kumar Verma Nation’s World Happiness Report 2019?
(a) 127 (b) 13
113. Which entity has developed a storage device that (c) 133 (d) 140
translates digital information into the DNA
©LegalEdge Tutorials Page 46 of 76
Replication or other unauthorized use of this material is prohibited by the copyright laws of India
LE Prep Assist
122. Name the Indian rail coach manufacturer,which inorder to bridge temporary liquidity mismatches
became the world’s largest rail coach in the state?
manufacturer surpassing Chinese manufacturers. (a) Ways and Means Advances
(a) Indian Railway Coach Factory,Haldia (b) Short term loans and Advances
(b) Modern Coach Factory, Raebareli (c) Short term repaying loans
(c) Integral Coach Factory (ICF),Chennai (d) Grants against Fixed Deposits
(d) Rail Coach Factory, Kapurthala
131. Who is chosen for Vyas Samman 2018 for his
123. Which telescope of NASA discovered pulsar collection of poems “Jitne Log Utne Prem”?
hurtling through space? (a) Sunita Jain (b) Mamta Kalia
(a) Fermi Gamma-ray Space Telescope (c) Surendra Verma (d) Leeladhar Jagudi
(b) Spitzer Space Telescope
(c) Hubble Space Telescope 132. Karen Uhlenbeck has become the first woman to
(d) Galileo National Telescope win Abel Prize. Abel Prize is referred as noble
prize of which field?
124. Name the Japanese unmanned spacecraft,that (a) Physics (b) Mathematics
probed Asteroid Ryugu and discoverd small (c) Chemistry (d) Poetry
amount of minerals containing water on the
surface of the asteroid? 133. Who has been appointed as the country’s first
(a) Hiten (b) Hayabusa2 Lokpal or the anti-corruption ombudsman by
(c) Akatsuki (d) SELENE President Ram Nath Kovind?
(a) Pinaki Chandra Ghose
125. Name the world’s largest ground-based telescope (b) Jagdish Singh Khehar
developed by pune based IT firm (c) H.J.Kania
ThoughtWorks,that is to become operational in (d) Mehr Chand Mahajan
Hawaii in the mid-2020.
(a) Gran Telescopio Canarias 134. Kassym-Jomart Tokayev was sworn in as
(b) South African Large Telescope (SALT) president of which country?
(c) Large Binocular Telescope (LBT) (a) Azerbaijan (b) Georgia
(d) Thirty Metre Telescope (TMT) (c) Kazakhstan (d) Uzbekistan

126. Where did 5 member team discovered,India’s 135. Name the Mumbai social activist,who was
deepest shaft cave during the 28th edition of the appointed as the first transgender election
annual ‘Caving in the Abode of the Clouds ambassador by the Election Commission of India.
Expedition’? (a) Sathyasri Sharmila (b) Joyita Mondal
(a) Nagaland (b) Maharashtra (c) Prithika Yashini (d) Gauri Sawant
(c) Meghalaya (d) Mizoram
136. As per US based researchers from NASA,which
127. Which country has been stopped from hosting the planet is closer to earth?
Junior Davis Cup and Fed Cup? (a) Uranus (b) Mercury
(a) India (b) Pakistan (c) Jupiter (d) Neptune
(c) Srilanka (d) Mozambique
137. Organisers of the Tokyo 2020 Olympics unveiled a
128. When is the International Day of Nowruz __________shaped torch for the 2020 Olympics?
observed? (a) Cherry-Blossom
(a) 18th March (b) 20th March (b) Trumpet-Shaped
(c) 19th March (d) 21st March (c) Stellate-Shaped
(d) Cruciform-Shaped
129. Which country has renamed its capital as
Nursultan recently? 138. Name the former Supreme Court judge,who
(a) Uzbekistan (b) Kazakhstan passed away at the age of 75 in New Delhi
(c) Tajikistan (d) Kyrgyzstan recently.
(a) K.N. Wanchoo
130. What is the name of the short term loan given by (b) Bijan Kumar Mukherjea
Reserve Bank of India(RBI) to state government (c) Satya Brata Sinha
©LegalEdge Tutorials Page 47 of 76
Replication or other unauthorized use of this material is prohibited by the copyright laws of India
LE Prep Assist
(d) S.M. Sikri (c) Russia (d) Thailand

139. Superfan Naotoshi Yamada often called as ‘Uncle 148. K.T. Irfan became the first Indian athlete to qualify
Olympics’ dies at the age of 92.He belongs to for Tokyo Olympics.He belongs to which sport?
which country? (a) Relay race (b) Hurdles
(a) Thailand (b) Myanmar (c) Racewalking (d) Sprint
(c) China (d) Japan
149. Which Indian city is all set to host the 4th edition
140. Where did External Affairs Minister Sushma of the Internet of Things (IoT) India Congress
Swaraj unveil a plaque of the renovated Indira 2019?
Gandhi Memorial Hospital? (a) Kolkata (b) Bengaluru
(a) New Delhi,India (c) Mumbai (d) Chennai
(b) Male,Maldives
150. Name the country that conduct coordinated
(c) Bengaluru,India
(d) Funadhoo,Maldives operation with India against insurgents posing
threat to the Kaladan multi-modal transit
141. Which country had exempted the Tier 2 (General) transport project.
(a) Bangladesh (b) Nepal
cap of PhD-level work visas?
(a) Russia (b) US (c) Srilanka (d) Myanmar
(c) UK (d) Malaysia 151. Name the project launched by Microsoft in order
to boost India’s Swachh Bharat Mission.
142. Who was sworn in as 13th Chief Minister of Goa (a) Micro-Bharat (b) Swachh
after the demise of Manohar Parrikar? (c) Sangam (d) Micro Boost
(a) Neiphiu Rio (b) N. Biren Singh
(c) Bhupesh Baghel (d) Pramod Sawant 152. President Ram Nath Kovind awarded the ‘Nari
Shakti Puraskar‘ to Seema Mehta. She belongs to
143. Which Indian city is hosting the Africa-India Joint which field?
Field Training Exercise AFINDEX-19 between the (a) Politician (b) Kathak Dancer
Indian Army and 16 African nations? (c) Singer (d) Actress
(a) Pune (b) Bengaluru
153. Which bank had got RBI’s nod to acquire Gruh
(c) Kolkata (d) Chennai Finance?
(a) Yes Bank (b) ICICI Bank
144. Greta Thunberg, who has been nominated for 2019
(c) HDFC Bank (d) Bandhan Bank
Nobel Peace Prize, is an environmental Activist of
which country?
154. Election Commission of India has introduced
(a) Swedan (b) France which app for electoral observers?
(c) Russia (d) Australia
(a) cVIGIL App (b) Observer App
(c) Vote App (d) Elect App
145. What is the name of the Russian spacecraft that
was launched from Kazakhstan’s Baikonur 155. West Nile Virus (WNV), a mosquito-borne
Cosmodrome,which successfully reached disease, mostly reported in the continental United
International Space Station (ISS)? States affected which Indian state recently?
(a) Mercury (b) Vostok (a) Andhra Pradesh (b) Tamil Nadu
(c) Soyuz (d) Shenzhou (c) Karnataka (d) Kerala

146. Which country had became the ICC-full member 156. Which country will host the FIFA U-17 Women’s
in the year 2017 to play official test cricket? World Cup for first time in 2020?
(a) Costa Rica (b) Argentina (a) Srilanka (b) India
(c) Afghanistan (d) Denmark (c) Pakistan (d) Bangladesh

147. Which country has been stopped from hosting the 157. European Union added 10 countries to Tax
Blacklist making it to 15 countries in the list.Which
Junior Asian Wrestling Championship 2019 and
shifted to Thailand by United World Wrestling of the following countries is not Tax Blacklisted?
(a) United Arab Emirates
(UWW)?
(a) India (b) Pakistan (b) Marshall Island

©LegalEdge Tutorials Page 48 of 76


Replication or other unauthorized use of this material is prohibited by the copyright laws of India
LE Prep Assist
(c) Sudan them?
(d) Barbados (a) Uttar Pradesh (b) Meghalaya
(c) Mizoram (d) Andhra Pradesh
158. Which of the following agency released the sixth
Global Environment Outlook 2019? 166. Which country’s central bank released the new
(a) United Nations Environment Programme coin that features the Crimea or Kerch Strait
(UNEP) Bridge,to mark Crimea’s reunification?
(b) United Nations Industrial Development (a) Russia (b) Iceland
Organization (UNIDO) (c) Spain (d) Austria
(c) World Health Organization (WHO)
(d) World Meteorological Organization (WMO) 167. “Sirsi Supari” gets GI tag recently,which is the
first in the areca nut sector.It is cultivated in which
159. Name the country,that made the first shipment of the following state?
under the United Nations ‘Transports (a) Karanataka (b) Andhra Pradesh
Internationaux Routiers’ (TIR) through Iran’s (c) Telangana (d) Kerala
Chabahar Port.
(a) Russia (b) US 168. Name the country which successfully test-fired an
(c) Iran (d) Afghanistan indigenously developed “smart weapon” from a
JF-17 Thunder fighter jet recently.
160. Who is the head of the sub-committee of the (a) Pakistan (b) Russia
Financial Stability and Development Council (c) Iran (d) Iraq
(FSDC) to review issues concerning the economy?
(a) Hemant 169. State Governments got approval from Central
government to use the properties of the people
(b) Ajay Tyagi
(c) Shaktikanta Das who left India for Pakistan and __________ after
(d) Subhash Chandra Khuntia 1965 and 1971 wars?
(a) Srilanka (b) China
161. Who was appointed as mediator for resolving (c) Bangladesh (d) Nepal
various disputes related to cricket administration
170. What is the name of the exercise between India
by Supreme Court?
(a) A.M. Spare (b) P.S.Narasimha and Oman that held in Jabel Al Akhdar Mountains
,Oman?
(c) S.A. Bobde (d) D.K.Jain
(a) Exercise Sarvada Vijay
162. Which Asian country is hosting the Special (b) Exercise Ajeya Warrior IV
Olympics World Summer Games 2019? (c) Exercise Shoor Veer
(a) Phnom Penh,Cambodia (d) Exercise Al Nagah III
(b) Naypyidaw,Myanmar (Burma)
171. Which plane was banned by India after the
(c) Abu Dhabi,United Arab Emirates (UAE)
(d) Amman,Jordan Ethiopian Airlines crash?
(a) Boeing 777 Max 200
163. Supreme Court has lifted the life ban imposed on (b) Boeing 747 Max 400
which of the following cricketer by the BCCI? (c) Boeing 737 Max 8
(a) Ajay Sharma (d) Boeing 742 Max 100
(b) Ajay Jadeja
(c) Sreesanth 172. What is the rank of India in holding gold as per
latest report by the World Gold Council (WGC)?
(d) Mohammad Azharuddin
(a) 1 (b) 4
164. According to Supreme Court’s recent decision,IPS (c) 9 (d) 11
officers who has minimum of __________ month
173. Name the first Thari Hindu woman elected to the
tenure left in service can be considered for the post
of Director General of Police (DGP)? Senate of Pakistan?
(a) Krishna Kumari Kohli
(a) 3 months (b) 6 months
(c) 9 months (d) 12 months (b) Ayesha Rajab Ali
(c) Shahnaz Saleem Malik
165. 12 states banned the usage of e-cigarettes or (d) Musarat Rafique Mahesar
Electronic Nicotine Delivery Systems
(ENDS).Which of the following state is one among 174. Name the multi-wavelength space
observatory,used by astronomers to discover new

©LegalEdge Tutorials Page 49 of 76


Replication or other unauthorized use of this material is prohibited by the copyright laws of India
LE Prep Assist
population of ultra violet stars in the Globular (b) Pokhran, Rajasthan
Cluster NGC 2808? (c) Sriharikota, Andhra Pradesh
(a) EXOSAT (b) AstroSat (d) Bengaluru, Karnataka
(c) ROSAT (d) Granat
182. National Pharmaceutical Pricing Authority
175. Name the new collider to be built near the Large (NPPA) reduced the prices of 390 Anti-Cancer
Hadron Collider (LHC) along the Swiss-French
Non-Scheduled medicines up to 87%.NPPA works
border for understanding of matter and the under which of the following Ministry?
universe, according to recent report by European
(a) Ministry of Consumer Affairs, Food and
Organization for Nuclear Research (CERN). Public Distribution
(a) Sudbury Neutrino Observatory
(b) Ministry of Health and Family Welfare
(b) Laboratori Nazionali del Gran Sasso (LNGS) (c) Ministry of Science and Technology
(c) Large Hadron Collider (d) Ministry of Chemicals and Fertilizers
(d) Future Circular Collider
183. Name the Indian Region,that attained the status of
176. What is the name of the Biopic in Mother ‘Wetland of International Importance’ in the
Teresa,which is to be released on 2020?
Ramsar Convention recently?
(a) Journey of Mother Teresa (a) Tsomoriri (b) Sunderbans
(b) Mother Teresa and her Charity (c) Kanjli (d) Ropar
(c) Mother Teresa and her Simplicity
(d) Mother Teresa: The Saint 184. Which country is banned over doping by
International Association of Athletics Federations
177. When was 89th anniversary of the Dandi
(IAAF) until further notice?
Satyagraha or Salt Satyagraha observed? (a) India (b) Pakistan
(a) 10th March (b) 11th March
(c) Russia (d) South Africa
(c) 12th March (d) 13th March
178. Who was conferred with an Honorary Doctorate 185. Dipa Karmakar, hailing from Tripura had a Barbie
by the University of Peace founded by the United doll modelled after her.She belongs to which
Nations Organisations (UNO) for his contribution sport?
“to the Rule of Law, democracy and sustainable (a) Hockey (b) Tennis
development in India”? (c) Badminton (d) Gymnastics
(a) Narendra Modi
(b) Ram Nath Kovind 186. Which city host the country’s first International
(c) Venkaiah Naidu Beach Volleyball Tournament?
(d) Sushma Swaraj (a) Visakhapatnam, Andhra Pradesh
(b) Thiruvananthapuram, Kerala
179. As per Stockholm International Peace Research (c) Margao, Goa
Institute (SIPRI),which country became the (d) Kochi, Kerala
world’s largest importer of weapons between
187. Name the historic book of CISF,which is released
2014-2018? on the 50th Raising Day of Central Industrial
(a) Saudi Arabi (b) India
Security Forces (CISF) observed on 10th March.
(c) Egypt (d) Australia (a) Achievements of CISF
180. Who received Yaswantrao Chavan National (b) History of CISF
Award,for his contribution towards economic (c) CISF: Force of the Future
development? (d) CISF and its History
(a) Urjit Patel
188. Name the country,where the entire government
(b) Raghuram Rajan has resigned after failing to achieve a key policy
(c) Shaktikanta Das goal on social welfare and healthcare reform.
(d) D. Subbarao (a) Finland (b) Swedan
181. Where did Defence Research and Development (c) Norway (d) Russia
Organisation (DRDO) test fired the Guided Pinaka
189. Who was appointed as the Prime Minister of
weapons system? Palestine?
(a) Chandipur, Odisha
(a) Ahmed Qurei (b) Ismail Haniyeh

©LegalEdge Tutorials Page 50 of 76


Replication or other unauthorized use of this material is prohibited by the copyright laws of India
LE Prep Assist
(c) Salam Fayyad (d) Mohammad Shtayyeh 195. Who was appointed as the Finance Secretary of
India?
190. Which country is planning to release a digital (a) Hasmukh Adhiya
currency called “SOV”(Sovereign)? (b) Subhash Chandra Garg
(a) Marshall Islands
(c) Ashok Lavasa
(b) Papua New Guinea (d) Bimal Jalan
(c) New Zealand
(d) Fiji 196. Name the new communication satellite launched
by China using a Long March – 3B carrier rocket
191. Where did Prime Minister Narendra Modi into a geostationary orbit in Sichuan province?
inaugurate Pandit Deendayal Upadhyaya Institute (a) ChinaSat 6C (b) Gaofen 1
of Archaeology? (c) ChinaSat 9 (d) Dong Fang Hong I
(a) Mumbai, Maharashtra
(b) Greater Noida, Uttar Pradesh 197. Name the country which has signed a deal to lease
(c) Lucknow, Uttar Pradesh Chakra III, a nuclear – powered attack submarine
(d) Jahanpanah, New Delhi for Indian Army worth USD 3 billion for a period
of 10 years.
192. Supreme Court (SC) constituted a 3-member (a) Russia (b) US
mediation panel for the amicable settlement of
(c) Iran (d) China
Ayodhya’s Ram Janmabhoomi-Babri Masjid land
dispute case.Who is appointed as the head of the 198. Two new benches of National Company Law
panel? Tribunal (NCLT) is established in which of the
(a) Sriram Panchu following city?
(b) F. M. Kalifullah i. Amaravati in Andhra Pradesh
(c) Sri Ravi Shankar ii. Chennai in Tamil Nadu
(d) Ranjan Gogoi iii. Indore in Madhya Pradesh
(a) Option i and ii are correct
193. Insolvency and Bankruptcy Board of India (IBBI) (b) Option i and iii are correct
signed an agreement with which of the following
(c) Option i is correct
member of World Bank Group(WBG) for technical (d) Option ii is correct
assistance to IBBI up to June 30, 2021?
(a) International Bank for Reconstruction and 199. Reserve Bank of India(RBI) has eased business
Development (IBRD) guidelines for which of the following ATM’s to
(b) International Development Association (IDA) source cash directly from the central bank?
(c) International Centre for Settlement of (a) White Label ATM
Investment Disputes (ICSID) (b) Green Label ATM
(d) International Finance Corporation (IFC) (c) Yellow Label ATM
(d) Pink label ATM
194. Who won the Chameli Devi Jain Award 2018 for
outstanding women journalist? 200. Who was appointed as the Goodwill Ambassador
(a) Nidhi Razdan (b) Priyanka Dubey of United Nations Development Programme
(c) Neha Dixit (d) Barkha Dutt (UNDP) recently?
(a) Raveena Tondon (b) Padma Lakshmi
(c) Rohit Sharma (d) MS Dhoni

©LegalEdge Tutorials Page 51 of 76


Replication or other unauthorized use of this material is prohibited by the copyright laws of India
LE Prep Assist

ANSWER KEY AND EXPLANATIONS


Prep Test – Legal Aptitude
1. b 2. b 3. d 4. b 5. b 6. d 7. a 8. b 9. a 10. b
11. c 12. a 13. b 14. a 15. d 16. a 17. b 18. a 19. c 20. a
21. b 22. b 23. c 24. b 25. c 26. d 27. d 28. c 29. d 30. b
31. c 32. d 33. d 34. d 35. d 36. c 37. a 38. a 39. d 40. b
41. a 42. c 43. b 44. d 45. b 46. b 47. a 48. c 49. d 50. d

1. As per principle (i), any person who is of the age of Indian Penal Code is or is not in force in the place
majority and of sound mind, may employ an agent. In where the coercion is employed. In the instant case,
the instant case, while Daulat is a major, he is of Dhawal’s actions amount to coercion. Hence, option
unsound mind and thus, may not employ an agent or (a) is the answer.
be a principal. Consequently, a principal agent 8. It is evident from the principle that frivolous litigation
relationship does not exist between Daulat and is the practice of starting or carrying on lawsuits that,
Shramik. Hence, option (b) is the answer. due to their lack of legal merit, have little or no chance
2. As per principles (i) and (ii), Andrew and Zimmer of being won and just because a claim is lost, does not
share a principal agent relationship and Andrew has make a suit frivolous. In the instant case, the fact that
acted in his capacity as an agent as per principle (ii) Srishti lost the suit does not make it frivolous. Hence,
and what is allowed in emergency situations for an option (b) is the answer.
agent as under principle (iii). Hence, option (b) is the 9. The principle clearly states the definition and essential
answer. ingredients of malicious prosecution. In the instant
3. As per the principle, the court may decree restitution case, the defendant (Kaalia) had prosecuted the
of conjugal rights to a petitioner only when it is plaintiff now (Tenali). The proceedings had been
satisfied of the truth of the statements made in a decided in favour of Tenali but had been instituted
petition and that there are no legal grounds why the without a reasonable/justifiable cause and with
application should not be granted. In the instant case, malice, such that Tenali had suffered damage to his
the fact that Seema had left Seetaram after being beaten reputation as a result. Thus, there was malicious
up and threatened with losing her life will play a major prosecution in this case. Hence, option (a) is the
role in determining whether the court will grant answer.
restitution of conjugal rights or not. Hence, option (d) 10. The principle elucidates the meaning and essential
is the answer. ingredients that are attributed to malicious
4. The explanation accompanying the principle clarifies prosecution. In the instant case, the chain of events
that the burden of proving reasonable excuse for suggests that Dilshan did not prosecute Tilakratne
withdrawal from the society will lie on the person who maliciously and had reasons for the same. Moreover,
has withdrawn from the society. In the instant case, Tilakratne did not suffer any damage because of this.
that is Seema, Seetaram’s wife. Hence, option (b) is the Hence, there was no malicious prosecution. Hence,
answer. option (b) is the answer.
5. As per the explanation, it is evident that a person in 11. The principle provides that sale is not effected merely
possession of documents required in the course of by the exposure of goods by a shopkeeper or when the
proceedings will have to produce them in Court. If customer picks them up. In the instant case, Tina
he/she does not do so, he/she may be asked by the cannot be compelled to buy the bracelets just because
Court to do so by means of search/seizure etc. In the she tried them as sale had not been effected yet. She
instant case, the will as well as property documents, can choose to buy whatever she wants or not buy
including the one claimed by Bhiku are necessary in anything at all. Hence, option (c) is the answer.
the case. Hence, option (b) is the answer. 12. The principle makes it clear that if a person causes an
6. As per the principle, it is clear that if the person in effect by means whereby he intended to cause it or
possession of documents required for Court which he knew or had reason to believe would be
proceedings does not produce them in Court, then the likely to cause it, it will amount to an effect caused
Court may issue directions and order the police to “voluntarily”. In the instant case, Balu used the gun
procure them by means of search and seizure. In the knowing full well that it could have grave
instant case, it is obvious that the documents claimed consequences, which it eventually did. Thus, his act
to be with Bhiku, Miku and Brijlal are all instrumental will be said to be voluntary. Hence, option (a) is the
to the case. Hence, option (d) is the answer. answer.
7. As per the principle, coercion is the committing, or 13. As per principle (iii), the seller of goods is deemed to
threatening to commit, any act forbidden by the Indian be an “unpaid seller” when the whole of the price has
Penal Code or the unlawful detaining, or threatening not been paid or tendered. In the instant case also,
to detain, any property, to the prejudice of any person Tehreek is an unpaid seller since he has only received
whatsoever, with the intention of causing any person some token advance payment and not the whole of the
to enter into an agreement. It is immaterial whether the price. Hence, option (b) is the answer.

©LegalEdge Tutorials Page 52 of 76


Replication or other unauthorized use of this material is prohibited by the copyright laws of India
LE Prep Assist
14. Principle (ii) clearly states that if the buyer or his agent instant case also, Kirti can continue to live at Soumitr’s
in that behalf obtains delivery of the goods before their ancestral home. Hence, option (c) is the answer.
arrival at the appointed destination, the transit is at an 20. As per the principle given, it is clear that both Reena
end. In the instant case, however, all that Zohrab (the and Seema were unaware of the fact that the gold
buyer) did was to inform of a change of destination necklace involved in the agreement had been stolen.
while the ship was very much in transit halfway to This amounted to a mistake of fact. Hence, the
Egypt. agreement would be void. Hence, option (a) is the
15. As per principle (i), subject to provisions in the Sale of answer.
Goods Act, when the buyer of the goods becomes 21. The explanation accompanying the principle clearly
insolvent, the unpaid seller who has parted with the states that an erroneous opinion as to the value of the
possession of the goods has the right of stopping them thing which forms the subject-matter of the agreement,
in transit, that is to say, he may resume possession of is not to be deemed a mistake as to a matter of fact. In
the goods as long as they are in the course of transit, the instant case, the fact that Roshan did not know of
and may retain them until payment or tender of the the value of the subject-matter of the agreement, i.e. the
price. The use of the word ‘may’ indicates that it is upto motorcycle, does not make the agreement void as it
the unpaid seller, Tehreek in this case, to retain does not amount to a mistake as to a matter of fact.
possession until full payment in such a situation. Hence, option (b) is the answer.
However, the same is not mandatory. Hence, option 22. As per the principles given, it can be seen that in the
(d) is the answer. instant case, the restraint was not complete and some
16. As per principle (i), the Protection of Women from reasonable conditions had been imposed by the
Domestic Violence Act, 2005 covers women who are or occupier of the NGO, Ruchi. Further, an honest belief
have been in a relationship with the abuser where both to justify the confinement existed. Thus, it does not
parties have lived in a ‘shared household’ and are qualify as false imprisonment. Hence, option (b) is the
related by consanguinity, marriage or through a answer.
relationship in the nature of marriage or adoption. 23. A cursory perusal of the principles and facts would
Even those women who are sisters, widows, mothers, show that Shyamu was restrained completely, without
single women, or living with abuser are entitled to any reasonable condition or honest belief to justify the
legal protection under the Act. In the instant case, Kirti confinement on the part of the Bhatias. Thus, both
was married to the abuser, Soumitr and lived in a statements II and III are correct. Hence, option (c) is the
shared household with him. Thus, she will be entitled answer.
to protection under the Act. Hence, option (a) is the 24. The answer is II as the loss is a purely economic loss.
answer. Hence, option (b) is the answer.
17. A part of principle (i) states that even those women 25. The answer is III as Ronald’s negligent act broke the
who are sisters, widows, mothers, single women, or chain of causation and the damage can be attributed to
living with abuser are entitled to legal protection his negligence. Hence, option (c) is the answer.
under the Act. Thus, Jayshri can sue Soumitr for the 26. Answer is option IV as the explosive device has not
torture he inflicted on her. However, as per principle caused mischief after escaping from the company’s
(ii), the Act does not enable any female relative of the land and therefore the company cannot be held strictly
husband or male partner to file a complaint against the liable in the light of principle 1. Principle 2 is not
wife or the female partner. Thus, Jayshri cannot relevant, as the company was not discharging public
complain against Kirti. Hence, option (b) is the answer. duty.
18. As per principle (iv), “shared household” means a In the light of the principles, choose the most relevant
household where the person aggrieved lives or at any option. Hence, option (d) is the answer.
stage has lived in a domestic relationship either singly 27. The answer is (d). According to the principle, a person
or along with the respondent and includes such a professing of possessing a skill must possess it. The
household whether owned or tenanted either jointly skill professed by Dr. Panparag is that of a doctor,
by the aggrieved person and the respondent, or owned which he possess. There is no guarantee for the success
or tenanted by either of them in respect of which either of a particular treatment. Hence, option (d) is the
the aggrieved person or the respondent or both jointly answer.
or singly have any right, title, interest or equity and 28. Solution: The answer is (c). As a spectator, claimant
includes such a household which may belong to the accepted the risks involved in a horse race he came to
joint family of which the respondent is a member, watch. The spectators were supposed to be sitting
irrespective of whether the respondent or the behind the barriers but he was sitting at on a bench at
aggrieved person has any right, title or interest in the a different place. Hence, option (c) is the answer.
shared household. In the instant case, it is a ‘shared 29. Sheela’s injury was caused by an accident and not by
household’ even though it is Soumitr’s ancestral home. the prior mens rea possessed by Reema. Thus, it does
Hence, option (a) is the answer. not constitute a crime seeing as the act and the guilty
19. Principle (iii) elucidates that the divorced wife would mind occurred at different times. Hence, option (d) is
come within the category of aggrieved person and is the answer.
entitled to live with shared household. Thus, in the 30. The mens rea existed at the time of the kidnapping.
That is, act and guilty mind coincided, which means

©LegalEdge Tutorials Page 53 of 76


Replication or other unauthorized use of this material is prohibited by the copyright laws of India
LE Prep Assist
that the perpetrator is guilty of the crime. Hence, 41. a. Abha knew with reasonable degree of certainty that
option (b) is the answer. her action would result in the unwanted or offensive
31. G was acting under a threat of losing her life, and she contact. So that gives rise to liability. Hence, option (a)
would not otherwise have stolen something from the is the answer.
store. She was not responsible for her situation because 42. Option (c) is correct as according to Principle I, to be a
she did not consent to or ask for the robber to threaten citizen of India one must be ordinarily resident in India
her. Thus, the defence is valid. Hence, option (c) is the and a) was born in the territory of India or b) either of
answer. whose parents were born in the territory of India or c)
32. Ram Singh had intended to convert to the Islamic has been ordinarily a resident in the territory of India
religion for a long time. His second marriage was 5 years immediately preceding commencement.
about 20 years after his conversion. Clearly, it follows Raghu’s Father was born in Thiruvananthapuram and
that his act of conversion was a separate decision, and he was now living ordinarily in India for more than 5
cannot be associated with his desire to marry the years since he has completed his law education. Hence,
second time. Hence, option (c) is the answer. option (c) is the answer.
33. None of the given options are correct. Option (A) and 43. Option (b) is correct. The nature of the law passed is
(B) argue about whether rum based chocolates come such that it will cover all assets accumulated on, before
under the definition of intoxicating materials, which is or after the law comes into existence. There is no ex
not the point of the given question. Option (C) post facto application of law. Hence, option (b) is the
contradicts itself, as it states that a person, who is answer.
incapable of understanding the nature of her act, 44. Option (d) is right. According to the principle a judge
should not be exempted. Hence, option (d) is the acts in judicial capacity only when he is deliberating a
answer. matter at hand. Ill-advised and offhand comments
34. Note that the explanation to the section uses the word made out of court cannot be said to have been made in
"person". Here, Akshay did not have any intention to judicial capacity. Hence, option (d) is the answer.
cause death of a person. Hence, option (d) is the 45. Option (b) is correct. According to the principle, the
answer. right against self-incrimination is not available if a
35. All the above options satisfy the criteria laid down in person is forced to provide physical evidence like
the Principle and the Explanation, which are: lack of blood sample, DNA or fingerprints. In the given fact
intention to cause death, done in good faith, done for situation, the police can force a suspect to provide
the benefit of another, as well as, express or implied fingerprints by force if he refuses to willingly provide
consent of the act with all the associated risks. Hence, it. Hence, option (b) is the answer.
option (d) is the answer. 46. Since the amendment only related to commercial
36. To arrive at the correct answer for this question, one services, passenger services of the railways would still
should check the facts strictly in light of the principles be under the sole control of the Central Government.
and NOT in light of the previous knowledge. The Hence, option (b) is the answer.
exception clearly provides that any act which was done 47. The Act in question is unconstitutional for lack of
with the intention to cause death to the person would legislative competence and accordingly the work in
be considered as an offence. In this case, whatever be this relation done by the Corporation can be removed
her good faith, Sara's mother did remove her by state agencies in accordance with Court orders.
daughter's life support system with the clear intention Hence, option (a) is the answer.
to cause death to her. Therefore, she is liable for the 48. The State Legislation in question was enacted before
same. Hence, option (c) is the answer. the Constitutional Amendment was made. As per the
37. The fact scenario is derived from the American case, stated principle such an Act would be invalid. Hence
Bridges v Hawkesworth. According to the principle, even if the entire subject of railways would have been
the finder of a chattel has the title to the property, moved to the Concurrent list, the Act would still be
which is superior to all, but the owner. Hence, the invalid. Hence, option (c) is the answer.
finder has a claim superior than the owner of the 49. The principle states that only when a fresh legislation
premises. Hence, option (a) is the answer. is enacted after an amendment will it become
38. This is a direct application of the principle given. Since consonant with the Constitution. A mere amendment
the court must look at the relevant custom and usage will not affect its invalidity. Hence, option (d) is the
of the industry, option (A) is the most suitable answer. answer.
Anything suggesting otherwise is not relevant for this 50. The legislation passed by the Bihar state legislature
question. Hence, option (a) is the answer. was enacted after the Constitutional amendment. Mere
39. The damage caused to Saj was not direct. It was remote similarity with the Gujarat state law would not make it
damage. Hence, option (d) is the answer. invalid and the Bihar state Act would be
40. Sasha had the intent because he thought it would be constitutionally valid. Hence, option (d) is the answer.
funny. Hence, the battery suit will succeed. Hence,
option (b) is the answer.

©LegalEdge Tutorials Page 54 of 76


Replication or other unauthorized use of this material is prohibited by the copyright laws of India
LE Prep Assist

ANSWER KEY AND EXPLANATIONS


Prep Test – Mathematics

1. d 2. b 3. a 4. d 5. b 6. a 7. a 8. d 9. d 10. b
11. a 12. d 13. a 14. c 15. b 16. b 17. a 18. d 19. a 20. d

1x 1 1 1 1 1
× y ∶ x × y ∶ (1 − − )x × 1 y
6 6 3 3 6 3
1. Put 𝑥 = 0, 0 − 4𝑦 = 8 1 1 1
⇒ 𝑦 = −2 ∶ ∶
36 9 2
And 1 : 4 : 18
Put 𝑦 = 0𝑥 − 0 = 8 4
Profit share of B = 4600 ×
𝑥=8 23
So it gives two point (0, −2) and (8, 0) = 800
Hence, the answer is (d). Hence, the answer is (b).
(60−48) 11. Let C.P. be Rs.100.
2. Stoppage time per hour × 60 minutes = 12
60 So, marked price = Rs. 125
minutes. 84
Then, selling price = Rs. ( × 125) = Rs.105
Hence, the answer is (b). 100
2 (105 – 100)
3.
1 1
d1 = 4√2 cm. ⇒ area = d1 2 = × (4√2) = 16 cm2 Hence, Profit % = × 100 = 5%.
100
2 2
Area of new square = (2 × 16) cm2 = 32 cm2. Hence, the answer is (a).
1 12. Number of selections = Number of ways of selecting 2
∴ d2 2 = 32 ⇒ d2 2 = 64 ⇒ d2 = 8 cm.
2 men out of 5 men × number of ways of selecting 1
Hence, the answer is (a). woman out of 3 women.
4. When 10,000 is divided by 19, the reminder is 6. = 5𝑐2 × 3𝑐1
Therefore, 10,000 − 19 = 9981 is the largest 4 digit number 5×4
which leaves a remainder of 6 when divided by 19. = × 3 = 30
1×2
Hence, the answer is (d). Hence, the answer is (d).
5. A number divisible by 2 and 3 both will be divisible by 13. A = 1⁄2 × 3600 = 1800
the L.C.M. of 2 and 3 = 6. Quotient obtained when 200 is
B = 1⁄3 × 3600 = 1200
divided by the L.C.M. of 2 and 3, i.e., 6 is 33. Hence, the
answer is (b). C = 1⁄12 × 3600 = 300
6. Explanations: Remaining apples after C gets his share is 3600 (1800
Let the total income be 100.
1200 300) = 300
Then, income left = (100 − 80)% of [100 − (35 + 25)] = 20%
A gives 1⁄5 × 3600 = 720 apples
of 40
20 Therefore total apples remaining = 720 300 = 1020.
=( × 40)= 8
100
A now has = 1800 – 720 = 1080
Hence, the answer is (a).
Hence, the answer is (a).
7. Let the average in his first 9 innings be x,
14. Since, there are only two types of socks in the bag. so if
Total runs in first 9 innings = 9x
murari picks up 3 socks, then certainly two of them are
The average after the 10 innings = x + 4
of same type. thus, this is a certain event. hence,
Total runs after 10 innings = 10 (x + 4)
required probability = 1
Score in the 10th innings = 50
Hence, the answer is (c).
10 (x + 4) − 9x = 50
15. Since <OAP = 90°
10x − 9x + 40 = 50
𝐴𝑃 2 + 𝐴𝑂 2 = 𝑂𝑃 2 → 𝐴𝑃 2 = 𝑂𝑃 2 − 𝑂𝐴2
x = 10
= 52 − 32 = 16
Therefore, the new average is 10 + 4 = 14.
= 𝐴𝑃 = 4𝑐𝑚.
Hence, the answer is (a). Hence, the answer is (b).
8. The tank gets filled at 50 litres per minute and emptied 16.
at 30 litres per minute. Therefore, every minute 20 litres 3−7+10 5−4+2
The coordinates of Centroid are ( , ) = (2, 1)
is in the tank. Therefore, in 40 minutes, 20 × 40 = 800 3 3
litres. Hence, the answer is (b).
Hence, the answer is (d). 17. For first year S.I. = C.I.
9. Let initially there be x litres of milk Now 16 is the S.I. or S.I. for 1 year.
X − 10 2 25 10 is S.I. on Rs.100.
Now, ( ) = . ∴Rs.16 is S.I. on (100/10)×16 = 160.
X 25 + 24
X − 10 5
⇒ = ⇒ 7x – 70 = 5x ⇒ x = 35. So, S.I. on principal for 1 year at 10% is 160.
X 7
Hence, the answer is (d). ∴ Principal = 100 × 16010 × 1 = 1,600.
10. Let x be the total amount & y be the total time period Amount for 2 years compounded half yearly = 1600 ×
investment (1+5/100)power of 4 = 1,944.81.
Ratio of the profit for A, B, & C is ∴ C.I. = (1,944.81 - 1,600) = 344.81.

©LegalEdge Tutorials Page 55 of 76


Replication or other unauthorized use of this material is prohibited by the copyright laws of India
LE Prep Assist
S.I. = 1600 × 5 × 4/100 = 320. 20. The product of four consecutive numbers is always an
∴ (C.I. – S.I.) = Rs. (344.81 – 320) = 24.81. even number.
Hence, the answer is (a). ∴ n = 1 + x is an odd number
18. Proportion of milk in 3 samples is 3⁄4 , 3⁄5, 5⁄6. Let the four consecutive number be k – 1, k, k + 1, k + 2.
∴ n = 1 + (k - 1)k (k + 1) (k + 2)
Proportion of water in 3 samples is 1⁄4, 2⁄5, 1⁄6.
= 1 + (k - 1) (k + 2)k (k +1)
Since equal quantities are taken.
= 1 + (k 2 + k − 2)(k 2 + k)
Total proportion of milk is 3⁄4 + 3⁄5 + 5⁄6= 131⁄60 Let k 2 + k = x
Total proportion of water is 1⁄4 + 2⁄5 + 1⁄6 = 49⁄60 ∴ n = 1 + (x − 2)(x) = x 2 − 2x + 1
Proportion of milk and water in the solution is 13 1 : 49. = (x − 1)2 = (k 2 + k − 1)2
Hence, the answer is (d). ∴ 𝑛 = 𝑥 + 1 is definitely a perfect square
19. He will gain maximum profit when he use 700g weight. Hence, the answer is (d).
300
ie, = × 100 = 42. 85%
700
Hence, the answer is (a).

©LegalEdge Tutorials Page 56 of 76


Replication or other unauthorized use of this material is prohibited by the copyright laws of India
LE Prep Assist

1. c 2. d 3. c 4. c 5. c 6. d 7. b 8. a 9. a 10. a
11. b 12. c 13. b 14. d 15. c 16. b 17. d 18. a 19. b 20. a
21. b 22. d 23. b 24. b 25. d 26. d 27. c 28. d 29. c 30. a
31. a 32. d 33. b 34. c 35. c 36. a 37. b 38. b 39. c 40. d
41. b 42. a 43. d 44. b 45. c 46. b 47. a 48. c 49. b 50. c
51. d 52. c 53. d 54. a 55. c 56. b 57. c 58. d 59. a 60. c
61. d 62. a 63. d 64. b 65. c 66. b 67. a 68. d 69. b 70. d
71. a 72. d 73. d 74. c 75. d 76. b 77. a 78. d 79. c 80. c
81. d 82. d 83. c 84. c 85. b 86. c 87. a 88. b 89. b 90. c
91. c 92. c 93. a 94. d 95. c 96. b 97. d 98. a 99. c 100. b
101. a 102. d 103. a 104. b 105. b 106. c 107. a 108. d 109. d 110. a
111. b 112. c 113. d 114. a 115. c 116. a 117. c 118. a 119. b 120. d
121. d 122. c 123. a 124. b 125. d 126. c 127. a 128. d 129. b 130. a
131. d 132. b 133. a 134. c 135. d 136. b 137. a 138. c 139. d 140. b
141. c 142. d 143. a 144. a 145. c 146. c 147. a 148. c 149. b 150. d
151. c 152. b 153. d 154. b 155. d 156. b 157. c 158. a 159. d 160. c
161. b 162. c 163. c 164. b 165. a 166. a 167. a 168. a 169. b 170. d
171. c 172. d 173. a 174. b 175. d 176. d 177. c 178. c 179. a 180. b
181. b 182. d 183. b 184. c 185. d 186. a 187. c 188. a 189. d 190. a
191. b 192. b 193. d 194. b 195. b 196. a 197. a 198. b 199. a 200. b

1. On 25th March, NITI Aayog has organized a FinTech National Exhibition of Art, scheduled from March 25 to
Conclave at Dr. Ambedkar International Center, New April 8 at the National Gallery of Modern Art (NGMA)
Delhi. The Conclave was inaugurated by RBI Governor, and Sir JJ School of Art in mumbai.Click here for winner
Shaktikanta Das. The motive of the conference was to list.
shape India’s continued ascendancy in FinTech, build the 4. On 24th March, 2019, K. Govindaraj has been
narrative for future procedure and policy efforts, and to unanimously re-elected as president of the Basketball
deliberate steps for complete financial inclusion.The Federation of India (BFI) at its annual general-body
Conclave has attributed 300 representatives from the meeting.Govindaraj announced that a new six-team
financial space including central ministries, regulators, women’s league will be held in next three months.He has
bankers, startups, service providers and declared to give proposal on restructure the National
entrepreneurs.Discussions were held on the themes such championships in the next 45 days and intend to have a
as Digital Onboarding of Customers/Merchants for zone-wise competition before the final round.
Financial Inclusion, Building Financial Products for 5. US-based e-commerce platform eBay has partnered with
Millennial India, Emerging Areas of FinTech, Fast Google to offer Google Pay as a form of payment option
Tracking Investments in FinTech Industry and Financial in its new “managed payments” plan. The collaboration
Inclusion of MSMEs. will enhance customer experience tailored to their
2. Explanation:The top three performers are Sweden, personal preferences. The partnership will enable
followed by Switzerland and Norway.In the second customers with Google Pay accounts to check out
edition of the Energy Transition Index, India improved without leaving the platform. But it will only be available
its rank by two points from last year (2018) higher than on products sold by sellers who are participating in the
China (82nd).Despite the fact that India has high managed payments programme.Earlier, eBay added
pollution levels and high CO2 intensity amongst these Apple Pay as a payment method under its managed
115 countries, it has made a significant improvement in payments programme.
its energy access and regulations in recent years.In the 6. The 2nd edition of Badminton Asia Mixed Team
BRICS block of emerging economies, After Brazil (rank Championships aka Tong Yun Kai Cup 2019 was held in
46th) India ranked second best. Among the major Queen Elizabeth Stadium, Hong Kong. The
economies, UK ranked 7th, Singapore ranked 13th, championship was organized by Badminton Asia and
Germany ranked 17th, Japan ranked 18th and the US Hong Kong Badminton Association. China has grabbed
ranked 27th in this index.India scored low in terms of the championship by defeating Japan with 3-2. In the
system performance (ranking 97 and 86, respectively), mixed doubles, Chinese pair He Jiting and Du Yue has
but it ranks considerably higher in terms of readiness to defeated apanese pair Yuta Watanabe and Arisa
adapt to future energy needs (45 and 61 respectively). Higashino by 21-17, 21-17.In the men’s doubles, Yugo
3. Explanation:On 22nd March, Lalit Kala Akademi (LKA) Kobayashi and Takuro Hoki of China beat Han Chengkai
announced 15 winners of the 60th Annual Academy and Zhou Haodong of Japan by 16-21, 21-18, 21-13.In the
Awards. All the awardees will be honoured with a women’s doubles, Li Yinhui and Du Yue won for China
plaque, a shawl and prize money of Rs 1 lakh at the 60th

©LegalEdge Tutorials Page 57 of 76


Replication or other unauthorized use of this material is prohibited by the copyright laws of India
LE Prep Assist
by beating Ayako Sakuramoto and Yukiko Takahata World Tuberculosis Day 2019 was ‘Its time’ which drew
with 21-16, 21-19. the attention of the global leaders towards the immediate
7. On 24th March renowned South Asian singer, Shahnaz need to end the global epidemic tuberculosis.India
Rahmatullah Popularly known as Shahnaz Begum (aged achieves 84% fall in TB deaths among HIV affected
67 years), passed away at her Baridhara residence Dhaka, patients by 2017. The global efforts to combat TB was
Bangladesh after suffering a heart attack. She was born successful by saving around 54 million lives since 2000.
on January 2, 1952, in Dhaka, Bangladesh. She was It reduced the mortality rate by 42%.Centre and State
awarded with National Film Awards (best female singer) Government working together to make India TB-free by
in 1990 and Ekushey Padak the second highest civilian 2025,which is a five year ahead of the global target.
award in Bangladesh in 1992. 14. International Day for the Right to the Truth Concerning
8. On 25th March 2019, International Day of Remembrance Gross Human Rights Violations and for the Dignity of
of the Victims of Slavery and the Transatlantic Slave Victims observed on 24th March every year to Honor the
Trade was observed all over the world to raise awareness memory of victims of gross and systematic human rights
about the dangers of racism and to honour and to violations and to promote the importance of justice.The
remember people who suffered for the slavery system. day was recognized by United Nations General
The theme for International Day of Remembrance of the Assembly on 21 December 2010.
Victims of Slavery and the Transatlantic Slave Trade 2019 15. India extended a financial grant of 35.5 million Nepalese
was “Remember Slavery: The Power of the Arts for rupees to Nepal for the construction of an educational
Justice”.The day was first observed in 2008, which was campus to boost the learning environment of the
recognized by United Nations General Assembly students. The educational campus was jointly
(UNGA) after adopting a resolution in 2007. inaugurated by District Coordination Committee and
9. On 23rd March 2019, the Central Government banned Campus management committee on 22nd March 2019. It
Jammu Kashmir Liberation Front (JKLF) under the was made under the Development Partnership
provisions of Unlawful Activities Prevention Act Programme which is administered by the Development
(UAPA), 1967 as an unlawful association.Yasin Malik Administration Partnership division of the Ministry of
lead JKLF is creating a separatist ideology in the Kashmir External Affairs. The building was constructed at
Valley. It has been doing its separatist activities and Siddheshwar Education Public Campus in Ramechhap
violence since 1988.In accordance with the policy of zero district. It consists of 12 class rooms, meeting hall,
tolerance against terrorism the Centre decided to ban laboratory room and a library.
JKLF. It has been creating hatred against the government 16. On 22nd March 2019, the forest certification scheme
as well as the armed rebellion.UAPA aims to prevent ‘Certification Standard for Sustainable Forest
unlawful activities association in India and to provide Management (SFM)’ developed by the Network for
necessary powers to the investigating agencies to act Certification and Conservation of Forest (NCCF) got
against activities directed against the nation. global recognition by a Geneva based non-profit firm.
10. On 23rd March 2019, The 64th Filmfare Awards 2019 was The decision was made through a postal ballot by the
held at Jio Garden, BKC, Mumbai. The event was hosted council of Programme for Endorsement of Forest
by Bollywood’s leading Actors, Directors, Musicians and Certification (PEFC) which is an international non-profit
Singers. The winners of the 64th Filmfare Awards are as firm that provides independent third party certification
below: for Sustainable Forest Management. The Government
11. On 17th March 2019, the 2019 Lingshui Masters has emphasized the importance of buying products
Badminton Tournament held in Agile Stadium in China. made from certified wood for promoting sustainable
Weng Hongyang won the Men’s Singles title in forest management under the Green Good Deeds
tournament.This was the first Super 100 Tournament of Movement. iv.Sustainable Forest Management
the 2019 BWF World Tour.The tournament was Certificates become mandatory for exports as several
organized by the Chinese Badminton Association and developed countries have put trade restrictions on
sanctionedby the Badminton World Federation import of non-timber forest products, non-certified
(BWF).The Super 100 Tournament commenced on 12th timber and wood based goods into their countries.
March with a total prize money of USD 75000. 17. Rear Admiral Sanjay Jasjit Singh, NM took over as the
12. On 24th March 2019, Suresh Raina, a batsman of Chennai Western Fleet Commander, the “Sword Arm” of the
Super Kings became the first batsman to make a record Indian Navy. He will succeed Rear Admiral MA
by surpassing 5,000 runs in the Indian Premier League in Hampiholi, AVSM, NM who has been appointed as
Chennai. He scored 5000 runs from 177 matches at a Director General Naval Operations in the rank of Vice
strike-rate of 138.15 consisting 35 fifties and one Admiral.Initiated his career in 1986, the Navigation and
hundred.He is also the only batsman to breach the 300- Direction specialist, Sanjay Jasjit Singh is an alumnus of
run mark in all 11 seasons. the National Defence Academy, Khadakwasla.
13. On 24th March 2019, World Tuberculosis Day was
observed to raise public awareness about the devastating 18. On 22nd March 2019, the Amateur International Boxing
health, social and economic consequences of Federation (AIBA) President Gafur Rahimov announced
tuberculosis.World Health Organization (WHO) and the his resignation claiming that political based discussions
Global Fund and Stop TB launched a joint initiative ‘Find. were harming the organizations progress.Gafur
Treat. All. #End TB’ on the World TB day. The theme for Rahimov was elected as the President of AIBA four

©LegalEdge Tutorials Page 58 of 76


Replication or other unauthorized use of this material is prohibited by the copyright laws of India
LE Prep Assist
months ago. Boxing was contested in every Summer National Stock Exchange (NSE) which provides
Olympics except in 1992 Olympics which was held at technology solutions across industries such as financial
Stockholm.Rahimov will be replaced by an interim markets, capital markets, banking and insurance.
president. The debt of AIBA is $15.6 million for this year 25. On 20th March 2019, Xiaomi launched Mi Pay, a UPI
when compared to $18 million in last year.International (Unified Payments Interface) Based mobile payment
Olympic Committee (IOC) executive board was planning service in India to send money from one bank account to
for the Olympic Boxing Tournament of next year which another or between UPI (Unified Payments Interface)
had been frozen and contact suspended between Tokyo accounts.The service provider of this application is ICICI
2020 organizing committee and AIBA. Bank.It was certified by National Payments Corporation
19. On 20th March 2019, Indian Institute of Technology of India (NPCI).A customer can pay various types of
Madras has partnered with ESPNcricinfo to launch bills, make DTH recharge with the help of this
Superstats which is a next level artificial intelligence to application.
analyse cricket statistics through machine learning and 26. On 22 March 2019, a new application named ‘Shiksha
forecast. Superstats is developed by a team headed by the Vani’ was launched by Central Board of Secondary
IIT Madras research scholars Raghunathan, Education (CBSE) to publicize crucial information to
Rengaswamy, Mahesh Panchagnula along with the students and parents in a timely manner”. As a part of it,
ESPN team.It will analyse cricket using data science for the app has already uploaded the circular regarding the
the first time, give context to every event in a game and evaluation procedure for the 2019 Board Examinations.
also venture new territories such as forecasting and 27. An Artificial Intelligence (AI) and block chain based
luck.It also reveals the trends and patterns during a game software products firm Intain Fintech will launch eMulya
as the actual match data spread on real time. It has by the end of 2019.eMulya is an intelligence blockchain
combined big data, cricket intelligence and data science which facilitates secure, efficient and trustworthy asset
to produce a set of numbers which will help the fans to securitization transactions for Non-Banking Financial
understand and appreciate the game better. Companies (NBFCs) in India.eMulya will utilize AI to
20. The 35 years old, limited over’s captain of Sri Lanka, onboard authenticated loans and asset data to the
Lasith Malinga, is all set to be retired from international platform and allows investors originators issuers rating
cricket after ICC Men’s T20 World Cup 2020, to be held agencies and services to manage the securitization life
in Australia over October-November 2020. He would cycle in complete trust and transparency.Siddhartha S is
also quit one-day internationals after the 2019 World Cup the founder and CEO of Intain and it has a global
in England and Wales.He made his ODI debut in 2004, Artificial Intelligence lab in Pune.
becoming the first to clinch two World Cup hat-tricks 28. On 22nd March 2019, India clinched South Asian Football
and the first to take three ODI hat-tricks.He is also the Federation (SAFF) Women’s Championship 2019 by
only bowler to take four wickets in four consecutive balls. defeating Nepal for 3-1 in Biratnagar, Nepal. It was for
21. World Meteorological Day is observed annually on the fifth time in a row that India won the championship.
March 23 to commemorate the establishment of World Indian Midfielder Indumathi Kathiresan and Nepali
Meteorological Organization (WMO) in 1950. The day forward player Sabitra Bhandari became the top scorers
also underlines the efforts of National Meteorological in the match with four goals each.The competition was
and Hydrological Services for the safety and well-being scheduled to host in Sri Lanka in December 2018 which
of society. The theme 2019 was “The Sun, the Earth and was rescheduled as the former withdrew. Afterwards
the Weather”.The day was first observed in 1961. Nepal hosted the competition in March 2019.A total of six
22. The “100 Most Influential People in Climate Policy” list teams participated in the competition which were India,
is comprised of seven Indian names including Union Nepal, Bangladesh, Maldives, Sri Lanka and Bhutan.
Ministers Piyush Goyal and Dr Harsh Vardhan, who Afghanistan and Pakistan withdrews participation in the
took important measures to combat climate change. fifth edition of SAFF Women’s championship 2019.Since
Other5winners India incepted the championship in 2010, it is India’s
23. Ford Motor Co named Amazon.com Inc veteran Tim 23rd straight victory. Dalmia Chhibber of India scored
Stone as chief financial officer, the automaker’s first first goal of the match.
external hire for the role in seven decades. Stone, 52, 29. World football governing body FIFA has imposed a
replaces Bob Shanks, who will retire at the end of 2019 lifetime ban on 72 years old, former Ecuadorian Football
after serving the company for 42 years. Stone held Federation (FEF) President, Luis Chiriboga. He has also
various financial roles at Amazon for two decades, and fined with million swiss francs ($1.01 million). Luis
was most recently CFO of Snapchat messaging app Chiribogai. He had breached the article 27 (bribery) of the
owner, Snap Inc. 2018 edition of the FIFA Code of Ethics by accepting
24. On 22nd March 2019, a wholly owned Subsidiary of bribes in exchange for awarding lucrative media and
National Stock Exchange of India, NSEIT decided to marketing rights for tournaments.He was also the former
acquire a global cyber security company Aujas Networks member of FIFA’s standing committee and the South
for an undisclosed sum.It will help Aujas to improve its American Football Confederation’s (CONMEBOL)
services and capabilities with a strong focus on helping executive committee.
customers with security challenges. Aujas which is 30. The European T20 competition set to take place in
founded in 2008 is operational in the Middle East, North Ireland, Scotland, and the Netherlands for the first time
America and India.NSEIT is the IT focused Subsidiary of later this year has been officially named as ‘Euro T20

©LegalEdge Tutorials Page 59 of 76


Replication or other unauthorized use of this material is prohibited by the copyright laws of India
LE Prep Assist
Slam’. The competition which was unveiled earlier in Darussalam, Japan, South Korea, Russia, New Zealand
2019, will consist of six-city based franchises, two from and China apart from 10 member ASEAN countries.The
each of the host nations. objective behind this meet was to urge all participating
31. Renowned Indian artist Haku Shah, who was known for governments to talk openly about their plans for
his tribal and folk art has passed away due to cardiac increasing cooperation and for building trust in the
arrest at the age of 85 years. He hailed from Ahmedabad, Pacific Ocean and Indian Ocean region. The Indo-Pacific
Gujarat.He established a tribal museum at Gujarat region is comprised of three-fifth of the world’s
Vidyapith in Ahmedabad.In 1980, he also set up the first population with gross domestic product (GDP)
of its kind Crafts Village – Shilpgram in Udaipur.His amounted to US$52 trillion.
memoir Manush received several awards including the 37. To expand the Renewable Energy Capacity in India,
Padma Shri in 1989. Asian Development Bank (ADB) signed a loan
32. On 20th March 2019, ‘The Great Disappointment: How agreement of USD50 million with India’s renewable
Narendra Modi Squandered A Unique Opportunity to energy producer company Avaada Energy Private
Transform the Indian Economy’ book released by Limited (AEPL). It was signed by Avaada Energy
economic commentator and Congress member Salman chairman Vineet Mittal and ADB Principal Investment
Anees Soz. This book is the evaluation of P.M Modi’s Specialist Mayank Choudhary.This agreement will help
impact on Indian economy in 5 years.The book has been in Government’s strategy to give a boost in the share of
published by Penguin Random House India. renewable energy generation capacity from about 20% in
33. On 21st March 2019, The Indian English word ‘chuddies’ 2018 to 40% by 2030, and to reduce India’s emission
recognized as official English word by the Oxford intensity of its gross domestic product (GDP) by 33% to
English Dictionary (OED). The word is one of the new 35% by 2030.The investment will be done through ADB’s
650 entries announced by the OED.The contribution of Ordinary Capital Resources and Leading Asia’s Private
word “chuddies” to the English vocabulary was made by Infrastructure Fund (LEAP), a funding arm of Japan
a show which was aired on BBC in the mid-1990s named International Cooperation Agency (JICA) administered
‘Goodness Gracious Me’. The show received the gold by ADB.
award for comedy in 1997.The word underpants or 38. On 19th March 2019, Brazilian physicist and astronomer
means short trousers or shorts. It qualifies as a regional Marcelo Gleiser won the 2019 Templeton Prize, worth
vocabulary because it is a mixture of English and Hindi $1.4m (Rupees 9,61,20,500), for combining science and
which is an example of Queen’s Hinglish. spirituality in his work. 2019 Templeton Prize – Marcelo
34. Atal Mission for Rejuvenation and Urban Gleiser.He is the first Latin American to win the award,
Transformation (AMRUT) is an initiative taken by which is given for exceptional contribution for directing
Ministry of Housing and Urban affairs to focus on water life’s spiritual dimension.The prize started by late global
supply, sewerage facilities, storm water drains to reduce investor Sir John Templeton in 1972.In the year 2018, it
flooding, creating and upgrading green spaces and other was won by King Abdullah II of Jordan.
facilities in the urban areas.Theme for World Water Day 39. On 20th March 2019, Chennai City FC crowned as the l–
2019 is “Leaving no one behind”.In the year 1993, it was League Champion by All India Football Federation
first observed by United Nations General Assembly. (AIFF) in Chennai. The I-League aka Hero I-League is the
35. The 33rd edition of the India-Indonesia Coordinated men’s top most professional football league in India. The
Patrol (Ind-Indo Corpat) was started on March 19, 2019 owner of Chennai City FC is Rohit Ramesh.
and will continue till 4th April 2019 at Port Blair, 40. On 20th March 2019, former India hockey captain Sardar
Andaman & Nicobar Islands in order to enhance Singh along with 13 Indian Olympic Association (IOA)
interoperability and strengthen existing bonds of members, have been named in various Standing
friendship between India and Indonesia.The closing Committees of the Olympic Council of Asia (OCA) at its
ceremony will be held at Belawan, Indonesia. Under this 38th General Assembly which held in Bangkok.Sardar
both countries’ ship and aircraft would undertake Singh has been appointed in the athletes standing
patrolling of 236 nautical miles long International committee, Rajeev Mehta who is IOA’s secretary general
Maritime Boundary line on the respective sides.The appointed in the culture standing committee, Hockey
Indian Navy, Commodore Ashutosh Ridhorkar, VSM India secretary general Mohd Mustaque Ahmad is in the
Naval Component Commander, Andaman and Nicobar media standing committee, Athletics Federation of India
Command are led the Indian delegation.The Indonesian (AFI) president Adille Sumariwala and Lalit Bhanot have
side led by Cmde Dafit Santoso.It is also a part of the been appointed in the sports for environment and sports
Indian Government’s vision of SAGAR (Security and standing committees respectively for four-year term
Growth for All in the Region), for assisting countries in (2019 to 202(c).
the Indian Ocean Region. 41. Japanese unmanned spacecraft,Hayabusa2 reached
36. The Government of Indonesia has hosted the first ever Asteriod Ryugu on June,2018.Since then,the spacecraft
High-Level Dialogue on Indo-Pacific Cooperation (HLD- landed multiple robotic probes on its rocky terrain of the
IPC) in its capital city Jakarta. The theme of the HLD was Asteroid Ryugu.The spacecraft had surveyed 69,000
“Towards a Peaceful, Prosperous, and Inclusive Region”. locations covering almost 90 percent of Asteroid Ryugu
It was inaugurated by Indonesia’s Vice President Jusuf by using a near-infrared spectrometer capable of
Kalla. The Dialogue saw the participation of detecting hydrated minerals.According to its study
representatives from India, Australia, US, Brunei published in U.S. Journal Science,the spacecraft probed

©LegalEdge Tutorials Page 60 of 76


Replication or other unauthorized use of this material is prohibited by the copyright laws of India
LE Prep Assist
and found the small amounts of minerals containing commemorates the Sharpeville massacre on 21 March
water on the surface of the asteroid.A part of the water 1960, which involved killing of 69 people by police, at a
might be from asteroids and comets. peaceful demonstration against the Apartheid regime (a
42. Google unveiled Stadia, a service that will let people play regime which embraced racial discrimination) in
video games without the need for dedicated consoles or Sharpeville, South Africa.The UN General Assembly
high-end computers. Instead, gamers will be to play on proclaimed the day as a UN Day of observance in 1966.
standard laptops, tablets or phones, with all the 47. World Down Syndrome Day is observed annually on
heavyweight processing performed on Google’s servers. March 21 to raise awareness about Down syndrome, a
The platform will let users immediately play any game congenital disorder caused by having an extra 21st
that is available on the service, without the need to chromosome. This year the focus of the day was on
purchase it individually or download a copy to their “Leave no one behind” by creating opportunities for
device. “Stadia offers instant access to play,”.Stadia is a people with Down syndrome to live fulfilling lives.The
cloud gaming service, meaning that processing and day was first recognized by World Health Organization
graphics rendering will be performed at Google data (WHO) on March 21, 2007 then from 2012 onwards, UN
centres. As a player moves around in a game, video of General Assembly decided to designate 21 March as
the gameplay will be streamed to their device. Providing World Down Syndrome Day.
the internet connection is good enough, this will happen 48. World Poetry Day observed on 21st March 2019 to
seamlessly. encourage the reading, writing, and teaching of poetry
43. The T10 cricket League is set to be played in Zayed throughout the world.In the year 1999, UNESCO, in its
Cricket Stadium of Abu Dhabi for the next five years, 30th General Conference in Paris, has declared this day
starting from 2019 season. The first match in the T10 to support linguistic diversity and to safeguard
League will be held on 23rd October 2019. Abu Dhabi endangered languages.
Cricket (ADU) partnered with the Department of Culture 49. International Day of Forests(IDF) 2019 was observed on
and Tourism, Abu Dhabi (DCT Abu Dhabi) and Abu 21st of March 2019 all over the world. Theme for
Dhabi Sports Council (ADSC) has entered into a five year International Day of Forests: “Forests and Education –
agreement with organizers of the T10 league for 5 Learn to Love Forests”. It has been organized since 2013
years.T10 is the shortest form of cricket which was first by the United Nations General Assembly with an aim to
played in UAE in 2017. raise awareness and educate people about the
44. The 15th Special Olympics World Summer Games 2019 importance of forest for maintaining the balance of the
for athletes with Intellectual disabilities held in Abu ecosystem.
Dhabi, United Arab Emirates with motto “Meet the 50. On 19 March 2019, the annual Economist Intelligence
Determined”. The event saw the participation of 190 Unit (EIU) survey listed Singapore, Paris and Hong Kong
countries. The next Special Olympic Games are set to take as the world’s most expensive cities. In the 30 year long
place in Sweden in 2021.India wins 368 medals at the history it is for the first time the three cities are sharing
Special Olympics World Summer Games including 85 the top spot.Zurich and Geneva in Switzerland ranked
gold, 154 silver and 129 bronze. Jitendra Pawal won a fourth and fifth respectively in the Expensive Cities list
gold medal in the 200m race.Medals are won in Judo, while Osaka in Japan ranked 6th position.As per EIU,
Athletics, Cycling, Aquatics, Powerlifting, Basketball Bengaluru, Chennai and New Delhi ranked 5th, 7th and
Traditional, Table Tennis, Roller skating, Badminton, 8th position respectively among the top ten cheapest
Handball Traditional and Football 7 Side Female cities.Caracas in Venezuela topped the cheapest cities in
competitions in the game.Out of 24 Olympic type sports the world list. Damascus in Syria 2nd followed by
which featured the World Cup, India participated in 14 Tashkent in Uzbekistan and Almaty in Kazakhstan.The
types. It is for the 9th time India has participated in the survey will help the companies to calculate cost of living
Special Olympics.India participated in Judo and Futsal allowances and build compensation packages for
for the first time. business travelers and expatriates.
51. 4th Round of Free Trade Agreement (FTA) talks between
45. The 1971 India-Pakistan war hero, Captain Mohan India and Peru concluded in Lima on 15th March.The
Narayan Rao Samant of the Indian Navy, passed away at FTA agreement is aimed to enhance the two-way
the Bharatiya Arogya Nidhi Hospital in Juhu at the age commerce between India and Peru.The Bilateral trade
of 89 years due to cardiac arrest. He was a native of Pune between India and Peru has been increased from 1.77
(Maharashtra).He was an awardee of Maha Vir Chakra billion dollars in 2016-17 to 3.13 billion dollars in 2017-18
for his bravery.He had trained over 400 Bengali college and Lima ranked third among export destination for
students as maritime commandos in the run up to the India in Latin America and Caribbean region.
1971 war against Pakistan.In 2012 Bangladesh Govt 52. On 19th March 2019, Export-Import Bank of India (Exim
conferred him the “Friend of Liberation War” honour. Bank) signed a loan agreement on behalf of the
46. The International Day for the Elimination of Racial government of USD 83.11 million (approx
Discrimination is observed annually on 21 March to raise Rs.5,72,08,76,850) to Congo for financing three solar
awareness about the negative consequences of racial power projects in the central African country.It aims to
discrimination. In 2019, it was observed with a theme provide support for financing three solar photovoltaic
“Mitigating and countering rising nationalist populism power projects with a total capacity of 35 megawatts in
and extreme supremacist ideologies”. The day the three provinces — Karawa, Mbandaka, and

©LegalEdge Tutorials Page 61 of 76


Replication or other unauthorized use of this material is prohibited by the copyright laws of India
LE Prep Assist
Lusambo.Exim Bank.During the three days of 14th CII- succeed Raghupati Singhania, CMD of JK Tyres &
Exim Conclave 2019 which was concluded on 19th March Industries Ltd.Initiated in 1975, Automotive Tyre
in New Delhi, these LOC agreements were exchanged Manufacturers’ Association (ATMA) is a national
between Ambassador of Congo to India Mossi Nyamale industry body representing Rs 60,000 crore ($ 8.5 billion)
Rosette and Exim Bank Managing Director David Indian automotive tyre sector.
Rasquinha.Till now, Exim Bank has extended 10 LOC 57. On 20th March 2019, Election Commission(EC) of India
total value of USD 578.05 million, to the government of has appointed 2 retired Indian Revenue Service(IRS)
Congo on behalf of the Government of India.With this officers of the Income Tax Department, Shailendra
agreement, Exim Bank has a total of 244 LOCs covering Handa and Madhu Mahajan as special expenditure
63 countries in Africa, Asia, Latin America and the observers for the upcoming Lok Sabha Election. The 2
Commonwealth of Independent States (CIS) with around IRS officers will look after the following exertion:
$23.43 billion credit commitments. 1.Supervising and monitoring work done by election
53. On 19th March 2019, India registered the highest growth machinery. 2.To curb black money and illegal
of 27% among countries in patent applications in 2018 inducements of money to lure voters. 3.To Supervise
with the World Intellectual Property Organization complaints received through the C-Vigil app.
(WIPO). India filed 2013 international patent applications 58. The World Sparrow Day(WSD) is observed annually on
during the year.In the volume of patent filings, India fall 20th March to spread awareness about the house
below China and the US. China is expected to overtake sparrow as well as to educated people about the threat to
the US as the leading hub for global innovation as per their population. The Theme for this year’s World
WIPO.China filed 53345 international patent applications Sparrow Day is “I love sparrows”. The Nature Forever
last year which was 48905 in 2017. Innovators and Society of India founded by Mohammed Dilawar in
research organizations in India in the previous year filed collaboration with the Eco-Sys Action Foundation
1583 patent applications with WIPO.The report on (France) has taken this international initiative.In the year
patents are filed by members of the WIPO’s Patent 2010, the first World Sparrow Day was celebrated and in
Cooperation Treaty (PCT) under which innovators from the year 2012, a campaign “Rise of Sparrows” launched
the member countries file their patent applications with by Delhi Government aimed at conservation of House
the WIPO which is to be transmitted to the patent Sparrow and also declared it as ‘state bird’ of Delhi.
regulators in different countries for securing patent 59. On March 20th of every year the ‘International
rights.US-based applicants filed 56,142 PCT applications, Happiness Day’ is celebrated worldwide.The concept of
followed by those from China (53,345), Japan (49,70(b), happiness day was founded by prominent United
Germany (19,88(c), and Korea (17,01(d). Nations special advisor Jayme Illien and it was
54. On 19th March 2019,SpiceJet received the International celebrated from the year 2013.Theme of International
Air Transport Association (IATA) Day of Happiness 2019 is ‘Happier Together’. This year
membership,becoming the first among the Indian low- also celebrates with a central aim “ShareHappiness and
cost carriers.The membership will be helpful for SpiceJet be Part of Something Amazing.”Main concept of the day
for its expansion plans by to have code shares and is to create awareness about importance of happiness in
agreements with other carriers.SpiceJet is the fifth everyone’s life around the world.
member of IATA from India. The membership will allow 60. PD. Tshering,Chief Postmaster General, Jammu &
the carrier to infuse innovative ideas and best practices. Kashmir Circle, released a ‘Special Stamp Cover on Ice
IATA is a trade association for world’s airlines. It Stupa’ by the Department of Post, Government of India
represents around 290 airplanes or 82% of the total air during a function organised at the Ice Stupasite at
traffic. Gangles in Leh. Ice Stupa team of Himalayan Institute of
55. On 15th March 2019, Indian psychologist professor Ashis Alternatives, Ladakh (HIAL) with support from Leh-
Nandy, who was Former director of the Centre for the Phodo society and Army, has built an Ice Stupa at
Study of Developing Societies (CSDS) and co-founder of Gangles with an aim to reduce shortage of water in
the postcolonial theory, has been chosen for the Hans- Leh.The team plans to build more such artificial glaciers
Kilian Award 2019 which carries an amount of 80,000 (Ice Stupas) in Leh, so that problem related to water
Euros.He received the prestigious Fukuoka Asian supply, faced by Ladakhi farmers due to receding of
Culture Prize in 2007.The award is given for the glaciers, can be reduced.About 12 villages across Ladakh
achievements of providing deeper insight regarding have built Ice Stupas on experimental basis and on 22nd
human psyche and aims to boost the growth of March, ‘World Water Day’ will be celebrated at Ice Stupa
interdisciplinary social psychology.It was named after at Shara, where teams from other villages, who have built
the late social psychologist and psychoanalyst Hans Ice Stupas, will be felicitated.
Kilian and given by the Köhler Foundation. Dr. Hartmut 61. On 18th March 2019, Government of India and
Bohme (201(a), Dr. Hans Joas (201(c), Dr. Jessica Government of Japan organized the 3rd Indo-Japan
Benjamin (2015) and Dr. Jaan Valsiner (2017) are the Workshop on Disaster Risk Reduction(DRR) in New
previous recipients of this award. Delhi. Additional Principal Secretary to Prime Minister,
56. On 18th March 2019, Automotive Tyre Manufacturers’ Dr. P. K. Mishra inaugurated the workshop. The
Association (ATMA) has appointed Chairman & objective behind this meet was to enhance collaboration
Managing Director of Madras Rubber Factory between research institutes, cities and the private sector
Limited(MRF) Mr. K M Mammen as its chairman.He will in the field of DRR.The workshop was attended by about

©LegalEdge Tutorials Page 62 of 76


Replication or other unauthorized use of this material is prohibited by the copyright laws of India
LE Prep Assist
140 delegates from India and Japan including experts Ceremony,which was held at Rashtrapati Bhavan, New
from premier research institutes, city administers and Delhi.
specialized Disaster Management agencies.In September 67. Rear Admiral Krishna Swaminathan, VSM, has assumed
2017, India and Japan signed Memorandum of charge as Flag Officer Sea Training (FOST) at Kochi on
Cooperation (MoC) in the field of DRR. 18th March 2019.He is a specialist in Communication and
62. Uttar Pradesh for the first time has undertaken census of Electronic Warfare was commissioned into the Indian
otters in protected areas of the state. The census exercise Navy on 1st July 1987, and is an alumnus of the National
was started from Pilibhit Tiger Reserve (PTR) and will be Defence Academy, Khadakwasla at Pune.He will
completed by end of March 2019 covering all protected succeed Rear Admiral Sanjay J Singh, who has been
areas of state. Otter It is important part species (mammal) appointed as Flag Officer Commanding Western Fleet
of forest ecosystem. It spends much of its time in or close (FOCWF).
to water bodies. It lives on fish. Its den is known as holt 68. Designer Ritu Beri, First Indian to be Appointed as
and it is close to water bodies. Its thriving population Cultural & Tourism Ambassador of Uzbekistan in
indicates healthy river ecosystem and water bodies are India.She received this recognition for her contribution
pollution-free. Three species of otters are found in India. towards bringing India and Uzbekistan together, fusing
It is classified as vulnerable species on IUCN Red List of cultures in her clothing line.
Threatened Species. 69. Indian Institute of Public Health Gandhinagar, Gujarat,
63. External Affairs Minister Sushma Swaraj held a 2-day (IIPHG) in collaboration with National Mental Health
joint ministerial meeting for 17th and 18th March 2019, Program, Department of Health and Family Welfare,
with her Maldivian counterpart Abdullah Shahid, where Government of Gujarat has developed a mobile
the latter has assured to remain sensitive towards India’s application called as ‘Conquer Exam, Be a Warrior’ for
security and strategic concerns.An Agreement was students to reduce the exam pressure.This application
signed between Export Import Bank of India (EXIM) and creates awareness as well as help students to analyze
Maldives Finance Ministry for line of credit (LOC) of their weak points and overcome from the exam pressure
$800 million to finance the infrastructure projects of the by providing assistance.At present the app launched in
new government in Maldives. India will provide finance English language only but in coming days it will
assistance of 1.4 billion US dollars, as per the incorporate with Hindi, Gujarat and Marathi language.
announcement made during Ibrahim Mohamed Solih’s 70. Former White House Economist, Alan B. Krueger, who
visit in December 2018. The Line of Credit is a part of this advised two presidents and helped lead economics
finance assistance. Exim Bank will charge 1.75 percent toward a more scientific approach to research and
interest rate and the loan will come with a 15-year policymaking, was found dead at his home in Princeton,
repayment period after the 5 years of moratorium. New Jersey, United States after committing suicide.He
64. Metropolitan Clearing Corporation of India Ltd (MCCIL) was born on 17th September 1960 in Livingston, New
has become the first clearing corporation in India to Jersey, United States and was 58 years old at the time of
receive recognition as a Third-Country Central his death.He was an assistant secretary of the Treasury
Counterparty (TC-CCP) by Bank of England for offering from 2009 to 2010, as President Barack Obama’s
various clearing services to UK-based entities.The administration tried to lead the United States out of its
recognition has been granted under United Kingdom’s worst recession since the Great Depression.President
Temporary Recognition Regime (TRR).In the year 2017, Obama later named him chairman of the Council of
the European Securities and Markets Authority (ESMA) Economic Advisers, a post he held from 2011 to 2013.He
also recognized the Metropolitan Clearing Corporation was the Labor Department’s chief economist under
of India Ltd (MCCIL) as a third Country Central President Bill Clinton from 1994 to 1995.
Counterparty.A financial institution which provides 71. Global Recycling Day is held on 18th March every year,
clearing and settlement services for trading in foreign with an aim to change the mindset of governments,
exchange, securities, and derivative contracts as well as businesses, communities and individuals around the
deals with the counter party credit risk between the two world, to see recyclables as resource – not waste.The
parties known as Central counterparty clearing (CCP). theme for Global Recycling Day 2019 was “Recycling into
65. Indian journalist, author and chief of The Wire, ‘Raghu the Future”.It was created in 2018 to help recognize, and
Karnad’ received the prestigious Windham-Autgor celebrate, the importance recycling plays in preserving
RaghuCampbell prize of USD 165,000 for his debut book, our precious primary resources and securing the future
‘Farthest Field: An Indian story of Second World War’, of our planet.
published in the year 2015 in non-fiction category.The 72. Chief of the Naval Staff, Admiral Sunil Lanba, presided
award was given by Yale University in the United over the day long review session of largest War Game of
Kingdom to 8 recipients of English language writers from the Indian Navy – the Theatre Level Operational
around the world.The book was also awarded the Readiness Exercise (TROPEX 19) in Kochi, intended to
Sahitya Akedemi Yuva Puraskar for a writer in English examine the conduct of the exercise and to assess the
in the year in 2016, the book was shortlisted for the operational preparedness of the Indian Navy.The
Hessell-Tiltman Prize. exercise had commenced 7th January 2019 and was
66. On 19th March 2019, the Gallantry Awards and planned to be terminated by 10th March 2019 but due to
Distinguished Service Decorations were conferred by the JEM (Jaish-e-Mohammed) sponsored terrorist attack
President Ram Nath Kovind at the Defence Investiture on the Central Reserve Police Force (CRPF) convoy in

©LegalEdge Tutorials Page 63 of 76


Replication or other unauthorized use of this material is prohibited by the copyright laws of India
LE Prep Assist
Pulwama on 14th February 2019 led to the rapid ICICI Lombard.It will strengthen the relationship
redeployment of the Indian Navy for Operations in between MobiKwik and ICICI Lombard and will provide
North Arabian sea.TROPEX 19 had commenced with Tri- innovative and unique products against the new-age
services Amphibious Exercise in the Andaman & risks.
Nicobar Islands with participation of Army and Air 76. On 18th March 2019, WhatsApp has joined hands with
Force and about 60 ships of the Indian Navy, 12 ships of NASSCOM Foundation, to tackle the challenge of
the Indian Coast Guard and 60 aircraft were part of the misinformation. The aim of the partnership is to train
exercise.This was followed by the largest Coastal 100,000 Indians to spot false information and provide tips
Defence Exercise code-named SEA VIGIL on 22nd and tricks to stay safe on WhatsApp.With this
January and 23rd January 2019 with the participation of partnership, Whatsapp aims to reach approximately
all 13 Coastal States and Union Territories along with all 100,000 Indians with training to spot false information
maritime stakeholders. and provide tips and tricks to stay safe on WhatsApp.The
73. On 16th March 2019, The Prime Minister Narendra Modi company said that the co-created curriculum encourages
launched a ‘Main Bhi Chowkidar’ campaign calling people to be mindful about forwarding rumors. The
everyone who was fighting corruption, social evils and training will include real-world anecdotes, tools which
working hard for the progress of India, as can be used to verify a forward and actions users can take
watchmen.Prime Minister also shared a 3.45-minute like reporting problematic content to fact checkers and
video titled “Take The Pledge“, urging people to join law enforcement also the curriculum will be
Modi on March 31 at 6 p.m. for a video programme titled disseminated in multiple regional languages.On March
‘Main Bhi Chowkidar”.BJP has also started a unique tech 27, the first training will start in Delhi and will be
innovation called conversation cards on twitter for this followed by more planned interventions like hosting
campaign where people will get a personalized message training workshops for representatives from rural and
from PM on Twitter on participating in this campaign. urban areas along with road shows across numerous
74. The Government of India has formed an inter-ministerial colleges.
panel for monitoring, sanctioning and implementation of 77. 13 years old pianist from Chennai Lydian Nadhaswaram
projects under the Rs 10,000-crore Faster Adoption and won the prize of $1 million (6.9 crore) for his performance
Manufacturing of Electric Vehicles in India (FAME-II) on the American reality show ‘The World’s Best’.The
programme, aimed at incentivising clean mobility.The young prodigy also made his debut on The Ellen Show.
Project Implementation and Sanctioning Committee, for Lydian Nadhaswaram is also a student of the Chennai-
the purpose of overall monitoring, sanctioning and based KM Music Conservatory which is run by A.R.
implementation of the scheme, will be chaired by the Rahman in Chennai.
secretary in the Department of Heavy Industries, Dr. A. 78. Asia’s richest man and Reliance Industries chairman
R. Sihag.The programme under the second phase of the Mukesh Ambani topped the Forbes World’s Richest
FAME India scheme will be implemented over a period Sports Team Owners 2019 list with a total net worth of
of 3 years with effect from 1st April 2019. 10 lakh about $50 billion.He bought the IPL team Mumbai
registered electric two-wheelers with a maximum ex- Indians for over $100 million in 2008 through RIL’s 100
factory price will be eligible to avail incentive of Rs 20,000 percent subsidiary IndiaWin Sports.He has been
each and it will also support 5 lakh e-rickshaws having followed by CEO of Microsoft, Steve Ballmer ($41.2
ex-factory price of up to Rs 5 lakh with an incentive of Rs billion), who owns Kroenke Sports & Entertainment and
50,000 each.It will offer an incentive of Rs 1.5 lakh each to Micky Arison (Net worth: $8.9 billion), owner of the NBA
35,000 electric four-wheelers with an ex-factory price of franchise Miami Heat.
up to Rs 15 lakh, and incentive of Rs 13,000 each to 20,000 79. C Prem Kumar who is a Cinematographer-turned-
strong hybrid four-wheelers with ex-factory price of up director, will be awarded with 22nd Gollapudi Srinivas
to Rs 15 lakh.It will support 7,090 e-buses with an National Award for his debut film Tamil ‘96’ on 12th
incentive of up to Rs 50 lakh each having ex-factory price August 2019 in Chennai, Tamil Nadu. The award is
of up to Rs 2 crore.The scheme will have a Rs 1,500-crore presented to debutant filmmaker by Gollapudi Srinivas
outlay in 2019-20; Rs 5,000 crore in 2020-21 and Rs 3,500 Memorial Foundation in the memory of Gollapudi
crore in 2021-22. It will cover buses with EV technology; Srinivas who passed away while making his first
electric, plug-in hybrid and strong hybrid four wheelers; project.The award consists of a cash prize of Rs 1.5 lakh
electric three-wheelers including e-rickshaws and electric and a memento. ’96’ is the 2nd Tamil film after Kutty
two-wheelers. (201(a) directed by Janaki Viswanathan to receive this
75. On 18th March 2019, ICICI Lombard and digital financial award. The Movie ’96’ was set against the backdrop of a
services company MobiKwik entered into a partnership school alumni reunion. Trisha and Vijay Sethupathi
to provide cyber insurance cover of Rs. 50000.It aims to played the leading characters in the movie.
provide protection against unauthorised and fraudulent 80. The Reserve Bank of India (RBI) has prohibited India’s
transactions online across debit/credit cards, bank largest mortgage lender HDFC from owning more than
accounts and mobile wallets.The service will be available the regulatory limit in Bandhan Bank.HDFC can hold
through an app at Rs. 99 per month along with an insured 9.9% in the bank which is the maximum a non-banking
sum of Rs. 50000.To enjoy the benefit of a stress-free and finance company can own in a private bank.HDFC,
secured transaction MobiKwik users can avail the which will merge its low-cost arm Gruh Finance with
‘Commercial Cyber Insurance’ policy underwritten by Bandhan Bank, had sought approval to hold 14.96%.

©LegalEdge Tutorials Page 64 of 76


Replication or other unauthorized use of this material is prohibited by the copyright laws of India
LE Prep Assist
81. Indian Coast Guard Ship ‘Vijit’ became the first-ever with 20 medals. Japan got 6 gold, 8 silver and 6 bronze
Coast Guard ship to visit Sabang, Indonesia, highlighting medals.Boys medley relay team Karan Hegiste,
the close maritime proximity between India and Shanmuga Nalubothu, Shashikant Angadi and Abdul
Indonesia and strengthening cooperation in the area of Razak Rasheed defeated Sri Lankan team to acquire first
maritime security and safety between New Delhi and position in the game.Avantika Santhosh Narale received
Jakarta.During its maiden voyage from 17th March to gold in 100m and a silver medal in 200m final while
20th March 2019, officers and crew of Vijit will hold Deepthi secured bronze medal in the 200m event.
interaction with officials of Bakamla (Indonesian Coast 88. India’s Asian marathon champion Gopi Thonakal has
Guard), Indonesian Armed Forces and the civil qualified for the World Athletics Championships to be
administration at Sabang.This visit of the coast guard held in Doha in September-October after finishing 11th
ship follows from the first-ever-visit of Naval Ship INS in the Seoul International Marathon.The 30-year-old
Sumitra to Sabang in July 2018.The ship is commanded Gopi clocked his personal best time of 2 hours 13 minutes
by Commandant T Ashish (0532-J), who is a long 39 seconds in the marathon race on Sunday, bettering the
Navigation Direction officer. World Championships qualification mark of 2:16:00. His
82. Mercedes Formula One (F(a) driver Valtteri Bottas has earlier best was 2:15:16 last year.
clinched his first championship in the Australian Grand 89. :Goa Chief Minister Manohar Parrikar passed away at
Prix held in Melbourne. He was followed by Lewis the age of 63 years after fighting a prolonged battle with
Hamilton of Mercedes and Red Bull’s Max pancreatic cancer at his residence in Panaji, Goa.He is the
Verstappen.He is a Finnish racing driver and now India’s 18th and Goa’s second chief minister to die in
competing in Formula One with Mercedes.Valtteri Bottas office.Former Defence Minister and four-time Goa chief
achieved his first Formula One pole position in minister, he was among the few from Goa who made it
Bahrain.He won the 2017 Russian Grand Prix and 2017 to the national level.He held the Defence portfolio in the
Austria Grand Prix. Narendra Modi-led government from 2014 to 2017 after
83. Bengaluru Football Club (FC) won their first Indian which he returned to being Goa Chief Minister once
Super League (ISL) title at the Mumbai Football Arena again.The Central government announced national
defeating FC Goa by 1-0 in the final.Both teams did not mourning on 18th March 2019 following his demise and
score any goals in the 90-minute time frame and the game the National Flag was flown at half-mast in New Delhi
went on for extra time.In the 118th minute, Rahul Bheke and capitals of all States and Union Territories.Manohar
jumped up to shoot a header into the far corner and get Parrikar was born in Mapusa, Goa on 13th December
Bengaluru their winning goal. 1955 and did his schooling at Loyola High School,
84. On 17th March 2019, the professional tennis tournament Margao. After finishing his secondary education in
Indian Wells Masters 2019 also known as BNP Paribas Marathi, he went on to graduate in metallurgical
Open held in California, United States. Prize money for engineering from the Indian Institute of Technology,
the game was $9035428 for ATP and WTA Bombay (IIT Bombay) in 1978.He is the first IIT alumnus
separately.Austrian player Dominic Thiem defeated five to serve as MLA of an Indian state and was awarded the
time champion Roger Federer of Switzerland to win Distinguished Alumnus Award by the Indian Institute of
Men’s Singles title. It is Dominic’s first Masters 1000 Technology, Bombay in 2001.
title.Croatia’s Nikola Mektic and Argentina’s Horacio 90. Explanation:
Zeballos won Men’s Doubles title by defeating Poland’s On 18th March 2019, Veteran Bengali actor, Chinmoy
Lukasz Kubot and Brazil’s Marcelo Melo. It was their first Roy who was known for his portrayal of comic
Doubles title. characters, passed away at his salt lake residence
85. Canadian player Bianca Andreescu defeated German following Cardiac arrest at the age of 79.Chinmoy Roy
player Angelique Kerber in Women’s Singles Category to was born in Kumilla district of present day Bangladesh
win her first ever Indian Wells title. The score was 6-4, 3- in 1940 and he started his career in Bengali films in
6, 6-4.Elise Mertens of Belgium and Aryna Sabalenka of 1960s.He worked in films like Basanta Bilap and Dhonni
Belarus won their first Doubles title as a team against Meye. He also worked in children’s fantasy Goopy Gyne
Barbora Krejcikova of Czech Republic and Katerina Bagha Byne.
Siniakova of Czech Republic. The duo won the game for 91. Ordnance Factories Day is observed annually on March
6-3, 6-2. 18 all over India to commemorate the first production of
86. The 2019 Swiss Open badminton championship held at Ordnance Factory of India. Indian Ordnance Factories
St. Jakobshalle in Basel, Switzerland from 12th March to are regarded as the backbone of India’s Defence
17th March 2019. The total prize money amounts to USD production.On 18th March 1802, the production of the
150000. oldest Ordnance Factory of India was commenced at
87. India finishes second in Asian Youth Athletes Cossipore, Kolkata.On the 200th anniversary of the first
Championship held in Hong Kong with a gold added to ordnance factory which is now known as the Gun and
the medal tally on the final day of the Shell Factory, the day was declared as Ordinance
championship.India secured second position in the factories day.The day is celebrated by displaying the
overall medals tally with 26 medals including 8 gold, 9 major weapons manufactured in the factories, through
silver and 9 bronze medals.China topped the exhibitions across all over India.The Ordnance Factories
championship with 31 medals including 12 gold, 11 silver Board is known as the country’s “Fourth Arm of
and 8 bronze. Japan ranked third in the Championship

©LegalEdge Tutorials Page 65 of 76


Replication or other unauthorized use of this material is prohibited by the copyright laws of India
LE Prep Assist
Defence” and the remaining three arms are – the Navy, Bank of India.The appointment of KVS Manian and
the Air Force, and the Army. Gaurang Shah is also subject to share¬holders’ approval.
92. The Indian Institute of Technology (IIT) Kharagpur will KVS Manian is currently working as the president –
adopt Amazon Web Services (AWS) Educate programme corporate, investment bank and Gaurang Shah is the
to help students gain cloud computing skills including president- group chief risk officer of the bank.
hands-on experience in artificial intelligence (AI).It will 96. Astronomers have discovered 83 quasars powered by
be introduced to provide AWS Cloud Computing supermassive black holes 13 billion light-years away
experience and AI-enablement for all our students, from the Earth, from a time when the universe was less
irrespective of their branch of study.The programme than 10% of its present age.The finding, published in the
offers a robust set of no-cost tools, resources and AWS Astrophysical Journal, increases the number of black
Promotional Credits for students and educators to boost holes known at that epoch (a particular period of time in
their cloud skills and experience. As part of AWS history or a person’s life) considerably, and reveals, for
Educate, the students will gain access to 12 Cloud Career the first time, how common they are early in the
Pathways covering topics that are in demand by universe’s history.
employers, such as machine learning, cybersecurity, and 97. On 16th March 2019, Indian woman golfer Diksha Dagar
software development, each with over 30 hours of became the 2nd Indian winner on Ladies European Tour
content.Upon completion, learners will be eligible to after clinching South African Women’s Open in Cape
receive an AWS Educate Certificate of Completion or an Town. She follows Aditi Ashok, who in 2016 became the
AWS Educate Badge. first Indian to win on the Ladies European Tour after
93. On 16th March 2019, Reserve Bank of India maintained clinching Hero Women’s Indian Open in Gurugram.
that there is no dilution in its stand regarding February 98. Senior journalist and environmental activist Darryl
12 circular on stressed assets recognition and D’Monte died at a hospital in Mumbai after a brief illness
resolution.The central bank said in a statement that it is at the age of 74 years. In a career spanning several
restated that the RBI maintained its stand on all aspects decades, he served as the resident editor of Mumbai
of the Framework as has been consistently stated in its editions of The Times of India and The Indian
communications, including the clarification given during Express.Besides journalism, he had a keen interest in
the post-monetary policy press conference which was environmental issues and took some initiatives for the
held on February 7, 2019.Also there would be no changes cause, he added.
in the circular, said RBI Governor Shaktikanta Das on last 99. The Energy and Resources Institute (TERI), one of
month.The circular directed lenders to refer any loan country’s leading think-tanks, will set up an industrial
account more than Rs 2,000 crore under the Insolvency incubation facility of TERI-Deakin Nanobiotechnology
and Bankruptcy Code (IBC) if the loan amount is not Research Centre, in Odisha. The facility will support
resolved within 180 days of default. Also it underscored development of biotechnology solutions to address
IBC’s status as the cornerstone of the bad loan resolution issues affecting agriculture, environment and energy.
framework, scrapping all previous mechanisms.The The Odisha government is making significant efforts to
circular imposed a one-day default rule and banks have promote innovation, and latest effort being launching of
to treat a company as a defaulter even if it misses the Odisha Biotechnology Policy-2018 that is based on
repayment schedule by a day. three key pillars– innovation, entrepreneurship and
94. The two-day national convention of War Decorated India investment. The TERI-Deakin Nanobiotechnology
was held at the Manekshaw Auditorium in Research Centre (TDNBC) offers to participate and to
Chandimandir, the headquarters of the Western support the state government’s vision to promote and to
Command, near Chandigarh and was attended by some assist further in implementing innovation and
living gallantry award winners and relatives of the translational research.
deceased awardees. Next of kin, relatives of the awardees 100. The 3rd edition of Namaste Thailand festival, organised
were felicitated at the convention. Among Victoria Cross by the Royal Thai Embassy with an aim to strengthen
awardees, the relatives of Late Naib Subedar Nand Singh bilateral ties and increase cultural exchange between
(Sikh Regiment), Late Subedar Ram Sarup Singh (1 India and Thailand, began at Select Citywalk in New
Punjab) and Late Badlu Ram (Cavalry) were Delhi on 15th March 2019 and will conclude on 17th
felicitated.Among Param Vir Chakra awardees, relatives March 2019. The festival will feature stage performances
of Late Col. Hoshiar Singh (Grenadiers) and Late by Thai artistes including the much popular Thai folk-
Subedar Joginder Singh (Sikh Regiment) were jazz band Asia-7. It will feature a series of Thai craft
felicitated.Among Maha Vir Chakra awardees those activities including mulberry paper mini umbrella
felicitated were the relatives of late Brigadier N.S. making, fan painting, body paint, and button badge
Sandhu (Dogra), late Brigadier K.S. Chandpuri (Punjab making along with the traditional Thai cuisines.
Regiment), late Lt General Gowry Shankar (Signals), late 101. On 25th March 2019 Reserve Bank of India (RBI) has
Lt Ved Prakash Trehan (Rajputana Rifles), late appointed 5-member committee under the chairmanship
SepoyAnusuya Prasad (Mahar Regiment) and late Porter of Nandan Nilekani to strengthen digital payments as
Mohammed Ismail (Punjab Regiment Civilian Porter) . well as to boost financial inclusion through Financial
95. KVS Manian and Gaurang Shah appointed as Kotak Technology.It will provide recommendations in 3
Mahindra Bank’s whole-time directors for a three-year months.The decision regarding the formation of
term effective from the date of approval from the Reserve committee was taken during the first event of Financial

©LegalEdge Tutorials Page 66 of 76


Replication or other unauthorized use of this material is prohibited by the copyright laws of India
LE Prep Assist
Technology (Fintech) Conclave, which was organized by action, demand justice and to protect UN
NITI Aayog in New Delhi.The purpose of the event was staff,peacekeepers and the workers in non-governmental
to shape India’s predominance in FinTech, build the community and the press.Alec Collett worked for the
future strategy and policy to uplift the financial United Nations Relief and Works Agency for Palestine
inclusion.With the revolution in the FinTech sector Refugees in the Near East (UNRWA).
India’s aim is to emerge as a 10 trillion dollar 107. Italy and China want to revive the spirit of the ancient
economy.Financial technology, popularly known as Silk Road by deepening their trade and investment ties.
FinTech, is the new technology and innovation to make Italy has signed a non-binding protocol with China to
financial services more accessible for the common take part in Beijing’s new Silk Road of transport and
people. Mobile banking, internet banking, digital trade links stretching from Asia to Europe. Italy became
payment are examples of Financial technologies. the first G7 country to sign up for the project. Chinese
102. Peter Tabichi, a Kenyan math and physics teacher and a President Xi Jinping and Italian Prime Minister Giuseppe
member of the Franciscan religious order has been Conte both attended a ceremony in Rome yesterday for
conferred with the prestigious $1 million Varkey the signing of 29 Memoranda of Understanding, which
Foundation Global Teacher Prize for 2019 at a ceremony Italian media said were worth 5.6 to 8 billion US Dollars.
in Dubai hosted by Hollywood actor, Hugh Jackman. 108. On 23rd March 2019, The 64th Filmfare Awards 2019 was
Peter Tabichi gives away 80 per cent of his monthly held at Jio Garden, BKC, Mumbai. The event was hosted
income to the poor.The award recognises the by Bollywood’s leading Actors, Directors, Musicians and
“exceptional” teacher’s commitment to pupils in a Singers. The winners of the 64th
remote part of Kenya’s Rift Valley. 109. On 21st March 2019, Italy’s new Earth-observation
103. Naresh Goyal, the founder of Jet Airways (India) Ltd, has satellite named PRISMA (Precursore Iperspettrale della
resigned from the board of the troubled airline after a Missione Applicativa) launched from the Guiana Space
meeting with the airlines board of directors. The lenders Centre in Kourou, French Guiana by 98 foot (30 meters)
will immediately pump in about Rs.1,500 crore into Jet tall Vega rocket which was developed by the joint
Airways by way of issue of appropriate debt instruments venture of Italian Space Agency OHB Italia SpA and the
against security of assets. A bidding process will be European Space Agency LEONARDO SpA. PRISMA is
initiated by lenders for issuing shares to a new investor an electro-optical instrument, which is designed to take
in Jet Airways in June. medium resolution photos to give information about
104. On the lines of Indo-US deal inked in September 2015 for environmental monitoring, natural resources, pollution
the purchase of 15 Chinooks by India, the first four heavy and crop health.The PRISMA satellite will operate in a
lift CH-47F (I) Chinook helicopters were inducted in the sun-synchronous orbit.
Indian Air Force, at Air Force Station, Chandigarh, in 110. On 23rd March 2019, former Chief Election
Squadron No 126 Helicopter Flight. These multi-role Commissioner of India Navin Chawla’s book ‘Every
helicopters will act as life-saver during HADR Vote Counts’ was launched by former Vice President of
(humanitarian assistance and disaster relief) India, Hamid Ansari in New Delhi. This book is about
missions.Regarded as one of the most modern heavy lift how the electoral machinery works in India.The book has
chopper, the payload capacity of Chinook is about 10 been published by HarperCollins India.
tonne, as it is able to lift artillery, vehicles, road 111. In accordance with the preliminary world airport traffic
construction and engineer equipment as well as troops rankings for 2018 released by Airports Council
and supplies to mountainous sectors. It can also lift M- International (ACI), the Indira Gandhi International
777 ultra-light howitzers of the Indian armed forces.India Airport (IGIA) of New Delhi has elevated 4 ranks to reach
has purchased 15 Chinooks for $1.5 billion. The delivery at 12th spot in terms of busiest airport as compared to
of remaining helicopters is expected to be completed by 2017’s 16 spot. The list of world’s busiest airports by
2023.The manufacturer of Chinook is Boeing Defense, passenger traffic ranking has been topped by Hartsfield–
Space & Security, US. Jackson Atlanta International Airport (US) followed by
105. Oslo, the capital city of Norway will become the world’s Beijing Capital International Airport (China) and Dubai
first city to install wireless, induction-based charging International Airport (UAE).In 2018, India has followed
stations for its fleet of electric taxis with an aim to have a US and China to become the world’s third-largest
zero-emission cab system by the year 2023. Finland’s aviation market in terms of passenger throughput.
utility company ‘Fortum’ will collaborate with U.S. 112. On 23rd March 2019, Vice Admiral Karambir Singh has
company Momentum Dynamics and the government of been appointed as the next (24th) chief of naval staff. He
Oslo to install induction charging plates in the roads that will succeed present Chief of Naval Staff Admiral Sunil
allow for charging up to 75 kilowatts.Norway wants all Lanba who retires on May 2019. At present, he is a Flag
new cars to be zero emission by 2025 whereas other Officer Commanding in Chief of the Eastern Naval
nations, like Britain and France have similar goals for Command in Visakhapatnam. He is from Jalandhar in
2040. Punjab. He was commissioned into the Indian Navy in
106. On 25th March 2019, International Day of Solidarity with July 1980 and after two years he earned his wings as a
detained and missing staff members was observed all helicopter pilot.During his 39 years of service, he has
over the world to mark the anniversary of the abduction flying experience of the HAL Chetak and Kamov Ka-25
of Alec Collett, a former journalist who was kidnapped helicopters and commanded numerous ships including
by armed gunman in 1985. The day aims to prepare ICGS Chand Bibi, INS Vijaydurg, INS Rana and INS

©LegalEdge Tutorials Page 67 of 76


Replication or other unauthorized use of this material is prohibited by the copyright laws of India
LE Prep Assist
Delhi.He has been honored with the Param Vishisht Seva 118. By creating nanoscale patterns on the surfaces of objects,
Medal (PVSM) and the Ati Vishisht Seva Medal (AVSM). Scientists of California Institute of Technology (Caltech)
113. On 22nd March 2018, Microsoft developed a storage have designed a way to levitate and propel objects using
device in partnership with the University of Washington only light.It has been named as “photonic levitation and
that can translate digital information into DNA.The propulsion” system.This will also help to develop a
software used in the system converts the 1’s and 0’s of spacecraft that could reach the nearest planet outside of
digital data into DNA sequence (A’s, T’s, C’s and G’s) our solar system in 20 years.The spacecraft which is sized
and stored it as a liquid.The device converted the word with nanostructures can travel to other stars without
‘Hello’ into DNA which took 21 hours. having the fuel.The study has been published in journal
114. Operation Sahayata 2019-India Navy was the first Nature Photonics.
responder in the evolving humanitarian crises in the 119. NASA’s spacecraft has discovered an evidence of water
aftermath of Cyclone IDAI that hit Mozambique on 15 and plumes erupting from the surface of asteroid Bennu
March 2019.Assistance is also being sent to Zimbabwe and will return with a sample in 2023.From the very
and Malawi, two other countries hit by the cyclone. beginning of our solar system the asteroid Bennu, which
Cyclone IDAI made landfall in East & Southern Africa is slightly wider than the height of the Empire State
around 15 March 2019 causing widespread destruction Building, may contain unaltered material.The asteroid is
and loss of human lives in Mozambique, Zimbabwe and more rugged than expected, and for this reason, it
Malawi. In response to a request from the Republic of challenges the mission team to alter its flight and sample
Mozambique, the Government of India immediately collection plans.On December 2018, NASA’s Origins,
diverted 3 Indian naval ships to the port city of Beira. Spectral Interpretation, Resource Identification, Security-
Over the last several days, the 3 Indian ships, INS Sujata, Regolith Explorer (OSIRIS-REx) mission, started orbiting
ICGS Sarathi and INS Shardul are undertaking HADR the Bennu, a near-Earth asteroid.
(Humanitarian Assistance Disaster Relief) in 120. French Open remains the Grand Slam event with the
coordination with local authorities and the High least prize money on offer for the winners. The French
Commission of India, Maputo. As of now, Indian naval Open announced an 8% increase to its prize money for
crew has rescued more than 192 people. the 2019 tournament on Thursday, with the men’s and
115. Martyr’s Day (88th) or Shaheed Diwas observed on 23rd women’s singles winners to take home 2.3 million euros
March every year in India to commemorate the death ($2.62 million). Roland Garros remains the Grand Slam
anniversary of three of India’s freedom fighter Bhagat event with the least prize money on offer for the winners,
Singh, Shivaram Hari Rajguru and Sukhdev Thapar.On some way short of the $3.8 million claimed by last year’s
this day, Prime Minister Narendra Modi paid tributes to US Open champions, Novak Djokovic and Naomi Osaka.
the three freedom fighters who sacrificed their lives for But French Open tournament director Guy Forget
the country.On March 23, 1931, Bhagat Singh, Sukhdev wanted to narrow the gap between the champions and
Thapar, and Shivaram Rajguru were hanged by the players who exit in the opening week, with first-round
British Government for killing British police officer John losers given a 15-percent rise to 46,000 euros.
Saunders.They mistook John for Superintendent James 121. On 20th March 2019, United Nations released World
Scott who injured Lala Lajpat Rai during a protest against Happiness Report where India is ranked 140th out of 156
Simon Commission. nations. India shows a 7 spot decline along with five
116. The Reserve Bank of India (RBI) has turned down IDBI other countries. For the second consecutive year, Finland
Bank’s proposal to change its name following the transfer topped the list. Denmark ranked second followed by
of controlling 51 per cent stake to insurance behemoth Norway, Islands and The Netherlands.The report was
LIC from the government of India. The board of IDBI released on World Happiness Day(Mar 20) by the
Bank had last month sought RBI’s approval for change in Sustainable Development Solutions Network for the
the name of the lender to either LIC IDBI Bank or LIC United Nations.Pakistan ranked 67th, Bhutan 95th,
Bank following the takeover by Life Insurance China 93rd, Bangladesh 125th and Sri Lanka 130th while
Corporation (LIC). “The board of directors has in its South Sudan ranked last in the World Happiness
meeting held on March 19, 2019, taken note of RBI’s Report.Income, freedom, Social support, trust, healthy
communication conveying their inability to accede to life expectancy and generosity are the six key variables
bank’s request for change of name of IDBI Bank assessed for the report. The results of the report are
Limited,” IDBI Bank said in a regulatory filing. correlated with factors including social security and
117. On 21st March 2019, India’s fastest growing payments GDP.
platform PhonePe appointed Bollywood superstar 122. Indian Railways Integral Coach Factory (ICF) has
Aamir Khan as its brand Ambassador.PhonePe is the surpassed the top Chinese manufacturers after creating a
official co-presenting sponsor for VIVO IPL 2019. It is 40% record increase in its production. With this, Indian
planning to launch a series of new advertisements with a Railway’s oldest and premier coach manufacturing unit
mass appeal by its ambassador on the benefits of digital becomes the single largest rail coach manufacturer in the
payments.Bangalore headquartered PhonePe is a Unified world. ICF has manufactured 2,919 coaches from April
Payment Interface (UPI) based online payment system 2018 to February 2019 as compared to 2,600 coaches by
for electronic fund transfer launch by National Payments Chinese manufacturers.Besides ICF Indian Railways has
Corporation of India (NPCI). 2 coach manufacturing facilities namely Rail Coach

©LegalEdge Tutorials Page 68 of 76


Replication or other unauthorized use of this material is prohibited by the copyright laws of India
LE Prep Assist
Factory, Kapurthala and Modern Coach Factory, championships by United World Wrestling (UWW). The
Raebareli. stripping off of India’s hosting rights is due to the India’s
123. On 19th March 2019, NASA’s Fermi Gamma-ray Space denial of visas to Pakistani shooters for the ISSF World
Telescope and the National Science Foundation’s Karl G. Cup held in Delhi in February following the Pulwama
Jansky Very Large Array (VLA) discovered pulsar attack.
hurtling through space.This pulsar is named as PSR 128. In 2010, the United Nations General Assembly officially
J0002+6216 (J0002 for short). It is located 6,500 light years declared 21 March as International Nowruz Day. Iranian
away in the constellation Cassiopeia.It speeds nearly 2.5 New Year, Nowruz has been celebrated for more than
million miles per hour which is as fast as it could travel 3,000 years in Iran, previously known as Persia, and other
the distance between Earth and the Moon in just 6 countries that were formerly part of the ancient Persian
minutes.Pulsar is a super dense rapidly spinning neutron empires.The festival has roots in Zoroastrianism, a
stars which are left behind when a massive star monotheistic religion which originated in ancient Iran.
explodes.The study will help to better understand how 129. On 20th March 2019,Kazakhstan has renamed its capital
the explosions are able to kick the neutrons to start in as Nursultan from its previous name Astana in order to
such a high speed. recognize its outgoing leader Nursultan Nazarbayev(78),
124. Japanese unmanned spacecraft,Hayabusa2 reached who has served nearly 30 years as president of
Asteriod Ryugu on June,2018.Since then,the spacecraft Kazakhstan.The name was changed after Kassym-Jomart
landed multiple robotic probes on its rocky terrain of the Tokayev sworn in as president of Kazakhstan.The new
Asteroid Ryugu.The spacecraft had surveyed 69,000 president will serve till April 2020.
locations covering almost 90 percent of Asteroid Ryugu 130. On 18th March 2019, Reserve Bank of India proposed to
by using a near-infrared spectrometer capable of replace the previous expenditure-based system with a
detecting hydrated minerals.According to its study rule-based approach in fixing new Ways and Means
published in U.S. Journal Science,the spacecraft probed limits for the state governments and set up a panel to
and found the small amounts of minerals containing recommend the parameters of the new system.The
water on the surface of the asteroid.A part of the water objective of this reform is to prevent automatic
might be from asteroids and comets. monetization of deficits.To bridge temporary liquidity
125. A Pune-based IT firm is developing software for the mismatches Reserve Bank of India which acts as debt
world’s largest ground-based telescope, the Thirty Metre manager for state governments make available short-
Telescope (TMT), which is likely to come up on Mauna term loan for them. This short-term loan facility is called
Kea in Hawaii. The telescope is being constructed jointly Ways and Means Advances.The state Government also
by India, US, China, Japan and Canada and is likely to agreed to link their receipts and payment systems with
become operational in the mid-2020s. India is building Core Banking Solution(e-Kuber) of Reserve Bank of India
the telescope’s complex optical structure, called the for the effectiveness of this new reform.
Segment Support Assembly (SSA), which will comprise 131. On 15th March 2019, Hindi writer Leeladhar Jagudi
492 hexagonal mirror segments, collectively forming the honoured with Vyas Samman 2018 for his collection of
primary mirror. An eight-member team at the IT firm, poems “Jitne Log Utne Prem” published in 2013. He also
ThoughtWorks, is designing three different software — authored several poetry anthologies. The award carries
Common Software Services (CSS), Data Management an amount of Rs. 4 lakhs and it was started by K K Birla
Services(DMS) and Executive Software (ES) — to deal foundation in 1991. Leeladhar was born in Dhangal
with the control systems to manoeuvre the telescope and village of Uttarakhand. He received Padma Shri in 2004
manage the big data that will be generated once it and Sahitya Akademi Award, Akashvani National
becomes operational. Award and Uttarakhand Gaurav Samman as
126. On 21st March 2019 during the 28th edition of the annual well.Shankhmukhi Shikharo Par, Natak Jaari Hai, Raat
‘Caving in the Abode of the Clouds Expedition ’,a five- Ab Bhi Maujood Hai and Bhay Bhi Shakti Deta Hai are
member team of cavers of Meghalaya Adventurers’ some of his famous books.
Association (MAA) has discovered India’s deepest shaft 132. Mathematician Karen Uhlenbeck(76) became the first
at Krem Um Ladaw. It has 105 m deep shaft entrance woman to be conferred with 6 million Norwegian kroner
passage. Last year, the world’s longest sandstone cave, ($700,000) Abel Prize, which is regarded as Nobel prize
“Krem Puri” was discovered in Mawsynram area of of mathematics.Karen Uhlenbeck.She was recognized for
Khasi Hills district, Meghalaya. her work in the field of gauge theory and geometric
127. India has lost the hosting rights of two international analysis gave an extensive impact in mathematics as well
tennis tournaments viz. Junior Davis Cup and Fed Cup as in physics.In 2014, Maryam Mirzakhani, became the
which were scheduled to be held in RK Khanna stadium, only woman to won the other major international
New Delhi in April. Now both events will be held in mathematics prize, the Fields Medal.
Bangkok, Thailand. Reasons behind losing of hosting 133. On 20th March 2019, 66-year old Former Supreme Court
rights: Absence of a written guarantee from India that the judge Justice Pinaki Chandra Ghose appointed as the
visa entry would be granted to the Pakistani contingent country’s first Lokpal or the anti-corruption
for the Davis Cup.Closure of Pakistan’s air space after the ombudsman,by President Ram Nath Kovind.He is
Balakot air strike, resulting flights of participating teams serving as a member of the National Human Rights
have to take alternate and long routes to enter the Indian Commission (NHRC) since June 29, 2017.Former Chief
airspace. Earlier, India had stripped to host Junior Asian Justices Dilip B. Bhosale, Pradip Kumar Mohanty, and

©LegalEdge Tutorials Page 69 of 76


Replication or other unauthorized use of this material is prohibited by the copyright laws of India
LE Prep Assist
Abhilasha Kumari and current Chief Justice of 140. External Affairs Minister Sushma Swaraj held a 2-day
Chhattisgarh High Court Ajay Kumar Tripathi have been joint ministerial meeting for 17th and 18th March 2019,
appointed as judicial members under the Lokpal.After with her Maldivian counterpart Abdullah Shahid, where
five years of the Lokpal Act,the selection was made for the latter has assured to remain sensitive towards India’s
looking into the cases of corruption by the public security and strategic concerns. Sushma Swaraj has also
servants. unveiled a plaque of the renovated Indira Gandhi
134. 78-year old Kazakhstan President Nursultan Nazarbayev Memorial Hospital in Male,Maldives. She has dedicated
announced his resignation after nearly three decades in the hospital to the natives of the Maldives. It is going to
power without stating any specific reason. He served as be the first and largest government healthcare facility in
the first and only President of Kazakhstan, from 24 April the Maldives.
1990 until his resignation on 19 March 2019. Nursultan 141. U.K. Chancellor Philip Hammond announced in a
Nazarbayev will be succeeded by Kassym-Jomart Budget update, referred to as the annual Spring
Tokayev (65). Statement, that from Autumn 2019, PhD-level
135. On 20th March 2019, 38-year-old transgender activist occupations will be exempt from the Tier 2 (General) cap,
ShreeGauri Sawant from Mumbai has been appointed as and at the same time the government will update the
first transgender election ambassador by The Election immigration rules on 180-day absences so that
Commission of India.There was no separate classification researchers conducting fieldwork overseas are not
of transgenders as voters till 2014 but after that penalised if they apply to settle in the U.K. Indians are
transgenders were first enrolled as voters along with among the largest group of professionals set to benefit
males and females.The highest enrolment of from this decision, as they form the largest chunk of
transgenders (32(d)is in Mumbai North Lok Sabha highly-skilled professionals within the Tier 2 (General)
constituency. category of work visas, accounting for 54% of all such
136. On 19th March 2019, the US based researchers from visas granted in 2018.Indian nationals also marked the
NASA, Los Alamos National Laboratory and US army largest increase in the grant of Tier 2 visas last year, upby
disclosed a new ideology that mercury is closer to earth 6% at 3,023 more visas compared to the previous year.
than Venus.As per the research on the basis of average 142. On 19th March 2019, BJP MLA and Former Speaker of
distance,every other planet when compared to mercury Goa legislative assembly Pramod Sawant(45) took oath
is farther from earth including Venus based on average as 13th Chief Minister of Goa succeeding Manohar
distance. It also included mercury is the closest planet to Parrikar, who died due to Cancer. Governor Mridula
every other planet in the solar system when considering Sinha administered the oath of office of Pramod Sawant.
the average.To calculate distance between planetary 143. The 10 days long Africa-India Joint Field Training
bodies involves their relative distance from the sun Exercise (AFINDEX-19) between the Indian Army and 16
which is measured in Astronomical Unit (AU),as per the African nations was commenced in Pune, Maharashtra
current methodology. The study says mercury is at 0.387 on March 18, 2019 for training the participating
AU from earth while Venus is at 0.722 AU. contingents in Humanitarian Mine Assistance (HMA)
137. The organizers of the 2020 Summer Games in Tokyo have and Peace Keeping Operations (PKO) under the United
chosen a Japanese motif — the cherry blossom for their Nations Charter.It will be held in two locations in Pune
Olympic torch. The design, unveiled,makes use of not viz. Foreign Training Node at Aundh Military Station
only a traditional “Sakura-mon” blossom emblem but and College of Military Engineering in Kirkee.The Indian
also the same aluminum extrusion technology used in army is represented by Maratha Light Infantry while
the manufacture of the country’s well-known bullet African continent is comprised of 10 personnel, from
trains. It will generate five flames from a cluster of petal- each of the participating nations.The exercise is expected
shaped cylinders.The torch is made using the same to facilitate sharing of Indian Armed Forces experiences
cutting-edge technology as in the production of japan’s in United Nations Peace keeping operations with
bullet trains and is formed of a seamless, single piece of countries in Africa and shall remarkably boost relations
aluminium. It measures 71cm in length and weighs with the African continent.
1.2kg. 144. Greta Thunberg, the Swedish schoolgirl who has
138. Former Supreme Court judge Mr.Satya Brata Sinha died inspired an international movement to fight climate
at the age of 75 in New Delhi. His native was change, has been nominated as a candidate to receive this
Dhanbad,Jharkhand.He was serving as Supreme Court year’s Nobel Peace Prize. The 16-year-old was nominated
judge from October 3, 2002 until August 8, 2009. by three Norwegian MPs. If she were to win, she would
139. Superfan Naotoshi Yamada, famous in Japan for having be the youngest recipient since Pakistan’s Malala
been to every Summer Games since 1964, has died aged Yousafzai, who was 17 when she received the prize.
92 with an unfulfilled dream of watching the Olympics 145. On 15th March 2019, 3 astronauts, Nick Hague and
when it returns to Tokyo next year. A Japanese Olympic Christina Koch from US and Alexey Ovchinin from
mega-fan who attended every summer games since Russia arrived at the International Space Station (ISS) on
Tokyo in 1964 has died, just over a year before his home Russian Soyuz spacecraft which was launched from
city was to host its second Olympics.The Tokyo Kazakhstan’s Baikonur Cosmodrome on 14th March
businessman was a national celebrity in his own right 2019. Soyuz spacecraft carries people and supplies to and
with his repeated, gleeful appearances in Olympic from International Space Station and it has become the
stands.

©LegalEdge Tutorials Page 70 of 76


Replication or other unauthorized use of this material is prohibited by the copyright laws of India
LE Prep Assist
sole means of transportation after the retirement of Space Affairs (MoHUA) to promote its Project ‘Sangam’ which
Shuttle in 2011. is developed to accelerate Swachh Bharat Mission (SBM)
146. On 18th March 2019, Afghanistan defeated Ireland by 7 in India. It is one of the world’s largest Cleanliness
wickets to win their first ever Test Match in Dehradun, Programme.The Microsoft’s Project Sangam is a cloud-
Uttrakhand. Rahmat Shah, who scored 98 and 76 runs in hosted, mobile-first community learning platform. It
1st and 2nd innings respectively was awarded the Player trained over 110000 municipal functionaries across over
of the Match.Afghanistan won the T-20 series against 4000 cities on Swachh Bharat e-Learning Portal.The aim
Ireland by 2-0 and ODI series was level at 2-2. of Swachh Bharat Mission is to clean up the streets, roads
Afghanistan is the newest ICC-member to play test and infrastructure of India’s urban and rural areas.
cricket and this was Afghanistan’s 2nd Test Match. 152. President Ram Nath Kovind awarded the ‘Nari Shakti
Afghanistan was badly defeated by the Cricketing Puraskar‘ to the renowned Kathak dancer Seema Mehta
behemoth, India in its debut test match. for her contribution towards women empowerment over
147. The United World Wrestling (UWW) has taken away the the last 15 years at Rashtrapati Bhawan in New Delhi as
Junior Asian Wrestling Championship from India and part of the Women’s Day celebrations. She began her
asked all its affiliated federations to stop dealing with the journey in Kathak and set up her dance school in
Wrestling Federation of India (WFI) due to the country’s Mumbai in 2010. She is the Director of ‘Chhandam Nritya
diplomatic tension with Pakistan.After the original hosts Bharati‘ in Mumbai and was actively involved in
of the event, Lebanon withdrew, India had agreed to host promotion of Kathak dance form.
the re-scheduled event in July 2019.However, after India 153. On 14th March 2019, Reserve Bank of India provided a
denied visas to a three-member shooting contingent from no objection to Kolkata headquartered Bandhan Bank for
Pakistan for the Shooting World Cup in Delhi following the proposed acquisition of Gruh Finance. Gruh Finance
the Pulwama terror attack, the International Olympic Limited is a Ahmedabad headquartered Subsidiary of
Committee had censured the Indian Olympic HDFC Limited, is a Housing Finance Company (HFC)
Association.It was followed by UWW asking all its recognized by the National Housing Bank (NHB) and it
affiliated federations to suspend all communication with was taken over in January by Kolkata-based Bandhan
WFI and it shifted the junior Asian championship from Bank in a share-swap deal.Based on this approval,
India to Thailand. Bandhan Bank has to transfer 14.9 per cent stake to HDFC
148. 29-year-old Indian athlete from Kerala, K.T. Irfan has for merging with Gruh Finance.The ratio for this
qualified for the Tokyo Olympics as he finished fourth in amalgamation will be 568 shares of Bandhan Bank for
the men’s 20 kilometer category at the Asian Race every 1000 shares of Gruh Finance.
Walking Championships in Nomi, Japan. He holds the 154. On 14th March 2019, The Election Commission has
national record of 1:20:21 which he achieved during his launched a mobile application ‘Observer App’ through
10th place finish at the 2012 Olympics in London.He also which the Electoral Observers can submit the relevant
qualified for 2019’s World Championships as he bettered document and material to Election Commission directly.
the qualifying mark of 1:22:30. The app will provide notifications, urgent messages and
149. The Internet of Things (IoT) India Congress, an event alerts to the poll observers. It will also help to get their
which will see digital technology leaders converge to deployment status, download the ID card and update
discuss and explore business opportunities in their profile.‘cVIGIL’ is a mobile application also
mainstreaming IoT, will be held in Bengaluru on August launched by Election Commission of India and it allowed
22-23, 2019. The IoT India Congress 2019 will include the citizens to report on violation of model code of
tracks on segments such as healthcare, manufacturing, conduct. A written observation can be made by the
telecom, smart cities, energy, retail, cyber security, skills observer after the flying squads investigate the matter.
and development, IoT Standards, legal and regulatory, 155. A section of the media has reported that a seven year old
and agriculture. boy from Malappuram District of Kerala is suffering
150. A coordinated operation was conducted by the armies of from a West Nile Virus (WNV), a mosquito-borne
India and Myanmar from 17th February to 2nd March disease, mostly reported in the continental United States.
2019 against insurgents in Myanmarese territory to avert Union Minister of Health and Family Welfare is closely
a possible threat to the Kaladan multi-modal transit monitoring the situation and has spoken to the State
transport project.The objective of the operation was to Health Minister of Kerala in this regard. He has directed
crack down on the members of the Arakan Army, an for all support to be extended to Kerala in its prevention
insurgent group in Myanmar, the members of which had and management. Secretary (HFW) held a meeting with
also moved close to the international border along Additional Chief Secretary Shri Rajeev Sadanandan,
Mizoram. The Kaladan multi-modal transit transport Kerala and reviewed the situation. The Health Ministry
project is being viewed as India’s gateway to the has dispatched a multi-disciplinary Central team from
Southeast Asia. India entered into a framework National Centre for Disease Control (NCDC). The
agreement with Myanmar in April 2008 to facilitate Central team includes Dr. Ruchi Jain, RHO Trivandrum,
implementation of the project. On completion, the project Dr Suneet Kaur, Assitant Director, NCDC, Dr E
will help connect Mizoram with the Sittwe Port in Rajendran, Entomologist, NCDC, Calicut and Dr Binoy
Rakhine State of Myanmar. Basu, EIS Officer, NCDC. The team will support the State
151. On 8th March 2019, Microsoft has announced that it has Health Authorities in managing the disease. The Indian
partnered with The Ministry of Housing and Urban Council of Medical Research (ICMR) has also been

©LegalEdge Tutorials Page 71 of 76


Replication or other unauthorized use of this material is prohibited by the copyright laws of India
LE Prep Assist
alerted and a close watch is being maintained at Central finance ministry, discussed the key issues facing the
and State level. There are no reports available so far for economy at the meeting of the sub-committee of the
spread of this virus in other parts of the country. Financial Stability and Development Council (FSDC)
156. India will host the U-17 Women’s World Cup in 2020 chaired by RBI Governor Shaktikanta Das.The sub-
after the FIFA Council meeting in Miami, USA on MArch committee reviewed the major developments on the
15. This will be the second FIFA tournament India will be global and domestic fronts that impinge on the financial
hosting, after the U-17 Men’s World Cup in 2017. The U- stability of the country.The panel discussed ways to
17 women’s tournament began in 2008, when New address challenges pertaining to the quality of credit
Zealand hosted it. Spain is the current champion and ratings; and inter-linkages between housing finance
they had beaten Mexico 2-1 in the final in Uruguay in companies and housing developers.It also deliberated on
2018. the interlinking of various regulatory databases and the
157. On 12th March 2019, the European Union expanded the National Strategy for Financial Inclusion.
blacklist by 10 Countries adding the United Arab 161. The Supreme Court appointed senior advocate P. S.
Emirates, Barbados and the Marshall Island. The list had Narasimha, as mediator for resolving various disputes
previously included five countries and now it became 15 related to cricket administration in the country.The
countries. Due to their inability towards reform bench also asked P. S. Narasimha to look into the dispute
commitments, countries including Aruba, Belize, relating to release of funds by the court appointed
Bermuda, Fiji, Oman, Vanuatu, and Dominica are moved committee of administrators to various cricket
from grey list to black list.Naming and Shaming tactics associations.It has restrained all other courts in India
through Tax blacklist or Tax havens done by European from entertaining for proceeding with any matter
Union as a tool for securing a level playing field and as pertaining to BCCI and state cricket associations.
an external strategy for effective taxation by assessing, 162. Special Olympics World Games 2019 began at Abu
screening and listing third-country tax jurisdictions Dhabi, United Arab Emirates (UAE) on 14th Match 2019
which are non-cooperative in tax matters. and made history by welcoming a record-breaking 200
158. On 14th March 2019, United Nations Environment nations at the Games comprising 195 participating and 5
Programme (UNEP) released sixth Global Environment observing. The Indian contingent was the second largest
Outlook 2019 calls decision makers to take immediate after the host UAE and marched proudly holding the
action to address environmental issues to achieve the tricolour.The event is being held in the Middle East for
Sustainable Development Goals as well as other the first time and 7,500 athletes will take part in 24
Internationally Agreed Environment Goals, such as the Olympic-style sports over seven days.Nine venues in
Paris Agreement. As per the report, a quarter of all Abu Dhabi and Dubai will accommodate 500,000
premature deaths and diseases worldwide are due to spectators, 1,500 officials will referee the Games, while
manmade pollution and environmental damage. The 3,000 coaches will assist the athletes. For the first time,
report notes that as greenhouse gas emissions continue 200 nations are being represented at the World Games,
to rise.It also warns that deadly emissions, chemicals 195 participating and 5 observing.
polluting drinking water, and the accelerating 163. Supreme Court set aside the life ban on Sreesanth
destruction of ecosystems crucial to the livelihoods of imposed by the Board of Control of Cricket in
billions of people are driving a worldwide epidemic that India(BCCI) on him for indulging in spot-fixing during
hampers the global economy.The growing divide 2013 Indian Premier League. The bench comprising
between rich and poor as uncontrolled Justice Ashok Bhushan and Justice KM Joseph has
overconsumption, pollution and food waste in the directed the disciplinary committee of BCCI to take a
developed world leads to hunger, poverty and disease decision on the quantum of punishment within 3 months.
elsewhere.The lack of access to clean drinking supplies 164. On 13th March 2019, The Supreme Court clarified its last
leads 1.4 million people die each year from preventable year order on police reforms and said officers who have
diseases such as diarrhea and parasites linked to a minimum of six months tenure left in service can be
pathogen-riddled water and poor sanitation. considered for the post of Director General of Police
159. The first shipment under the United Nations ‘Transports (DGP).Chief Justice Ranjan Gogoi headed the bench and
Internationaux Routiers’ (TIR) convention arrived in recommendation for the post of DGP by the Union Public
India from Afghanistan through Iran’s Chabahar Service Commission (UPSC) and preparation of the
Port.India had joined the TIR Convention (the United appointment panel should be purely on the basis of
Nations Customs Convention on International Transport merit.The emphasis is to select the best and to ensure a
of Goods under cover of TIR Carnets) on June 15, 2017.It minimum tenure of two years’ service of such officer who
allows goods to be outlined in a TIR carnet and sealed in is to be selected and appointed.
load compartments. Customs officials verify the carnet 165. On 13th March 2019, following the advice of the Ministry
without physical checking, enabling shipments to pass of Health and Family Welfare, atleast 12 states in India
through countries without being opened at borders. have banned Electronic Nicotine Delivery Systems
160. Markets regulator Securities and Exchange Board of (ENDS).Punjab, Maharashtra, Karnataka, Kerala, Bihar,
India (SEBI), insurance regulator Insurance Regulatory Uttar Pradesh, Jammu and Kashmir, Himachal Pradesh,
and Development Authority(IRDAI), and pension fund Tamil Nadu, Puducherry and Jharkhand have taken
regulator Pension Fund Regulatory and Development steps to ban the use of ENDS. ENDS emits nicotine, the
Authority (PFRDA), along with senior officials of the addictive component of tobacco products which made

©LegalEdge Tutorials Page 72 of 76


Replication or other unauthorized use of this material is prohibited by the copyright laws of India
LE Prep Assist
the Doctors claim e-cigarettes pose significant health were held in Oman in January 2015 and India in March
risks to users that are frighteningly similar to those of 2017 respectively.
conventional cigarette. 171. The entry or transit of Boeing 737 Max 8 aircraft has been
166. A new coin that features the Crimea or Kerch Strait banned by the Directorate General of Civil Aviation
Bridge has been released by Russia’s central bank, to (DGCA) in the Indian airspace from 1600 hrs IST or 1030
mark Crimea’s reunification with Russia.Two million UTC 13th March 2019 following the Ethiopian Airlines
new five ruble (about $0.08, €0.07) coins were pressed crash that killed 157 people. The airlines affected by this
which depict an outline of the peninsula which was are SpiceJet which has 13 jets of the model 8 variant and
annexed by Russia in 2014.This is second Jet Airways which has 5 jets of this variant.All the top 10
commemoration of Crimea, the first being a 200 ruble countries in the air passenger travel category, except the
note released in 2015 and depicted the desire of the United States and Japan have banned flights of the
people of Russia to see these symbols on notes. Boeing 737 MAX 8.
167. ‘Sirsi Supari’ cultivated in Yellapura, Siddapura and Sirsi 172. According to the latest report by the World Gold Council
talukas of Uttar Kannada, Karnataka, has received the (WGC), India, the world’s largest consumer of gold, has
Geographic Indication (GI) tag which is the first time in the 11th largest gold reserve, with the current holding
the areca nut sector.Totgars’ Cooperative Sale Society secured at 607 tonnes.The number one slot is occupied by
Ltd., Sirsi, is the registered proprietor of the GI and the the U.S., which has gold reserves of 8,133.5 tonnes,
Registrar of Geographical Indications, under the Union followed by Germany with 3,369.7 tonnes.International
government, Chennai issued the certificate to the society. Monetary Fund (IMF) is positioned at third rank on the
Its GI number is 464.According to it, the particular list with total gold reserves of 2,814 tonnes followed by
arecanut “is medium in size, somewhat flat and rounded Italy and France with reserves of 2,451 tonnes and 2,436
in shape, somewhat ash coloured, and has a hard seed.” tonnes respectively.Among Asian countries, China
168. On 13th March 2019, Pakistan Air Force (PAF) (1,864.3 tonnes) and Japan (765.2 tonnes) have more gold
successfully test-fired an indigenously developed “smart reserves as compared to India.The Reserve Bank of India
weapon” from a JF-17 Thunder fighter jet amid tensions (RBI), which added 6.5 tonnes of gold to its reserves in
between India and Pakistan. JF-17 was previously January 2019, taking its total holding forming part of its
referred as FC-1 Xiaolong which is a single-engine multi- foreign exchange reserves to 607 tonnes, is set to become
role light fighter was being jointly produced by Pakistan the 10th largest holder of gold worldwide, displacing the
with China for several years with engines supplied by Netherlands.
Russia.JF-17 Thunder fighter jet will provide it a very 173. Krishna Kumari Kohli has been appointed as the first
potent day and night capability to engage a variety of female senator of Pakistan from the Hindu Dalit
targets with pinpoint accuracy.The trial is a great community. She is also the first Thari Hindu woman to
milestone for the country as the weapon has been be elected to the Pakistan senate. On 8th March 2019,
developed, integrated and qualified solely through which was celebrated as the International Women’s Day,
indigenous efforts of Pakistani scientists and she chaired the session of the Upper House of
engineers.PAF is a branch of the Pakistan Armed Forces, Parliament.She has earlier worked as a crusader for the
tasked primarily with the aerial defence of Pakistan, with rights of bonded labourers in Pakistan. She’s natively
a secondary role of providing air support to the Pakistan from the village-Dhana Gam in Nagarparkar area of
Army and the Pakistan Navy. Sindh.
169. Central Government allowed State Governments to put 174. A new population of ultraviolet stars has been
to public use some enemy properties that were left discovered in the Globular Cluster NGC 2808 by Indian
behind by people who left India for Pakistan and China astronomers from Thiruvananthapuram and Mumbai,
after 1965 and 1971 wars.The estimated value of all using the Indian multi-wavelength space observatory
enemy properties is approximately one lakh crore AstroSat.AstroSat is India’s multi-wavelength space
rupees.There are over nine thousand such properties left observatory which allows to observe the universe in the
behind by Pakistani nationals and over one hundred by optical, ultraviolet, low and high energy X-ray regions of
Chinese nationals. the electromagnetic spectrum. It was launched in
170. The 3rd edition of the series of bilateral joint exercise September 2015. NGC 2808 is a very huge globular
between the armies of India and Oman, called Exercise cluster and is located 47,000 light years away from the
Al Nagah III, commenced on 12th March 2019 and will Earth.Globular cluster is a collection of millions of stars
continue till 25th March 2019 at Jabel Al Akhdar which move as a single unit.
Mountains in Oman where both the armies will be 175. Large Hadron Collider (LHC) is the world’s largest
exchanging expertise in tactics, weapon handling and machine and it will examine the universe’s tiniest
firing, in order to enhance inter operability in counter- particles.LHC is part of a project helmed by the European
terrorist operations in semi urban mountainous Organization for Nuclear Research, also known as
terrain.The Indian Army contingent will be represented CERN. The LHC will cause the beams to collide with
by 4 Officers, 9 Junior Commissioned Officers and 47 each other, and then record the resulting events caused
other ranks of 10th Battalion of the Garhwal Rifles and a by the collision.According to CERN, The new
similar strength will be showcased by Jabel Regiment of collider,named Future Circular Collider (FCC) to be built
Royal Army of Oman (RAO).The first two joint exercises near the LHC along the Swiss-French border, would
significantly expand scientists’ understanding of matter

©LegalEdge Tutorials Page 73 of 76


Replication or other unauthorized use of this material is prohibited by the copyright laws of India
LE Prep Assist
and the universe.The Future Circular Collider (FCC) significantly boost the capability of the artillery to make
would be four times bigger and up to 10 times more precision hits.
powerful than the LHC, the European Organization for 182. On 8th March 2019, National Pharmaceutical Pricing
Nuclear Research. Authority (NPPA) which is an independent regulator
176. A biopic on Mother Teresa, Founder of Missionaries of under the Ministry of Chemicals and Fertilizers, has
Charity, was announced.The film, titled ‘Mother Teresa: reduced the prices of 390 non-scheduled cancer
The Saint’ will be written and directed by Seema medicines which will result in annual saving of around
Upadhyay and will be released in 2020. The cast will Rs 800 crore for patients. The revised prices has come into
consist of both Bollywood and Hollywood actors. Mother effect from 8th March 2019. Manufactures and Hospitals
Teresa was awarded the Nobel Peace Prize in 1979. have to revise the prices based on the trade margin
177. 12th March 2019 marks the 89th anniversary of the Dandi formula.
Satyagraha or Salt Satyagraha, an act of nonviolent civil 183. On 9th March 2019, The Indian Sundarban was accorded
disobedience in colonial India led by Mahatma with the status of ‘Wetland of International Importance’
Gandhi.Mahatma Gandhi had undertaken a 26-day in the Ramsar Convention. Indian Sundarban which is
march from March 12, 1930 to April 6, 1930 as a direct also a UNESCO (The United Nations Educational,
action campaign of tax resistance and nonviolent protest Scientific and Cultural Organization) world heritage site,
against the British salt monopoly.The 400 km Dandi is home to the Royal Bengal Tiger.Indian Sundarban met
March started from Sabarmati Ashram. four out of nine criteria required for the status, which are
178. President of Costa Rica and Vice President Venkaiah presence of rare species and threatened ecological
Naidu made a joint press statement. Vice President also communities, biological diversity, significant and
conferred and honorary doctorate by the University of representative fish and fish spawning ground and
Peace founded by the United Nations for his migration path.Ramsar status will help to highlight
contributions “to the Rule of Law, democracy and conservation issues of the Sundarbans at the
sustainable development in India”. international level at the time when concerns have been
179. India lost its position as the world’s largest importer of raised about natural ecosystems being changed for
weapons, which it held for over a decade, with Saudi cultivation of shrimp, crab, molluscs and
Arabia topping the global share of arms imports between fish.International Importance also called Ramsar
2014 and 2018, as per the data published by the Convention, is an international agreement promoting the
Stockholm International Peace Research Institute (Sipri), conservation and wise use of wetlands. It is the only
the leading Stockholm-based think tank that measures global treaty which focus on a single ecosystem.
weapons imports over five-year periods. Saudi Arabia 184. The International Association of Athletics Federations
accounted for 12% of the global share of arms imports (IAAF) had banned Russia in November 2015 because of
between 2014-2018, followed by India with a 9.5% evidence of state-sponsored doping and have decided to
share.Indian arms imports fell 24% between 2009-13 and maintain it until samples and data from Moscow’s
2014-18 because of delays in deliveries of fighter jets and former anti-doping laboratory are made available.It is
submarines produced under licence from Russian and the 10th time the IAAF has turned down Russia’s appeal
French original equipment manufacturers. India’s top for reinstatement, having requested the same assurances
arms supplier, Russia, accounted for 58% of India’s arms when it upheld the ban in December 2018.Other sporting
imports during the last 5 years, compared to 76% in the bodies such as the World Anti-Doping Agency (Wada)
previous five-year period.US, Israel and France increased and the International Olympic Committee (IOC), have
their arms exports to India in 2014-18. revoked their suspensions of Russia.
180. aormer Reserve Bank of India (RBI) Governor Raghuram 185. Ace gymnast Dipa Karmakar, hailing from Tripura had a
Rajan, for his contribution towards economic Barbie doll modelled after her in the list of honours
development, was awarded the ‘Yashwantrao Chavan achieved which includes – Arjuna Award in 2015, Rajiv
National Award 2018‘ on March 12, 2019, which was the Gandhi Khel Ratna Award in 2016, Padma Shri award in
106th birth anniversary of late Yashwantrao Chavan, the 2017. On the 60th anniversary of Barbie, the makers has
first chief minister of Maharashtra. i.The award is given released a new line of dolls to honour 20 female role
annually to individuals or institutions for their models across 18 countries speaking 14 different
outstanding contribution towards national integration. languages.
181. Defence Research and Development Organisation 186. The first-ever International Beach Volleyball
(DRDO) successfully test fired the Guided Pinaka Tournament is creating a buzz on the windswept shores
weapons system from Pokhran ranges in Rajasthan. The of Visakhapatnam. Ramakrishna Beach is now equipped
two rockets, which were tested using the guided Pinaka with floodlights, white sand filling (on the court), a
system, succeeded in hitting the 90-km mark signalling a seating capacity of 5,000 , and shower facility for players
boost in the efficiency, as the rockets had touched the 70- besides other supporting infrastructure. The tournament
km mark in their last trials, held in May, 2018.The is promoted by Leisure Sports Management in
weapon system is equipped with state-of-the-art collaboration with International Volleyball Federation,
guidance kit comprising of an advanced navigation and Volleyball Federation of India and AP and Vizag
control system.The Telemetry Systems tracked and Volleyball Association. Fifty-six teams from different
monitored the vehicle all through the flight path.The countries are vying for the title as they will battle wind,
indigenously developed Guided Pinaka by DRDO will sun, rain and each other. Czech Republic’s Martina

©LegalEdge Tutorials Page 74 of 76


Replication or other unauthorized use of this material is prohibited by the copyright laws of India
LE Prep Assist
Bonnerova and Martina Maixnerova concluded their Institute of Archaeology and unveiled a statue of Pandit
FIVB Beach Volleyball World Tour Vizag Open Deendayal Upadhyaya at the campus.Modi inaugurated
campaign with a gold medal. Armin Dollinger and Noida City Centre – Noida Electronic City Section of the
Simon Kulzer of Germany claimed their first ever FIVB Delhi Metro which will provide a convenient and faster
World Tour gold medal. mode of transport for the residents.
187. On 10th March 2019, Prime Minister Narendra Modi 192. For the amicable settlement of Ayodhya’s Ram
attended the 50th Raising Day of Central Industrial Janmabhoomi-Babri Masjid land dispute case, the five-
Security Force (CISF) at CISF Camp at 5th Battalion judge bench, headed by the Chief Justice of India (CJI)
Camp at Indirapuram in Ghaziabad.He also presented Ranjan Gogoi, constituted a three-member mediation
the Police and Fire Service Medals for distinguished and panel headed by former Supreme Court (SC) judge
meritorious services and laid wreath at the Martyr’s Justice F. M. Kalifullah to resolve the issue.The other
Memorial.President’s Medal for Distinguished Service members of the panel include spiritual guru Sri Sri Ravi
and Jeewan Raksha Padak were awarded to Sudhir Shankar and Senior Advocate Sriram Panchu.The
Kumar, IG (NCR), J S Negi, DIG (I&P), Inspector S proceedings in the case will be held in Faizabad.The
Muthusamy and Constable R Surya.History Book of CISF panel of mediators will file a progress report of the
“CISF: Force of the Future” and Annual Publication mediation proceedings within 4 weeks and the process
“Special Issue of Sentinel” were released by PM will be completed within 8 weeks.They may co-opt more
Narendra Modi. members and in case of any difficulty, inform the
188. Finland’s Prime Minister Juha Sipila, along with his Supreme Court registry.14 appeals have been filed in the
cabinet, resigned after failing to push through parliament Supreme Court against the 2010 Allahabad High Court
plans to overhaul health services and social care in the judgment, delivered in 4 civil suits, that the 2.77-acre land
face of an aging population, just a few weeks before the in Ayodhya be partitioned equally among the 3 parties –
general elections.He tendered his resignation to the Sunni Waqf Board, the Nirmohi Akhara and Ram
President Sauli Niinisto, who asked him to stay on as Lala.
caretaker until the elections in April 2019.Statistics 193. On 7th March 2019, The Insolvency and Bankruptcy
Finland has estimated that more than a quarter of the Board of India (IBBI) signed an agreement with the
population will be aged over 65 by 2030, making a more International Finance Corporation (IFC)in cooperation.
cost-effective welfare state a top priority.The Prime IFC is a member of the World Bank Group.The
Minister’s proposed solutions included creating regional cooperation agreement provides a technical assistance by
authorities for health and welfare services, rather than IFC to IBBI until 30th June, 2021.It will provide training
the local municipalities that currently manage the of insolvency professionals and trainers by the IFC to
system, and offering including private companies in the IBBI.IBBIprovides a platform for re-organization and
healthcare system to a greater extent to offer “freedom of insolvency resolution of corporate persons, individuals
choice”.The plan involved a drastic reorganization of the and partnership firmsin a time bound manner for
way health care is administered at a local level as well as maximization of the value of assets.IFC would help IBBI
a greater role for the private sector. to effectively implement the Insolvency and Bankruptcy
189. The President of Palestinian Authority Mahmoud Abbas Code, 2016 and its allied rules and regulations for the
appointed Mohammad Shtayyeh as the Palestinian purpose of the bankruptcy code.
Prime Minister. Mohammad Shtayyeh succeeds Rami al- 194. On 9th March 2019, a bilingual correspondent with the
Hamdallah who resigned from this post 6 weeks ago.He BBC at its Delhi bureau, Priyanka Dubey won the
is 61 years old and is a member of the central committee Chameli Devi Jain Award 2018 as outstanding women
of the president’s Fatah party.He is a former minister journalist. She was chosen for her multi-faceted,
who had earlier participated in numerous negotiating interrogative and investigative reports. She helped to
teams of Palestine during United States-brokered talks uncover the reality underlying in the social and political
with Israel. issues.Chameli Devi Jain Award has been awarded to
190. The Marshall Islands is planning to release a digital women journalist for upholding standards of excellence
currency called “SOV” (Sovereign) in 2019, however, the through a sustained body of work for the past 37 years.
launch date of the currency has not been decided. The award was presented at a function in Delhi.She was
Neema, an Israel-based company, is partnering with the a freedom fighter. This award is the countries longest
Marshall Islands government to develop the digital running media award for women. The jury comprised
currency.With SOV, every account would be fully Vandita Mishra, National Opinion Editor in Indian
identified and buyers would be checked against the US Express, Bharat Bhushan, former editor in NewsX and
Office of Foreign Asset Control so that only legitimate, Sheela Bhatt, National editor of NewsX.
law-abiding people can use the currency. 195. On 9th March 2019, Subhash Chandra Garg, the
191. On 8th March 2019, Prime Minister Narendra Modi Economic Affairs Secretary was appointed as the Finance
embarked on a visit to Kanpur, Ghaziabad and Varanasi Secretary by the Personnel Ministry. His appointment
in Uttar Pradesh. He laid foundation stone for various was approved the by the Appointments Committee of
development projects and schemes during his visit. PM the Cabinet which is led by the Prime Minister Narendra
Modi during his visit to UP address a gathering and Modi. He is an IAS officer from the 1983 batch and
unveiled several development projects in Greater belongs to the Rajasthan cadre. He has succeeded Ajay
Noida.PM inaugurated Pandit Deendayal Upadhyaya Narayan Jha, the former Finance Secretary who retired

©LegalEdge Tutorials Page 75 of 76


Replication or other unauthorized use of this material is prohibited by the copyright laws of India
LE Prep Assist
on February 28, 2019 and is a member of the 15th Finance Hyderabad and Madhya Pradesh comes under the
Commission. jurisdiction of NCLT Bench, Ahmedabad.The NCLT,
196. On 10th March 2019, China lifted its broadband which has been set up under the Companies Act, 2013,
communication system by launching a Long March – 3B has provided an effective and time-bound adjudication
carrier rocket with ‘ChinaSat 6C’ into a geostationary mechanism to deal with matters related to the
orbit in Sichuan province. Key Points i. The‘ChinaSat 6C’ Companies Act, 2013, the Insolvency and Bankruptcy
was launched from Xichang Satellite Launch Center in Code (IBC), 2016 and the LLP Act, 2008.Presently, 14
Sichuan province. It is a 19 story tall rocket which is numbers of NCLT Benches have been established,
expected to provide high quality radio and television including the Principal Bench in New Delhi, and three
transmission services. ii. It marked 300th launch of the recently set up benches at Jaipur, Kochi and Cuttack.
Long March carrier rocket series which was developed 199. On 7th March 2019, The Reserve Bank of India has eased
by the China Aerospace Science and Technology business guidelines for white label ATMs by allowing
Corporation and will be operated by China Satellite companies that manage these machines to source cash
Communications Co. Ltd. iii. The satellite will cover directly from the central bank.The guidelines released by
China, Australia, New Zealand and the South Pacific RBI include managing machines to source cash directly
Island Countries. Nearly 97% of China’s total launch from RBI offices, offer non bank services like bill
missions was on Long March rocket series. iv. China has payment and advertise even non financial products in
built 17 types of Long March rocket with five of them their premises.As per the statement, RBI has allowed the
being retired. The first carrier rocket A Long March-1 operators to source cash from any Scheduled banks in
with first satellite Dong fanghong 1 launched by China in India including cooperative and rural banks.RBI allowed
1970. White Label ATM Operators (WLAO) to offer bill
197. On 7th March 2019, India entered into a deal with Russia payment and deposit services depending on the technical
for leasing of a nuclear – powered attack submarine for feasibility of the device as determined by NPCI. The
Indian Navy worth USD 3 billion for a period of 10 years. ATMs setup, owned and operated by non-bank Entities
Under the pact, Russia will deliver Chakra III, a Akula are called White Label ATMs (WLAs). They provide
class submarine to the Indian Navy by 2025. It will be the banking services to customers of banks based on the
3rd Russian submarine to be leased to Indian Navy.India debit/credit/prepaid cards issued by banks.
earlier leased christened INS Chakra which is the first 200. On 8th March 2019, On the evening of International
Russian nuclear powered submarine in 1988 under a 3 Women’s Day United Nations Development Programme
year lease. The second lease was INS Chakra in 2012 for (UNDP) appointed Padma Lakshmi as its goodwill
a period of 10 years.The lease period as per the ambassador. She will have the responsibility to support
agreement for Chakra II will expire in 2022 and the the agency’s fight against inequality and discrimination
Indian government is looking at extending the lease which can affect people in rich and poor countries
period. alike.Padma Lakshmi is an Indian – American television
198. The Government approved the establishment of two new personality, supermodel, food expert, author and
benches of National Company Law Tribunal (NCLT), at executive producer of Top Chef.She has been a judge and
Amaravati in Andhra Pradesh and Indore in Madhya host as well as executive producer of Bravo Television’s
Pradesh in view of the increasing caseload, especially Emmy award winning series Top Chef.She is the co-
under the Insolvency & Bankruptcy Code 2016. The founder of the Endometriosis Foundation of America, a
creation of new benches will enable faster disposal of visiting scholar at the Massachusetts Institute of
cases.The jurisdiction of the Bench at Amaravati will be Technology and an ambassador for the American Civil
the state of Andhra Pradesh and that of Indore will be the Liberties Union advocating for immigrants rights.
state of Madhya Pradesh.Currently, Andhra Pradesh
comes under the Jurisdiction of NCLT Bench at

©LegalEdge Tutorials Page 76 of 76


Replication or other unauthorized use of this material is prohibited by the copyright laws of India
Lucknow Centre: #21, Ashok Marg , Opp Doordarshan Kendra, Hazratganj, Lucknow 226001. Contact +91 731-0100993.

You might also like